You are on page 1of 178

GATE Previous Year

Solved Paper
Civil Engineering
(Fully Solved)

GATExplore.com

(2015, 2016, 2017)


Free Download
CE-GATE-2015 PAPER| www.gateforum.com

GATE 2015 –CE on 8th February, 2015 – (Forenoon Session)

General Aptitude Questions


Q.No-1-5 Carry One Mark Each

1. Select the pair that best expresses a relationship similar to that expressed in the pair:
Children: Pediatrician
(A) Adult: Orthopaedist (B) Females: Gynaecologist
(C) Kidney: Nephrologist (D) Skin: Dermatologist
Answer: (B)
Exp: Community of people: Doctor

2. Extreme focus on syllabus and studying for test has become such a dominant concern of
Indian students that this has closed their minds to anything ________ to the requirements of
the exam
(A) related (B) extraneous (C) outside (D) useful
Answer: (B)
Exp: extraneous -irrelevant or unrelated to the subject being dealt with.

3. If ROAD is written as URDG, then SWAN should be written as:


(A) VXDQ (B) VZDQ (C) VZDP (D) UXDQ
Answer: (B)
Exp: R+3=U, O+3=R, A+3=D, D+3=G;
S+3=V, W+3=Z, A+3=D, N+3=Q

4. The Tamil version of __________ John Abraham-starrer Madras Café _____ cleared by the
censor board with no cuts last week, but the film’s distributors _______ no takers among the
exhibitors for a release in Tamil Nadu ________ this Friday.
(A) Mr., was, found, on (B) a, was found, at
(C) the, was, found, on (D) a, being, find, at
Answer: (C)
Exp: John-Abraham starrer Madras Café talks about the movie not the person, so Mr. is ruled out.
‘Find no takers’ is not the correct phrase. At this Friday is incorrect. So, option C is correct.

5. A function f(x) is linear and has a value of 29 at x = -2 and 39 at x = 3. Find its value at x = 5.
(A) 59 (B) 45 (C) 43 (D) 35
Answer: (C)
Exp: f(x)=2x+33

 India’s No.1 institute for GATE Training  1 Lakh+ Students trained till date  65+ Centers across India
1
CE-GATE-2015 PAPER| www.gateforum.com

Q.No-6-10 Carry Two Marks Each

6. The head of a newly formed government desires to appoint five of the six selected members
P,Q,R,S,T and U to portfolios of Home, Power, Defense, Telecom and Finance. U does not
want any portfolio if S gets one of the five. R wants either Home or Finance or no portfolio.
Q says that if S gets either Power or Telecom, then she must get the other one. T insists on a
portfolio if P gets one.
Which is the valid distribution of portfolio?

(A) P-Home, Q-Power, R-Defense, S-Telecom, T-Finance


(B) R-Home, S-Power, P-Defense, Q-Telecom, T-Finance
(C) P-Home, Q-Power, T-Defense, S-Telecom, U-Finance
(D) Q-Home, U-Power, T-Defense, R-Telecom, P-Finance
Answer: (B)
Exp: Since U does not want any portfolio, (C) and (D) are ruled out. R wants Home, or Finance or
No portfolio, (A) is not valid. Hence option (B) is correct

7. The exports and imports (in crores of Rs.) of a country from the year 2000 to 2007 are given
in the following bar chart. In which year is the combined percentage increase in imports and
exports the highest?

120
Exports Imports
110
100
90
80
70
60
50
40
30
20
10
0

2000 2001 2002 2003 2004 2005 2006 2007

Answer: 2006
100 − 70
Exp: Increase in exports in 2006= = 42.8%
70
120 − 90
Increase in imports in 2006= = 33.3%
90
which is more than any other year

 India’s No.1 institute for GATE Training  1 Lakh+ Students trained till date  65+ Centers across India
2
CE-GATE-2015 PAPER| www.gateforum.com

8. Most experts feel that in spite of possessing all the technical skills required to be a batsman of
the highest order., he is unlikely to be so due to lack of requisite temperament. He was guilty
of throwing away his wicket several times after working hard to lay a strong foundation. His
critics pointed out that until he addressed to this problem, success at the highest level will
continue to elude him.
Which of the statement (s) below is/are logically valid and can be inferred from the above
passage?
(i) He was already a successful batsman at the highest level
(ii) He has to improve his temperament in order to become a great batsman
(iii) He failed to make many of his good starts count
(iv) Improving his technical skills will guarantee success
(A) (iii) and (iv) (B) (ii) and (iii) (C) (i), (ii) and (iii) (D) (ii) only
Answer: (B)

9. Choose the most appropriate equation for the function drawn as a thick line, in the plot below.

( 2,0) π

( 0, −1)

(A) x = y-|y| (B) x = -(y-|y|) (C) x = y+|y| (D) x = -(y+|y|)


Answer: (B)

10. Alexander turned his attention towards India, since he had conquered Persia.
Which one of the statements below is logically valid and can be inferred from the above
sentence?
(A) Alexander would not have turned his attention towards India had he not conquered
Persia.
(B) Alexander was not ready to rest on his laurels, and wanted to march to India
(C) Alexander was completely in control of his army and could command it to move towards
India.
(D) Since Alexander’s kingdom extended to Indian borders after the conquest of Persia, he
was keen to move further.
Answer: (A)

 India’s No.1 institute for GATE Training  1 Lakh+ Students trained till date  65+ Centers across India
3
CE-GATE-2015 PAPER| www.gateforum.com

Section Name: Civil Engineering


Q.No-1-25 Carry One Mark Each

1. Consider the following statements for air-entrained concrete:


(i) Air-entrainment reduces the water demand for a given level of workability
(ii) Use of air-entrained concrete is required in environments where cyclic freezing and
thawing is expected.
Which of the following is TRUE?
(A) Both (i) and (ii) are True (B) Both (i) and (ii) are False
(C) (i) is True and (ii) is False (D) (i) is False and (ii) is True

Answer: (A)
Exp: (i) Air-entrainment reduces the water demand for a given level of workability-True
(ii) Use of air-entrained concrete is required in environments where cyclic freezing and
thawing is expected. -True

2. Which of the following statements is TRUE for the relation between discharge velocity and
seepage velocity?
(A) Seepage velocity is always smaller than discharge velocity
(B) Seepage velocity can never be smaller than discharge velocity
(C) Seepage velocity is equal to the discharge velocity
(D) No relation between seepage velocity and discharge velocity can be established
Answer: (B)
V
Exp: Vs = ∵ n <1
n
So, VS > V always
So, seepage velocity (VS) can never by smaller than discharge velocity

x2
3. The integral ∫
x1
x 2 dx with x 2 > x1 > 0 is evaluated analytically as well as numerically using a
single application of the trapezoidal rule. If I is the exact value of the integral obtained
analytically and J is the approximate value obtained using the trapezoidal rule, which of the
following statements is correct about their relationship?
(A) J > 1
(B) J<1
(C) J = 1
(D) Insufficient data to determine the relationship
Answer: (A)

 India’s No.1 institute for GATE Training  1 Lakh+ Students trained till date  65+ Centers across India
4
CE-GATE-2015 PAPER| www.gateforum.com

b
Exp: We know that the approximated value of ∫
a
f (x)dx obtained by trapezoidal rule is always
greater than the analytical value.
∴ J > I where J=approximate value
I=analytical value

4. A circular pipe has a diameter of 1m, bed slope of 1 in 1000, and Manning’s roughness
coefficient equal to 0.01. It may be treated as an open channel flow when it is flowing just
full, i.e., the water level just touches the crest. The discharge in this condition is denoted by
Qfull. Similarly, the discharge when the pipe is flowing half-full, i.e., with a flow depth of
0.5m, is denoted by Qhalf. The ratio Qfull/Qhalf is:
(A) 1 (B) 2 (C) 2 (D) 4

Answer: (C)
2
1 2 1
1 π  D 3
1
Exp: Q = .AR 3 S 2 ⇒ Q full = . .D 2 .  s 2
n n 4 4
2
1 πD 2  D  3 12
Q half = . .  s
n 8 4
Q full
=2
Q half

5. Which of the following statements is NOT correct?


(A) Loose sand exhibits contractive behavior upon shearing
(B) Dense sand when sheared under undrained condition, may lead to generation of negative
pore pressure
(C) Black cotton soil exhibits expansive behavior
(D) Liquefaction is the phenomenon where cohesionless soil near the downstream side of
dams or sheet-piles loses its shear strength due to high upward hydraulic gradient
Answer: (D)
Exp: Liquefaction is due to cyclic loads, not due to high hydraulic gradient

6. A fine-grained soil has 60% (by weight) silt content. The soil behaves as semi-solid when
water content is between 15% and 28%. The soil behaves fluid-like when the water content is
more than 40%. The ‘Activity’ of the soil is
(A) 3.33 (B) 0.42 (C) 0.30 (D) 0.20
Answer: (C)
Exp: IP = WL-WP = 40-28 = 12%
Ip 12
Activity = = = 0.3
C 100 − 60

 India’s No.1 institute for GATE Training  1 Lakh+ Students trained till date  65+ Centers across India
5
CE-GATE-2015 PAPER| www.gateforum.com

7. For steady incompressible flow through a closed-conduit of uniform cross-section, the


direction of flow will always be:
(A) from higher to lower elevation (B) from higher to lower pressure
(C) from higher to lower velocity (D) from higher to lower piezometric head
Answer: (B)

8. Two triangular wedges are glued together as shown in the following figure. The stress acting
normal to the interface, σn is __________ MPa.
Answer: 0
σx + σy σx − σy
Exp: σn = + cos 2θ
2 2 100MPa

100 − 100 100 − (−100)


= + cos90 = 0
2 2 σn

100MPa 100MPa
45°

100MPa

9. In a closed loop traverse of 1 km total length, the closing errors in departure and latitude are
0.3 m and 0.4 m, respectively. The relative precision of this traverse will be;
(A) 1:5000 (B) 1:4000 (C) 1:3000 (D) 1:2000
Answer: (D)

e = 2 + d2 = ( 0.3) + ( 0.4 ) = 0.5 m


5 2
Exp:
0.5
Relative precision = = 1: 2000
1000

10. Solid waste generated from an industry contains only two components, X and Y as shown in
the table below
Component Composition Density
( % weight ) ( kg m3 )
X c1 ρ1
Y c2 ρ2
Assuming (c1+c2) = 100, the composite density of the solid waste (ρ) is given by:
100 ρ ρ 
(A) (B) 100  1 + 2  (C) 100 ( c1ρ1 + c 2ρ2 ) (D) N77o 50′E
 c1 c 2   c1 c2 
 + 
 ρ1 ρ2 
Answer: (A)

 India’s No.1 institute for GATE Training  1 Lakh+ Students trained till date  65+ Centers across India
6
CE-GATE-2015 PAPER| www.gateforum.com

Exp: Let density of sludge is ρ


c1 + c2 c1 c 2
= +
ρ ρ1 ρ2
100
⇒ρ=
c1 c 2
+
ρ1 ρ2

11. The two columns below show some parameters and their possible values.
Parameter Value
P-Gross Command Area I-100 hectares/cumec
Q-Permanent Wilting Point II-6℃
R-Duty of canal water III-1000 hectares
S-Delta of wheat IV-1000 cm
V-40 cm
VI-0.12
Which of the following options matches the parameters and the values correctly?
(A) P-I, Q-II, R-III, S-IV (B) P-III, Q-VI, R-I, S-V
(C) P-I, Q-V, R-VI, S-II (D) P-III, Q-II, R-V, S-IV
Answer: (B)
Exp:
P-Gross Command Area=1000 ha
Q-Permanent Wilting Point=0.12
R-Duty of canal water=100 ha/cumec
S-Delta of wheat=40 cm

12. In an unconsolidated undrained triaxial test, it is observed that an increase in cell pressure
from 150 kPa to 250 kPa leads to a pore pressure increase of 80 kPa. It is further observed
that, an increase of 50 kPa in deviatoric stress results in an increase of 25 kPa in the pore
pressure. The value of Skempton’s pore pressure parameter B is;
(A) 0.5 (B) 0.625 (C) 0.8 (D) 1.0
Answer: (C)
∆U 80
Exp: B= = = 0.8
∆σ3 100

 India’s No.1 institute for GATE Training  1 Lakh+ Students trained till date  65+ Centers across India
7
CE-GATE-2015 PAPER| www.gateforum.com

13. Which of the following statements is TRUE for degree of disturbance of collected soil
sample?
(A) Thinner the sampler wall, lower the degree of disturbance of collected soil sample
(B) Thicker the sampler wall, lower the degree of disturbance of collected soil sample
(C) Thickness of the sampler wall and the degree of disturbance of collected soil sample are
unrelated
(D) The degree of disturbance of collected soil sample is proportional to the inner diameter of
sampling tube
Answer: (A)
Exp: As thickness of sampler increases, disturbance increases

14. Which of the following statements is FALSE?


(A) Plumb line is along the direction of gravity
(B) Mean Sea Level (MSL) is used as a reference surface for establishing the horizontal
control
(C) Mean Sea Level (MSL) is a simplification of the Geoid
(D) Geoid is an equi-potential surface of gravity
Answer: (B)
Exp: Mean Sea Level (MSL) is used as a reference surface for establishing the vertical control and
not horizontal control

15. For what value of p the following set of equations will have no solution?
2x+3y = 5
3x+py = 10
Answer: 4.5
Exp: Given 2x+3y=5
3x + py = 10
 2 3   x  5 
⇒   =  
3 p   y  10 
AX = B
2 3 5 
Augmented matrix [A / B] =  
3 p 10 
2 3 5
R 2 → 2R 2 − 3R 1  
0 2p − 9 5
system will have no solution if ρ(A / B) ≠ ρ(A)
⇒ 2p − 9 = 0
9
⇒p= = 4.5
2

16. In a two-dimensional steady flow field, in a certain region of the x-y plane, the velocity
1
component in the x-direction is given by v x = x 2 and the density varies as ρ = . Which of
x
the following is a valid expression for the velocity component in the y-direction, vy?
(A) v y = − x / y (B) v y = x / y (C) v y = − xy (D) v y = xy
 India’s No.1 institute for GATE Training  1 Lakh+ Students trained till date  65+ Centers across India
8
CE-GATE-2015 PAPER| www.gateforum.com

Answer: (C)
Exp: Continuity equation
∂ ∂
( ρY ) + ( ρV ) = 0
∂x ∂y
∂ ∂ 1
⇒ ( x ) +  .V  = 0
∂x ∂y  x 
∂ V ∂ V
⇒1+   = 0 ⇒   = −1
∂y  x  ∂y  x 
⇒ V = − xy

17. Workability of concrete can be measured using slump, compaction factor and Vebe time.
Consider the following statements for workability of concrete:
(i) As the slump increases, the Vebe time increases
(ii) As the slum increases, the compaction factor increases
Which of the following is TRUE?
(A) Both (i) and (ii) are True (B) Both (i) and (ii) are False
(C) (i) is True and (ii) is False (D) (i) is False and (ii) is True
Answer: (D)
Exp: As the slump increases, the Vebe time decreases

18. Consider the following probability mass function (p.m.f) of a random variable X:
q if X = 0

p(x,q) = 1 − q if X = 1
0 otherwise

If q=0.4, the variance of X is ________ .
Answer: 0.24
Exp: p(x,q) = q if X = 0
= 1 − q if X = 1
0 otherwise
given q = 0.4
⇒ p ( x,q ) = 0.4 if X = 0
= 0.6 if X = 1
=0 otherwise
X 0 1

p ( X = x ) 0.4 0.6
Required value = V ( X ) = E ( X 2 ) − {E ( X )}
2

E ( X ) = Σx i pi = 0 × 0.4 + 1 × 0.6 = 0.6


E ( X 2 ) = Σx i2 pi = 02 × 0.4 + 12 × 0.6 = 0.6

V ( X ) = 0.6 − ( 0.6 )
2

= 0.6 − 0.36
= 0.24
 India’s No.1 institute for GATE Training  1 Lakh+ Students trained till date  65+ Centers across India
9
CE-GATE-2015 PAPER| www.gateforum.com

19. Which of the following statements CANNOT be used to describe free flow speed (uf) of a
traffic stream?
(A) uf is the speed when flow is negligible
(B) uf is the speed when density is negligible
(C) uf is affected by geometry and surface conditions of the road
(D) uf is the speed at which flow is maximum and density is optimum
Answer: (D)
Exp: Free flow speed → speed when flow is negligible
→ speed when density is negligible
→ affected by Geometry, deriver’s perception, roadway condition etc.

20. Consider the singly reinforced beam shown in the figure below:
P P X

L L/2 L

At cross-section XX, which of the following statement is TRUE at the limit state?
(A) The variation of stress is linear and that of strain is non-linear
(B) The variation of strain is linear and that of stress is no-linear
(C) The variation of both stress and strain is linear
(D) The variation of both stress and strain is non-linear
Answer: (B)
Exp: At

Strain variation Stress variation

 India’s No.1 institute for GATE Training  1 Lakh+ Students trained till date  65+ Centers across India
10
CE-GATE-2015 PAPER| www.gateforum.com

21. For the beam shown below, the stiffness coefficient K22 can be written as
2
Note :1, 2 and 3
L are the d.o.f
3

A, E, I 1

6EI 12EI 3EI EI


(A) (B) (C) (D)
L2 L3 L 6L2
Answer: (B)
Exp:
2
∆ 3
6EI∆ 1
L2
12EI∆
L3
12EI
K 22 =
L3

22. The penetration value of a bitumen sample tested at 25o C is 80. When this sample is heated
to 60o C and tested again, the needle of the penetration test apparatus penetrates the bitumen
sample by d mm. The value of d CANNOT be less than ____ mm.
Answer: 8

23. The development length of a deformed reinforcement bar can be expressed as


(1 / k)(φσs / τbd ). From the IS:456-2000, the value of k can be calculated as ____ .
Answer: 6.4
φσs
Exp: Ld = But for deformed bars τbd is increased by 60%.
4τ bd
So,
φσst φσs
Ld = =
4 × 1.6 × τbd 6.4 τbd
So, k=6.4.

 India’s No.1 institute for GATE Training  1 Lakh+ Students trained till date  65+ Centers across India
11
CE-GATE-2015 PAPER| www.gateforum.com

24. Total Kjeldahl Nitrogen (TKN) concentration (mg/L as N) in domestic sewage is the sum of
the concentrations of:
(A) organic and inorganic nitrogen in sewage
(B) organic nitrogen and nitrate is sewage
(C) organic nitrogen and ammonia is sewage
(D) ammonia and nitrate in sewage
Answer: (C)
Exp: Total Kjeldahl Nitrogen (TKN) = Ammonia (60%) + Organic Nitrogen (40%)

25. For the beam shown below, the value of the support moment M is ____ kN-m.
20 kN

M 3m 1m 1m 3m

EI EI EI

Internal hinge

Answer: 5
10kN
Exp:

3m 1m

10×1
= 5kN−m
2

Q.No-26-55 Carry Two Marks Each

26. The directional derivative of the field u(x,y,z) = x 2 − 3yz in the direction of the vector
(iˆ + ˆj − 2k)
ˆ at po int (2, −1, 4) is ____ .

Answer: -5.72

 India’s No.1 institute for GATE Training  1 Lakh+ Students trained till date  65+ Centers across India
12
CE-GATE-2015 PAPER| www.gateforum.com

Exp: Let u(x,y,z) = x2-3yz



a = i + j − 2k and P ( 2, −1, 4 )
∂u ∂u ∂u
∇u = i + j +k
∂x ∂y ∂z
= i2x + j ( −3z ) + k ( −3y )
∇u ( 2,−1,4) = 4i − 12 j + 3k

a = 1+1+ 4 = 6

directionalderivative = ∇u.a

= ( 4i − 12 j + 3k ) .
( i + j − 2k )
6
4 − 12 − 6
=
6
−14
= = −5.72
6

27. For formation of collapse mechanism in the following figure, the minimum value of Pu is
cMp/L. M p and 3M p denote the plastic moment capacities of beam sections as shown in this
figure. The value of c is ____ .
Pu
1m 1m

Mp
3 Mp

2m

Answer: 13.33
Pu
Exp: Mechanism-I MP
L
3M P . θ + M P ( 2θ ) + MP.θ = PU × ×θ 3M P θ θ
4
L
⇒ 6M P .θ = PU . .θ
4 2θ
MP
MP
⇒ PU = 24
L

 India’s No.1 institute for GATE Training  1 Lakh+ Students trained till date  65+ Centers across India
13
CE-GATE-2015 PAPER| www.gateforum.com

Mechanism-II
1.φ = 3.θ ⇒ φ = 3θ Pu
MP
L
3M P .θ + M P ( θ + φ ) + M P .φ = PU × φ
4 3M P θ φ
L
⇒ 3M P θ + M P ( θ + 3θ ) + M P .3θ = PU × 3θ ×
4
L θ+φ
⇒ 10M P .θ = PU × 3θ ×
4 MP
40 M P M
⇒ PU = . =13.33 P
3 L L
So, C =13.33

28. A tapered circular rod of diameter varying from 20 mm to 10 mm is connected to another


uniform circular rod of diameter 10 mm as shown in the following figure. Both bars are made
of same material with the modulus of elasticity, E = 2 × 105 MPa. When subjected to a load
P = 30π kN, the deflection at point A is ____ mm.
d1 = 20 mm

2m

d 2 = 10 mm

1.5 m

P = 30 πkN

Answer: 15
Exp: 30π
4P.L
∆1 = 20mm
πd1d 2 × E
4 × 30π × 2 × 106
=
π × 20 × 10 × 2 × 105
= 6 mm
10mm

30π
 India’s No.1 institute for GATE Training  1 Lakh+ Students trained till date  65+ Centers across India
14
CE-GATE-2015 PAPER| www.gateforum.com

P×L
∆2 = 30π
AE
4 × 30π × 1.5 × 106
=
π × 10 × 10 × 2 × 105
= 9 mm
∆ = ∆1 + ∆ 2 = 15mm
30π

29. A water tank is to be constructed on the soil deposit shown in the figure below. A circular
footing of diameter 3m and depth of embedment 1m has been designed to support the tank.
The total vertical load to be taken by the footing is 1500 kN. Assume the unit weight of water
as 10kN/ m 3 and the load dispersion pattern as 2V:1H. The expected settlement of the tank
due to primary consolidation of the clay layer is ____ mm.

2m Silty Sand Bulk unit weight = 15 kN / m3

6m Sand Saturated unit weight = 18kN / m3 .

Saturated unit weight = 18 kN / m3


Normally
Compression index = 0.3
consolidated
10m Initial void ratio = 0.7
clay
Coefficient of consolidation = 0.004cm 2 / s

Dense Sand

Answer: 53.236
Cc  σ + ∆σ 
Exp: Settlement = H 0 log  0 
1 + e0  σ0  1
σ0 = 15 × 2 + (18 − 10 ) × 6 + (18 − 10 ) *5
2 3

= 118kN m 2
6 12
1500
∆σ0 = = 8.488kN m 2
π
(3 + 6 + 6)
2
10
4
6 6
15

 India’s No.1 institute for GATE Training  1 Lakh+ Students trained till date  65+ Centers across India
15
CE-GATE-2015 PAPER| www.gateforum.com

0.3  118 + 8.488 


∆H = × 10log10  
1 + 0.7  118 
= 0.0532 m
∆H = 53.236 mm

30. Consider the following differential equation:


y y
x(ydx + xdy) cos = y ( xdy − ydx ) sin
x x
Which of the following is the solution of the above equation (c is an arbitrary constant)?
x y x y y y
(A) cos = c (B) sin = c (C) xy cos =c (D) xysin =c
y x y x x x
Answer: (C)
Exp: Given D.E
y y
x ( ydx + xdy ) cos = y ( xdy − ydx ) sin
x x
y  y
⇒ x ( ydx + xdy ) cos +  − sin  y ( xdy − ydx ) = 0
x  x
 y  y  y ( xdy − ydx )
⇒ ( ydx + xdy ) cos   +  − sin  =0
x  x x
 y  y  xdy − ydx 
⇒ ( ydx + xdy ) cos   + ( xy )  − sin  =0
x  x  x2 
 y
By observing, the above equation is d  (xy) cos  = 0
 x
y
Byintegrating, xy cos   = c
x

31. The composition of an air-entrained concrete is given below:


Water :184 kg / m3
Ordinary Portland Cement(OPC) : 368 kg / m3
Sand : 606kg / m3
Coarse aggregate :1155 kg / m 3
Assume the specific gravity of OPC, sand and coarse aggregate to be 3.14, 2.67 and 2.74,
respectively, The air content is_________ liters/ m 3 .
Answer: 51
MC MS Ma
Exp: + + + Vw + Va = 1
ρC ρS ρa

368 606 1155 184


⇒ + + + + VV = 1.0
3.14 × 1000 2.67 × 1000 2.74 × 1000 1000
 India’s No.1 institute for GATE Training  1 Lakh+ Students trained till date  65+ Centers across India
16
CE-GATE-2015 PAPER| www.gateforum.com

⇒ 0.117 + 0.227 + 0.421 + 0.184 + VV = 1.0

⇒ VV = 0.051

= 0.051 × 1000 = 51  50.32 l m 3

32. An earth embankment is to be constructed with compacted cohesionless soil. The volume of
the embankment is 5000 m 3 and the target dry unit weight is 16.2 kN / m3 . Three nearby
sites (see figure below) have been identified from where the required soil can be transported
to the construction site. The void ratios (e) of different sites are shown in the figure. Assume
the specific gravity of soil to be 2.7 for all three sites. If the cost of transportation per km is
twice the cost of excavation per m 3 of borrow pits, which site would you choose as the most
economic solution? (Use unit Weight of water = 10kN / m 3 ) .

Site X
e = 0.6

140 km

Construction
site

Site Z Site Y
e = 0.64 e = 0.7

(A) Site X (B) Site Y (C) Site Z (D) Any of the sites
Answer: (B)
V V Vy V
Exp: = x = = z
1 + e 1 + e1 1 + e2 1 + e3
G 2.67
γd = .γ w ⇒ 16 = × 10
1+ e 1+ e
⇒ e = 0.67
5000 Vx Vy V
= = = z
1.67 1.6 1.64 1.7
⇒ Vx = 4790.42 m 3
Vy = 4910.18m 3
Vz = 5089.82 m 3

 India’s No.1 institute for GATE Training  1 Lakh+ Students trained till date  65+ Centers across India
17
CE-GATE-2015 PAPER| www.gateforum.com

C x = C × 479042 + 2 × C × 140
= 5070.42 C
C y = C × 4910.18 + 2 × C × 80 = 5070.18C
C z = C × 5089.82 + 2 × C × 100 = 5289.82C

33. The concentration of Sulfur Dioxide (SO 2 ) is ambient atmosphere was measured as
30µg / m3 . Under the same conditions, the above SO 2 concentration expressed in ppm is
___.
Given : P / (RT) = 41.6 mol / m 3 ; where P=Pressure; T=Temperature ; R=universal gas
constant; Molecular weight of SO 2 = 64.
Answer: 0.0133
Exp: 1 m3 of air has 30 mg SO2
106 m 3 of air has 30g SO 2
30
= mol SO 2
64
nRT n 30 64 mol
V= = =
P P RT 41.6 mol m 3
30
= = 0.0113m3
64 × 41.6
Concentration of SO 2 in ppm = 0.0113ppm

34. The 4-hr unit hydrograph for a catchment is given in the table below. What would be the
maximum ordinate of the S-curve (in m 3 / s) derived from this hydrograph?
Time(hr) 0 2 4 6 8 10 12 14 16 18 20 22 24
Unit
hydrograph 0 0.6 3.1 10 13 9 5 2 0.7 0.3 0.2 0.1 0
ordinate(m3/s)
Answer: 22
Exp:
Time UHO S-curve Addition SA
0 0 0
2 0.6 0.6
4 3.1 0 3.1
6 10 0.6 10.6
8 13 3.1 16.1
10 9 10.6 19.6
12 5 16.1 21.1
14 2 19.6 21.6
16 0.7 21.1 21.8
18 0.3 21.6 21.9
 India’s No.1 institute for GATE Training  1 Lakh+ Students trained till date  65+ Centers across India
18
CE-GATE-2015 PAPER| www.gateforum.com

20 0.2 21.8 22
22 0.1 21.9 22
24 0 22 22
Maximum S-curve ordinate is 22.

35. The drag force, FD, on a sphere kept in a uniform flow field depends on the diameter of the
sphere, D; flow velocity, V; fluid density, ρ; and dynamic viscosity, µ . Which of the
following options represents the non-dimensional parameters which could be used to analyze
this problem?
FD µ FD ρVD
(A) and (B) and
VD ρVD ρVD 2
µ
FD ρVD FD µ
(C) and (D) and
ρV D
2 2
µ ρV D
3 3
ρVD
Answer: (C)
ρvD
Exp: = Re → dim ensionles parameter
µ

FD ( kg − m s 2 )
→ dim ensionless parameter
 kg   m 
ρ  3  V2  2  × D2 ( m2 )
2

m   s 

36. Consider the singly reinforced beam section given below (left figure). The stress block
parameters for the cross-section from IS:456-2000 are also given below (right figure). The
moment of resistance for the given section by the limit state method is _________ kN-m.

M25 0.42x u
xu
0.36 f ck x u

300 mm d

4-12 Φ
Fe415
x u,max = 0.4d
for Fe415

200 mm
Answer: 42.82
π
Exp: A st = 4 × × (12) 2 = 453 mm 2
4
0.36 fck.b.xu = 0.87 fy Ast
 India’s No.1 institute for GATE Training  1 Lakh+ Students trained till date  65+ Centers across India
19
CE-GATE-2015 PAPER| www.gateforum.com

0.87 f y A st 0.87 × 415 × 453


⇒ xν = =
0.36f ck .b 0.36 × 25 × 200
= 90.86 mm
x ν , max = 0.48d
= 0.4 × 300 = 120 mm
x ν < x ν , max so U.R.section
M ν = 0.87 × f y × A st × ( d − 0.42 x ν )
= 0.87 × 415 × 453 × ( 300 − 42 × 90.86 ) = 42.82 kNm

37. Two reservoirs are connected through a 930 m long, 0.3 m diameter pipe, which has a gate
valve. The pipe entrances is sharp (loss coefficient = 0.5) and the valve is half-open (loss
coefficient = 5.5). The head difference between the two reservoirs is 20 m. Assume the
friction factor for the pipe as 0.03 and g = 10 m/s2. The discharge in the pipe accounting for
all minor and major losses is _________ m3/s.
Answer: 0.1413
Exp: Total loss = 20 m

0.5 × v 2 f × L v 2 5.5v 2 v 2
⇒ 20 = + × + +
29 d 2g 2g 2g
0.03 × 930 × v 2
⇒ 20 × 2 × 10 = 0.5 v 2 + + 5.5v 2 + v 2
0.3
400
⇒ v2 = =4
100
⇒ v =2m s
π π
θ = × d 2 × v = × ( 0.3) × 2 = 0.1413 m 3 s
2

4 4

38. A sign is required to be put up asking drivers to slow down to 30 km/h before entering Zone
Y (see figure). On this road, vehicles require 174 m to slow down to 30 km/h (the distance of
174 m includes the distance travelled during the perception-reaction time of drivers). The sign
can be read by 6/6 vision drivers from a distance of 48 m. The sign is placed at distance of x
m from the start of Zone Y so that even a 6/9 vision driver can slow down to 30 km/h before
entering the zone. The minimum value of x is _________ m.

Direction of vehicle movement

Sign Start of Zone Y


Road

x Zone Y

 India’s No.1 institute for GATE Training  1 Lakh+ Students trained till date  65+ Centers across India
20
CE-GATE-2015 PAPER| www.gateforum.com

Answer: 142
Exp: For a 6/6 person, driver can see from a distance of 48 m.
6
For a 6/9 person, driver can see from distance = 48 ×
= 32m
9
The vehicle requires 174 m to slow down to 30 km/hr
So, minimum distance, X=174-32=142 m.

39. The quadric equation x 2 − 4x + 4 = 0 is to be solved numerically, starting with the initial
guess x 0 = 3. The Newton-Raphson method is applied once to get a new estimate and then
the Secant method is applied once using the initial guess and this new estimate. The estimated
value of the root after the application of the Secant method is ___________.
Answer: 2.333
Exp: f(x) = x2-4x+4
x0 = 3
f ' ( x ) = 2x − 4
f ( x0 )
By Newton Raphson method x1 = x 0 −
f '( x0 )
1
= 3−
= 2.5
2
For secant method let x0 = 2.5 and x1 = 3
x1 − x 0
By secant method x 2 = x1 − f ( x1 )
f ( x1 ) − f ( x 0 )

( 3 − 2.5) f 3
= 3− ( )
f ( 3) − f ( 2.5 )
0.5
= 3− ×1
1 − ( 0.25 )
0.5
= 3−
0.75
= 3 − 0.6667
= 2.333

40. Consider the following complex function:


9
f (z) =
( z − 1)( z + 2 )
2

Which of the following one of the residues of the above function?


(A) −1 (B) 9/16 (C) 2 (D) 9
Answer: (A)

 India’s No.1 institute for GATE Training  1 Lakh+ Students trained till date  65+ Centers across India
21
CE-GATE-2015 PAPER| www.gateforum.com

9
Exp: f ( 3) =
( z − 1)( z + 2 )
2

z = 1 is a simple pole
z = -2 is a pole of order 2
9
 Res f ( z )  z =1 = lim ( z − 1)
( z − 1)( z + 2 )
z →1 2

9
= =1
9

1 d  9 
 Resf ( z )  z =−2 = lim ( z + 2 ) .
2

( z − 1)( z + 2 ) 
2
1! z →−2 dz 
−9
= lim
( z − 1)
z →−2 2

−9
= = −1
9

41. A short reach of a 2 m wide rectangular open channel has its bed level rising in the direction
of flow at a slope of 1 m in 10000. It carries a discharge of 4 m3/s and its Manning’s
roughness coefficient is 0.01. The flow in this reach is gradually varying. At a certain section
in this reach, the depth of flow was measured as 0.5m. The rate of change of the water depth
with distance, dy/dx, at this section is __________ (use g = 10 m/s2).
Answer: 0.0032
1
Exp: Adverse slope = −
10000
θ = 4 m 3 / s, n = 0.01, y = 0.5 m
dy S0 − Sf
=
dx 1 − Fr 2
V Q 4
Fr = = = = 1.79
gy By gy 2 × 0.5 × 10 × 5
1
Q= AR 2/3Sf 1/2
n
Q×n 4 × 0.01
Sf 1/2 = =
A × R 2/3  2 × 0.5 
2/3

2 × 0.5 ×  
 2 +1 
Sf = 6.92 × 10−3
1
− 6.92 × 10−3
dy 10000
= = 3.2 × 10−3 = 0.0032
dx 1 − (1.79) 2

 India’s No.1 institute for GATE Training  1 Lakh+ Students trained till date  65+ Centers across India
22
CE-GATE-2015 PAPER| www.gateforum.com

42. In a survey work, three independent angles X, Y and Z were observed with weights WX, WY,
WZ, respectively. The weight of the sum of angles X, Y and Z is given by:
 1 1 1   1 1 1 
(A) 1  + +  (B)  + + 
 WX WY WZ   WX WY WZ 
(C) WX + WY + WZ (D) WX2 + WY2 + WZ2
Answer: (A)

43. A hydraulic jump is formed in a 2m wide rectangular channel which is horizontal and
frictionless. The post-jump depth and velocity are 0.8 m and 1 m/s, respectively. The pre-
jump velocity is ___________ m/s. (use = 10 m/s2).
Answer: 4.94
Exp: B=2m, y 2 = 0.8m, U 2 = 1m / s
U2 1
F2 = = = 0.35
g.y 2 10 × 0.8
y1 1 1
= − + . 1 + 8F22
y2 2 2
y1 −1 1
⇒ = + . 1 + 8 × (3.5) 2 = 0.203
0.8 2 2
⇒ y1 = 0.203 × 0.8 = 0.162 m
Q = B.y 2 .V2 = B.y1V1
⇒ 0.8 × 1 = 0.162 × V1
⇒ V1 = 4.94 m / s.

44. A 20 m thick clay layer is sandwiched between a silty sand layer and a gravelly sand layer.
The layer experiences 30 mm settlement in 2 years.
Given
 π U 
2

   for U ≤ 60%
TV =  4  100 
1.781 − 0.933 log (100 − U ) for U > 60%
 10

Where Tv is the time factor and U is the degree of consolidation in %.


If the coefficient of consolidation of the layer is 0.003 cm2/s, the deposit will experience a
total of 50 mm settlement in the next ________ years.
Answer: 4.43
Exp: silty sand

20m clay

g ravel

 India’s No.1 institute for GATE Training  1 Lakh+ Students trained till date  65+ Centers across India
23
CE-GATE-2015 PAPER| www.gateforum.com

003 × 2 × 86400 × 365


TU = 2
= 0.189
 20 
 × 100 
 2 
π 2
U = 0.189 ⇒ U = 0.49 ≤ 60%
4
30
Consolidation = = 61.2.2 mm
0.49
Degree of consolidation for 50mm settlement
50
U= = 0.817 = 81.7%
61.22
⇒ Tv = 1.784 − 0.933 log10 (100 − U )
CV × t
= 0.608 =
d2
0.608 × (10 )
2
0.608 × H 2
⇒t= −4
= s
0.003 × 10 0.003 × 10−4
= 202666667 s
= 6.43 yr
Additional number of years = 6.43 − 2 − 4.43 years

45. A bracket plate connected to a column flange transmits a load of 100 kN as shown in the
following figure. The maximum force for which the bolts should be designed is __________
kN.
100 kN

600
75 75

75 75 All dimensions are in mm

Answer: 156.20
P 100
Exp: FD = = = 20 kN
n 5

 India’s No.1 institute for GATE Training  1 Lakh+ Students trained till date  65+ Centers across India
24
CE-GATE-2015 PAPER| www.gateforum.com

75

75
θ

Ft θ F
b

(P.d)r 100 × 600 × 75 2


Ft = = = 141.42 kN
∑ r2 4 × (75 2) 2

FR = FD2 + Ft2 + 2 × FD × Ft cos θ


1
= (20) 2 + (141.42) 2 + 2 × 20 × 141.42 ×
2
= 156.20 kN
1
cos θ =
2
⇒ θ = 45 .
o

46. Consider a primary sedimentation tank (PST) in a water treatment plant with surface
Overflow Rate (SOR) of 40 m3/m2/d. The diameter of the spherical particle which will have
90 percent theoretical removal efficiency in this tank is ___________ µm. Assume that
settling velocity of the particles in water is described by Stokes’s Law.
Given Density of water = 1000 kg/m3; Density of particle = 2650 kg/m3; g = 9.81 m/s2;
Kinematic viscosity of water ( v ) = 1.10 × 10−6 m 2 s
Answer: 22.58
Vs '
Exp: % removal = × 100
Vs

Vs ' = 0.9 Vs

0.9 × 40
= ms
86400

 India’s No.1 institute for GATE Training  1 Lakh+ Students trained till date  65+ Centers across India
25
CE-GATE-2015 PAPER| www.gateforum.com

1 g 0.9 × 40
⇒ × d 2 × ( ρS − ρ w ) =
18 µ 86400

0.9 × 40 × 18 × V. ρ w
⇒d =
86480(G S − 1) × ρ w × g

⇒ d = 22.58 µm

47. A non-homogenous oil deposit consists of a silt layer sandwiched between a fine-sand layer at
top and a clay layer below. Permeability of the silt layer is 10 times the permeability of the
clay layer and one-tenth of the permeability of the sand layer. Thickness of the silt layer is 2
time the thickness of the sand layer and two-third of the thickness of the clay layer. The ratio
of equivalent horizontal and equivalent vertical permeability of the deposit is ___________.
Answer: 10.967
Exp:
H1 (1) K1 finesand
H2 ( 2) K2 silt
H3 ( 3) K 3 clay

1
k 2 = 10k 3 = k1
10
⇒ k1 = 10k 2
= 10 × 10k 3
k1 = 100k 3
k1 = 10k 2
H2 = 2H1
2 3 3
H 2 = H 3 ⇒ H 3 = H 2 = × 2H1 = 3H1
3 2 2
H 3 = 3H1
1 1
K1H1 + K1 × 2H1 + K1 × 3H1
K H + K 2 H 2 + K 3 H3 10 100
Kx = 1 1 =
H1 + H 2 + H 3 H1 + 2H1 + 3H1
 2 3 
1 + +  K1H1
Kx =  10 100 
=
123
K1
6H1 100 × 6
H1 + H 2 + H 3 6H1
Ky = =
H1 H 2 H 3 H1 2H1 × 10 3H1 × 100
+ + + +
K1 K 2 K 3 K1 K1 K1
Kx 123 321
= × = 10.967
K y 100 × 6 6

 India’s No.1 institute for GATE Training  1 Lakh+ Students trained till date  65+ Centers across India
26
CE-GATE-2015 PAPER| www.gateforum.com

48. In a region with magnetic declination of 2o E, the magnetic Fore bearing (FB) of a line AB
was measured as N79o 50′E. There was a local attraction at A. To determine the correct
magnetic bearing of the line, a point O was selected at which there was no local attraction.
The magnetic FB of line AO and OA were observed to be S52o 40′E and N50o 20′W,
respectively. What is the true FB of line AB?
(A) N81o 50′E (B) N82o10′E (C) N84o10′E (D) N77o 50′E
Answer: (C)
Exp: δ = 2°E
Magnetic F.B. of AB = N79O50’E = 79O50.

Correct FB of OA = N50O20’W = 309O40’

∴ Correct B.B of OA = 129O 40'

∵ observed F.B. of AO = observed BB of OA

= 552O 40'E =127O 20'

Error = M.V- T.V = -2O20’

Correction + 2O20’

T.B. of FB of AB = N 79°50'E + 2 + 2°20'

= N84°10'E

49. For the 2D truss with the applied loads shown below, the strain energy in the member XY is
___________ kN-m. For member XY, assume AE = 30 kN, where A is cross-section area and
E is the modulus of elasticity.
5kN
10 kN

3m

3m

X
Y

3m

3m

 India’s No.1 institute for GATE Training  1 Lakh+ Students trained till date  65+ Centers across India
27
CE-GATE-2015 PAPER| www.gateforum.com

Answer: 5
Exp:
5kN

10kN D

E 3m

F 3m

G 3m

A
10kN RG
RA 3m

RA×3 + 10×9 = 0
⇒ RA = -30 kN
RG = 35 kN
Taking joint A

30kN

10kN 10kN

30kN
Joint G

10 2

10kN 35kN

Joint B
Fx = 10 kN 10
F × L 10 × 10 × 3
2
U= = = 5kN − m 10 2
2A E 2 × 30 30

 India’s No.1 institute for GATE Training  1 Lakh+ Students trained till date  65+ Centers across India
28
CE-GATE-2015 PAPER| www.gateforum.com

50. Two beam are connected by linear spring as shown in the following figure. For a load P as
shown in the figure, the percentage of the applied load P carried by the spring is
___________.
P
L

EI

EI
( )
Ks = 3EI 2L3

Answer: 33.33
Exp:
P

R
R

R
R

R
∆=
K
R
= × 2L3
3EI
2R.L3
∆=
3EI
PL3 RL3 2RL3
− =
3EI 3EI 3EI
3 3
PL 3RL P
⇒ = ⇒ R = = 33.33%.
3EI 3EI 3

 India’s No.1 institute for GATE Training  1 Lakh+ Students trained till date  65+ Centers across India
29
CE-GATE-2015 PAPER| www.gateforum.com

51. The smallest and largest Eigen values of the following matrix are:
 3 −2 2 
 4 −4 6 
 
 2 −3 5 
(A) 1.5 and 2.5 (B) 0.5 and 2.5 (C) 1.0 and 3.0 (D) 1.0 and 2.0
Answer: (D)
 3 −2 2 
Exp: Let A =  4 −4 6 
 2 −3 5 
Characteristic equation is
|A-λI| = 0
3−λ −2 2
⇒ 4 −4 − λ 6 =0
2 −3 5−λ
⇒ λ 3 − 4λ 2 + 5λ − 2 = 0
⇒ ( λ − 1) ( λ 2 − 3λ + 2 ) = 0
( λ − 1)( λ − 1)( λ − 2 ) = 0
λ = 1, 2

52. A square footing (2mx 2m) is subjected to an inclined point load, P as shown in the figure
below. The water table is located well below the base of the footing. Considering one-way
eccentricity, the net safe load carrying capacity of the footing for a factor of safety of 3.0 is
__________ kN.
The following factors may be used.
Bearing capacity factors: Nq = 33.3, N γ = 37.16; Shape factors: Fqs = Fγs = 1.314; Depth
factors: Fqd = Fγd = 1.113; Inclination factors: Fqi = 0.444, Fγi = 0.02

P
GL

30o 1m

0.85m
Unit weight = 18kN m3
2m Cohesion = 0
Friction angle = 35o

 India’s No.1 institute for GATE Training  1 Lakh+ Students trained till date  65+ Centers across India
30
CE-GATE-2015 PAPER| www.gateforum.com

Answer: 450
q nu
Exp: q safc =
3
q nu = cN c + qN q + 0.5γBN γ − 8∆
C=0
q nu = q ( N q − 1) + 0.5γ BN γ

q ns =
1
3
( )
q ( N q − 1) Fqs × Fqd + Fqp + 0.5γBN γ × Fγs × Fγo × Fγp

 1 
1 18 × 1( 33.3 − 1) × 1.314 × 1.113 × 0.444 + × 2 × 18 × 
q ns = 2
3  
 37.16 × 1.314 × 1.113 × 0.02 
397.03
= = 132.364 kN m 2
3
For one way shear (eccentricity) area to be reduced

0.85 0.85
2

Reduced area of footing = 2×1.7 = 3.4m2


Load carrying capacity = 132.364×3.4 = 450.kN

53. In a catchment, there are four rain-gauge stations, P, Q, R, and S. Normal annual precipitation
values at these stations are 780 mm, 850 mm, 920 mm, and 980 mm, respectively. In the year
2013, stations Q, R, and S, were operative but P was not. Using the normal ratio method, the
precipitation at station P for the year 2013 has been estimated as 860 mm. If the observed
precipitation at stations Q and R for the year 2013 were 930 mm and 1010 mm, respectively;
what was the observed precipitation (in mm) at station S for that year?
Answer: 1076.2

Ps 1  Pp P P 
Exp: =  + Q + R 
N s 3  N p N Q N R 
P 1  860 930 1010 
⇒ s =  + +
980 3  780 850 920 
⇒ Ps = 1076.20 mm

 India’s No.1 institute for GATE Training  1 Lakh+ Students trained till date  65+ Centers across India
31
CE-GATE-2015 PAPER| www.gateforum.com

54. The acceleration-time relationship for a vehicle subjected to non-uniform acceleration is,
dv
= ( α − β v0 ) e −βt
dt
Where, v is the speed in m/s, t is the time in s, α and β are parameters, and v0 is the initial
speed in m/s. If the accelerating behavior of a vehicle, whose drive intends to overtake a slow
moving vehicle ahead, is described as,
dv
= ( α − βv )
dt
dv
Considering α = 2 m s 2 , β = 0.05s −1 and = 1.3 m s 2 at t = 3 s, the distance (in m)
dt
travelled by the vehicle in 35 s is ____________.
Answer: 900.83
dV
Exp: = ( α − β VO ) e −βT
dt
∫ dv = ∫ ( α − β VO ) × e .dt
−βt

=
( α − βVO ) e−βt
−β
t = 0, V = V0

⇒ V0 =
( α − βV0 ) + C
−β
α − βV0 α
C = V0 + ⇒C =
β β
α − ( αβ V0 ) × e−βt
⇒V=
β

αt 0 α − β V0 −βt 0
x=
β
+
β2
e −1 ( )
dv
= ( α − β V0 ) e−3β = 1.3
dt t =3
1.3
⇒ α − β V0 =
e −3β
αt 1.3
x = 0 + 2 −3β e −βt 0 − 1
β β (e )
( )
2 × 35 1.3 ( e − 1)
−35×0.05

= +
0.05 ( 0.05 ) ( e
2 −3x 0.05
)
=1400 − 499.17 = 900.83m

 India’s No.1 institute for GATE Training  1 Lakh+ Students trained till date  65+ Centers across India
32
CE-GATE-2015 PAPER| www.gateforum.com

55. On a circular curve, the rate of super elevation is e. While negotiating the curve a vehicle
comes to a stop. It is seen that the stopped vehicle does not slide inwards (in the radial
direction). The coefficient of side friction is f. Which of the following is true?
(A) e ≤ f (B) f < e < 2f
(C) e ≥ 2f (D) None of the above
Answer: (A)
Exp:
N
fN

mg sin θ
θ

mg
mg cos θ

f N ≥ mg sin θ
⇒ f ( mg cos θ ) = mg sin θ
⇒ f ≥ tan θ
⇒f ≥ e
⇒ e ≤f

 India’s No.1 institute for GATE Training  1 Lakh+ Students trained till date  65+ Centers across India
33
CE-GATE-2015 PAPER| www.gateforum.com

GATE 2015 –CE on 8th February, 2015 – (Afternoon Session)

General Aptitude Questions

Q.No-1-5 Carry One Mark Each

1. Choose the most appropriate word from the options given below to complete the following
sentence The official answered _________ that the complaints of the citizen would be looked
into.
(A) respectably (B) respectfully
(C) reputably (D) respectively
Answer: (B)

2. Choose the statement where underlined word is used correctly


(A) The minister insured the victims that everything would be all right.
(B) He ensured that the company will not have to bear any loss.
(C) The actor got himself ensured against any accident.
(D) The teacher insured students of good results
Answer: (B)
Exp: insured-the person, group, or organization whose life or property is covered by
an insurance policy.
ensured- to secure or guarantee

3. Four cards are randomly selected from a pack of 52 cards. If the first two cards are kings, what is
the probability that the third card is a king?
(A) 4/52 (B) 2/50
(C) 1/52 × (1/52) (D) 1/52 × (1/52) × (1/50)
Answer: (B)
Exp: There are 4 kings in a pack of 52 cards.
If 2 cards are selected and both are kings, remaining cards will be 50 out of which 2 will be kings.

4. Which word is not a synonym for the word vernacular?


(A) regional (B) indigeneous
(C) indigent (D) colloquial
Answer: (C)
Exp: vernacular- expressed or written in the native language of a place
indigent -deficient in what is requisite

 India’s No.1 institute for GATE Training  1 Lakh+ Students trained till date  65+ Centers across India
1
CE-GATE-2015 PAPER| www.gateforum.com

5. Mr. Vivek walks 6 meters North-East, then turns and walks 6 meters South- East, both at 60
degrees to East. He further moves 2 meters South and 4 meters West. What is the straight
distance in meters between the point he started from and the point he finally reached?
(A) 2 2 (B) 2 (C) 2 (D) 1 / 2
Answer: (A)

Q. No. 6 - 10 Carry Two Marks Each

6. How many four digit numbers can be formed with the 10 digits 0, 1, 2, ..... 9 if no number can
start with 0 and if repetitions are not allowed?
Answer: 4536
Exp: In thousands place, 9 digits except 0 can be placed
In hundreds place, 9 digits can be placed (including 0, excluding the one used in thousands place)
In tens place, 8 digits can be placed (excluding the ones used in thousands and hundreds place)
In ones place, 7 digits can be placed (excluding the one used in thousands, hundreds and tens
place)
Total number of combinations = 9 × 9 × 8 × 7 = 4536

7. The word similar in meaning to ‘dreary’ is


(A) cheerful (B) dreamy
(C) hard (D) dismal
Answer: (D)
Exp: dreary- depressingly dull and bleak or repetitive.

8. There are 16 teachers who can teach Thermodynamics (TD), 11 who can teach Electrical
Sciences (ES), and 5 who can teach both TD and Engineering Mechanics (EM). There are a total
of 40 teachers, 6 cannot teach any of the three subjects, i.e. EM, ES or TD. 6 can teach only ES. 4
can teach all three subjects, i.e. EM, ES and TD. 4 can teach ES and TD. How many can teach
both ES and EM but not TD?
(A) 1 (B) 2 (C) 3 (D) 4
Answer: (A)
Exp:
TD ES

11 0 6
4
1 1
11

EM

 India’s No.1 institute for GATE Training  1 Lakh+ Students trained till date  65+ Centers across India
2
CE-GATE-2015 PAPER| www.gateforum.com

9. Read the following table giving sales data of five types of batteries for years 2006 to 2012

Year Type Type Type Type Type


I II III IV V
2006 75 144 114 102 108
2007 90 126 102 84 126
2008 96 114 75 105 135
2009 105 90 150 90 75
2010 90 75 135 75 90
2011 105 60 165 45 120
2012 115 85 160 100 145
Out of the following, which type of battery achieved highest growth between the years 2006 and
2012?
(A) Type V (B) Type III
(C) Type II (D) Type I
Answer: (D)
Exp: Type-I achieved a growth of 53% in the period which is higher than any other type of battery

10. The given question is followed by two statements: select the most appropriate option that solves
the question
Capacity of a solution tank A is 70% of the capacity of tank B. How many gallons of solution are
in tank A and tank B?
Statements:
I. Tank A is 80% full and tank B is 40% full
II. Tank A if full contains 14,000 gallons of solution
(A) Statement I alone is sufficient
(B) Statement II alone is sufficient
(C) Either statement I or II alone is sufficient
(D) Both the statements I and II together are sufficient
Answer: (D)
Exp: Statement I can be used to solve the question if capacity of both tanks is already known
Statement-II can be used if it is known what quantity of each tank is full/empty.
Therefore, by using both statements
Let capacity of tank B is x

 India’s No.1 institute for GATE Training  1 Lakh+ Students trained till date  65+ Centers across India
3
CE-GATE-2015 PAPER| www.gateforum.com

70
x = 14000
100
⇒ x = 20000 gallons
80
Solution in tank A = ×14000 = 11200 gallons
100
40
Solution in tank B = × 20000 = 8000 gallons
100
∴Total solution = 11200 + 8000 = 19200 gallons

Section Name: Civil Engineering

Q. No. 1-25 Carry One Mark Each

1. A column of size 450 mm × 600 mm has unsupported length of 3.0 m and is braced against side
sway in both directions. According to IS 456:2000, the minimum eccentricities (in mm) with
respect to major and minor principle axes are
(A) 20.0 and 20.0 (B) 26.0 and 21.0
(C) 26.0 and 20.0 (D) 21.0 and 15.0
Answer: (B)
L D
Exp: emin = + ( or ) 20mm which ever is minimum.
500 30
3000 600
e xx = + = 26mm
500 30
300 450
e yy = + = 21mm
500 30

2. The relationship between the length scale ratio (Lr) and the velocity scale ratio (Vr) in hydraulic
models, in which Froude dynamic similarity is maintained, is
(A) Vr = L r (B) Lr = Vr (C) Vr = L1.5
r (D) Vr = Lr
Answer: (D)
V
Exp: Fround number =
gy
Vα y
V.α L r ∵ y α Lr

π2
cos x + isin x
3. Given i = −1, the value of the definite int egral, I = ∫ cos x − isin x dx is :
0

(A) 1 (B) -1 (C) i (D) –i


 India’s No.1 institute for GATE Training  1 Lakh+ Students trained till date  65+ Centers across India
4
CE-GATE-2015 PAPER| www.gateforum.com

Answer: (B)
π2
cos x + i sin x
Exp: I= ∫ cos x − isin x dx
0
π2 π2
eix
= ∫ dx = ∫e
2ix
dx
0
e − ix 0

π2
 e2ix 
= 
 2 0
1  2i2π 0
= e − e 
2 
1 πi 0
=  e − e 
2
1
= [ cos π + isin π − 1]
2
1
= [ −1 + 0 − 1] = −1
2

4. SO2 and CO adversely affect


(A) oxygen carrying capacity of blood and functioning of lungs respectively
(B) functioning of the respiratory system and brain respectively
(C) functioning of the respiratory system and oxygen carrying capacity of blood respectively
(D) functioning of air passages and chest respectively.
Answer: (C)
Exp: Carbon monoxide effects the bloods carrying capacity

5. A guided support as shown in the figure below is represented by three springs (horizontal, vertical
and rotational) with stiffness kx, ky and kθ respectively. The limiting values of kx, ky and kθ are

(A) ∞, 0, ∞ (B) ∞, ∞, ∞
(C) 0, ∞, ∞ (D) ∞, ∞, 0
Answer: (A)
Exp: As rotation and horizontal deflection in zero as per given figure. Therefore its stiffness is
Force
' ∞ ' as deflection = 0. stiffness =
deflection
and stiffness is zero in y direction
 India’s No.1 institute for GATE Training  1 Lakh+ Students trained till date  65+ Centers across India
5
CE-GATE-2015 PAPER| www.gateforum.com

6. Let A = αij  ,1 ≤ i, j ≤ n with n ≥ 3 and aij = i.j . The rank of A is


(A) 0 (B) 1
(C) n – 1 (D) n
Answer: (B)
Exp: Given A = [aij] 1≤i, j≤n, n≥3
and aij = i.j
1 2 3 − − −
2 4 6 − − − 

⇒A= 
3 6 9 − − −
 − − − − − − 
If we apply R2-2R1, R3-3R1……..
Every row will be zero row, except first row in echelon form
∴ρ(A) = 1

7. A hydraulic jump takes place in a frictionless rectangular channel. The pre-jump depth is yp. The
alternate and sequent depths corresponding to yp are ya and ys respectively. The correct
relationship among yp, ya and ys is
(A) ya < ys < yp (B) yp < ys < ya
(C) yp < ys = ya (D) ya = ys = yp
Answer: (B)

8. A steel member ‘M’ has reversal of stress due to live loads, whereas another member ‘N’ has
reversal of stress due to wind load. As per IS 800:2007, the maximum slenderness ratio permitted
is
(A) less for member ‘M’ than that of member ‘N’
(B) more for member ‘M’ than for member ‘N’
(C) same for both the members
(D) not specified in the Code
Answer: (A)
Exp: M – due to live load
N – due to wind load
M − λ − 180
As per IS800. M<N
N − λ − 350

2x
 1
9. lim 1 +  is equal to
x →∞
 x
(A) e-2 (B) e (C) 1 (D) e2

 India’s No.1 institute for GATE Training  1 Lakh+ Students trained till date  65+ Centers across India
6
CE-GATE-2015 PAPER| www.gateforum.com

Answer: (D)
2x
 1
Exp: lim  1 + 
x →∞
 x
2
  1 
x

=  lim  x +  
 x →∞  x  

= e2

10. In a leveling work, sum of the Back Sight (B.S.) and Fore Sight (F.S.) have been found to be
3.085 m and 5.645 m respectively. If the Reduced Level (R.L.) of the starting station is 100.000
m, the R.L. (in m) of the last station is _______.
Answer: 97.440
Exp: ∑ BS = 3.085 ∑ F.s = 5.645m
Fall = ∑ Fs − ∑ BS = 5.645 − 3.085 = 2.560
R.L of last station = R.L first − fall
= −100 − 2.560 = 97.440m

11. In friction circle method of slope stability analysis, if r defines the radius of the slip circle, the
radius of friction circle is
(A) r sin φ (B) r (C) r cos φ (D) r tan φ
Answer: (A)

12. Net ultimate bearing capacity of a footing embedded in a clay stratum


(A) increases with depth of footing only
(B) increases with size of footing only
(C) increases with depth and size of footing
(D) is independent of depth and size of footing
Answer: (D)
Exp: Because q u = CN c + 8DN q + 0.5γBN r
It is clay ∴φ = 0 ⇒ N r = 0, N q = 1
q u = CN C + γD
q nu = CN C + γD − γD = CN C

13. A groundwater sample was found to contain 500 mg/L total dissolved solids (TDS). TDS (in %)
present in the sample is ________.
Answer: 0.05

 India’s No.1 institute for GATE Training  1 Lakh+ Students trained till date  65+ Centers across India
7
CE-GATE-2015 PAPER| www.gateforum.com

Exp: TDS = 500mg lit 1 lit = 500 mg = 500 × 10−3 gm


500 1 lit = 1000 gm
% TDS = × 10−3 × 100
1000

14. In Newton-Raphson iterative method, the initial guess value (xini) is considered as zero while
finding the roots of the equation: f(x) = –2 + 6x – 4x2 + 0.5x3. The correction, ∆x, to be added to
xini in the first iteration is ___________.
Answer: 0.3333
Exp: f(x) = -2+6x-4x2+(0.5)x3
x0 = 0
f ' ( x ) = 6 − 8x + 1.5x 2
f ( 0 ) = −2 f '(0) = 6
By Newton-Raphson method
f ( x0 ) ( −2 )
x1 = x 0 − =0−
f '( x 0 ) 6
2
=
6
= 0.3333
∆x = x1 − x 0 = 0.3333 − 0 = 0.3333

15. Prying forces are


(A) shearing forces on the bolts because of the joints
(B) tensile forces due to the flexibility of connected parts
(C) bending forces on the bolts because of the joints
(D) forces due the friction between connected parts
Answer: (B)

16. For the plane stress situation shown in the figure, the maximum shear stress and the plane on
which it acts are
σ y = 50MPa

σ x = 50 MPa σ x = 50 MPa

σ y = 50 MPa

 India’s No.1 institute for GATE Training  1 Lakh+ Students trained till date  65+ Centers across India
8
CE-GATE-2015 PAPER| www.gateforum.com

(A) –50 MPa, on a plane 45° clockwise w.r.t. x-axis


(B) –50 MPa, on a plane 45° anti-clockwise w.r.t. x-axis
(C) 50 MPa, at all orientations
(D) Zero, at all orientations
Answer: (D)
σx − σy
Exp: τθ = sin 2θ + τxy cos 2θ
z
τxy = 0 & σ x & σ y are equal
∴σ x − σ y = 0
∴τθ = 0 in any direction

17. A superspeedway in New Delhi has among the highest super-elevation rates of any track on the
Indian Grand Prix circuit. The track requires drivers to negotiate turns with a radius of 335 m and
33° banking. Given this information, the coefficient of side friction required in order to allow a
vehicle to travel at 320 km/h along the curve is
(A) 1.761 (B) 0.176
(C) 0.253 (D) 2.530
Answer: 0.685
Exp:
R = 335m

33°

V = 320 kmph
5
= 320 ×
18
800
= ms
9
V 2 tan θ + f
=
gR 1 − f tan θ
( 800 g )2 tan 33° + f
⇒ =
9.81 × 335 1 − f × tan 33°
0.64g + f
⇒ 2.40 =
1 − f × 0.649
⇒ f = 0.685

18. A horizontal beam ABC is loaded as shown in the figure below. The distance of the point of
contraflexure from end A (in m) is _________.

 India’s No.1 institute for GATE Training  1 Lakh+ Students trained till date  65+ Centers across India
9
CE-GATE-2015 PAPER| www.gateforum.com

10 kN
0.75 m

B C

1m

Answer: 0.25
Exp:
A 10kN
C
B DS = 1
0.75m 0.25m
Let us take RC as redundant
Deflection at B due to load at C
Deflection at C due to load at B (∆BC)
[By Marshall reciprocal theorem]
10 × (.75 ) 10 × (.75 )
3 2

so, ∆ BC = + × 0.25
3E I 2E I
2.11
= ↓
EI
Deflection at C due to Redundant R C

∆ cc = R C ×
( .75 )
3

=
0.141R C

3E I EI
∵ ∆C = 0
2.11 .141R C
⇒ − =0
EI EI
⇒ R C = 15kN
x
10 kN

15 kN

Mx = 10×x-15×(x-.25) = 0
⇒ 10x-15x-3.75 = 0
⇒ x = 0.75m
So, distance of point of contraflexure from end A
= 1-0.75 = 0.25m

 India’s No.1 institute for GATE Training  1 Lakh+ Students trained till date  65+ Centers across India
10
CE-GATE-2015 PAPER| www.gateforum.com

19. In the water content of a fully saturated soil mass is 100% the void ratio of the sample is
(A) Less than specific gravity of soil
(B) equal to specific gravity of soil
(C) greater than specific gravity of soil
(D) independent of specific gravity of soil
Answer: (B)
wn 100G
Exp: S = 100% e = =e= =G
Sr 100

20. The relationship between porosity (η), specific yield (Sy) and specific retention (Sr) of an
unconfined aquifer is
(A) Sy + Sr = η (B) Sy + η = Sr (C) Sr + η = Sy (D) Sy + Sr + η =1
Answer: (A)

21. While minimizing the function f(x), necessary and sufficient conditions for a point, x0 to be a
minima are:
(A) f’(x0) > 0 and f ”(x0) = 0 (B) f’(x0) < 0 and f ”(x0) = 0
(C) f’(x0) = 0 and f ”(x0) < 0 (D) f’(x0) = 0 and f ”(x0) > 0
Answer: (D)

22. The combined correction due to curvature and refraction (in m) for distance of 1 km on the
surface of Earth is
(A) 0.0673 (B) 0.673
(C) 7.63 (D) 0.763
Answer: (A)
Exp: C = 0.0673d 2 = 0.0673 × 1

23. Surcharge loading required to placed on the horizontal backfill of a smooth retaining vertical wall
so as to completely eliminate tensile crack is:
(A) 2 c (B) 2 cka (C) 2c k a (D) 2c / k a
Answer: (D)
2c
Exp: Surcharge load to be placed as =
ka

24. A nozzle is so shaped that the average flow velocity changes linearly from 1.5 m/s at the
beginning to 15 m/s at its end in a distance of 0.375 m. The magnitude of the convective
acceleration (in m/s2) at the end of the nozzle is _________.
Answer: 54

 India’s No.1 institute for GATE Training  1 Lakh+ Students trained till date  65+ Centers across India
11
CE-GATE-2015 PAPER| www.gateforum.com

du du du
Exp: Convective acceleration = u +v +w
dx dy dz
(15 − 1.5)
= 1.5 = 54 m s 2
0.375

25. The following statements are made related to the lengths of turning lanes at signalised
intersections
(i) 1.5 times the average number of vehicles (by vehicle type) that would store in turning lane per
cycle during the peak hour.
(ii) 2 times the average number of vehicles (by vehicle type) that would store in turning lane per
cycle during the peak hour.
(iii) Average number of vehicles (by vehicle type) that would store in the adjacent through lane
per cycle during the peak hour.
(iv) Average number of vehicles (by vehicle type) that would store in all lanes per cycle during
the peak hour.
As per the IRC recommendations, the correct choice for design length of storage lanes is
(A) Maximum of (ii and iii) (B) Maximum of (i and iii)
(C) Average of (i and iii) (D) Only (iv)
Answer: (A)

Q. No. 26-55 Carry Two Marks Each

26. Ultimate BOD of a river water sample is 20 mg/L. BOD rate constant (natural log) is 0.15 day–1.
The respective values of BOD (in %) exerted and remaining after 7 days are:
(A) 45 and 55 (B) 55 and 45
(C) 65 and 35 (D) 75 and 25
Answer: (C)
Exp: y u = 20mg L
After 7 days = y u e − kt = 20 × e 0.15×7 = 7
7
% is = = 35%
20
exerted = 100 − 35 = 65%

 India’s No.1 institute for GATE Training  1 Lakh+ Students trained till date  65+ Centers across India
12
CE-GATE-2015 PAPER| www.gateforum.com

27. A steel strip of length, L = 200 mm is fixed at end A and rests at B on a vertical spring of
stiffness, k = 2 N/mm. The steel strip is 5 mm wide and 10 mm thick. A vertical load, P = 50 N is
applied at B, as shown in the figure. Considering E = 200 GPa, the force (in N) developed in the
spring is _________.

A B
k

Answer: 3
Exp: P k=F δ for cantilever
δ
δ=
( P − R ) L3 = R
k=R
R δ 3EI K
R δ=R K ( 50 − R ) 200 3

=R
K 5 × 105 z
3 × 200 × 10 ×
5

12
R = 3N

28. Match the information related to test on aggregates given in Group-I with that in Group-II.
Group-I Group-II
P. Resistance to impact 1. Hardness
Q. Resistance to wear 2. Strength
R. Resistance to weathering action 3. Toughness
S. Resistance to crushing 4. Soundness
(A) P-1, Q-3, R-4, S-2 (B) P-3, Q-1, R-4, S-2
(C) P-4, Q-1, R-3, S-2 (D) P-3, Q-4, R-2, S-1
Answer: (B)
Exp: Resistance to impact →Toughness
Resistance to wear → Hardness
Resistance to weathering → Soundness
Resistance to crushing → Strength

29. A simply supported reinforced concrete beam of length 10 m sags while undergoing shrinkage.
Assuming a uniform curvature of 0.004 m–1 along the span, the maximum deflection (in m) of the
beam at mid-span is _______.
Answer: 0.0005

 India’s No.1 institute for GATE Training  1 Lakh+ Students trained till date  65+ Centers across India
13
CE-GATE-2015 PAPER| www.gateforum.com

0
Exp:
1 1
R= = = 250m
ψ 0.004
A

2
1
OA = ( 250 ) −   = 249.9995m
2

2
∆AA ' = 0.0005m

30. A 6 m high retaining wall having a smooth vertical back face retains a layered horizontal backfill.
Top 3 m thick layer of the backfill is sand having an angle of internal friction, φ = 30° while the
bottom layer is 3 m thick clay with cohesion, c = 20 kPa. Assume unit weight for both sand and
clay as 18 kN/m3. The totalactive earth pressure per unit length of the wall (in kN/m) is:
(A) 150 (B) 216 (C) 156 (D) 196
Answer: (A)
Exp:
A 1
φ = 30 ka1 = 1
3 above = B = ka1 8H = × 18 × 3 = 18kal
3 2
18 ka 2 = 1
6m B 18 below B = ka 2 8E − 2c ka 2
14
φ = 20 = 1 × 18 × 3 − 2 × 20 1 = 14 kN m 2
3
18 68 PC = Ka 2 γz − 2c ka 2 = 18 × 6 − 2 × 20
C

1 1
Pa = ×18 × 3 + (14 + 68) × 3 = 150 kN m 2
2 2

31. A simply supported beam AB of span, L = 24 m is subjected to two wheel loads acting at a
distance, d = 5 m apart as shown in the figure below. Each wheel transmits a load, P = 3 kN and
may occupy any position along the beam. If the beam is an I-section having section modulus, S =
16.2 cm3, the maximum bending stress (in GPa) due to the wheel loads is ___________
P P

A B
d
L

Answer: 1759.2
 India’s No.1 institute for GATE Training  1 Lakh+ Students trained till date  65+ Centers across India
14
CE-GATE-2015 PAPER| www.gateforum.com

Exp: Maximum bending stress occurs at the point of maximum bending moment. Maximum B.M. will
occur under one of the point load such that resultant of the load system and point load under
consideration is equidistant from the centre.
3kN 3kN

9.5m 5m 9.5m
3kN
3kN

M = 3 × 9.5 = 28.5kN

M M 28.5 × 106
σ= .y = = = 1759.2 GPa
I Z 16.2 × 103

32. For probability density function of a random variable, x is

f (x) =
x
4
( 4 − x 2 ) for 0 ≤ x ≤ 2
= 0 otherwise
The mean µ z of the random var ialbe is _____
Answer: 1.0667

Exp: f (x) =
x
4
(4 − x2 ) 0≤x≤2

mean = µ x = E ( x )
2
= ∫ xf ( x ) dx
0
2
x
= ∫ x   ( 4 − x 2 ) dx
0 
4
0
=
1
40∫ ( 4x 2 − x 4 ) dx

2
1  4x 3 x 5 
=  − 
4 3 5 0

 India’s No.1 institute for GATE Training  1 Lakh+ Students trained till date  65+ Centers across India
15
CE-GATE-2015 PAPER| www.gateforum.com

1  8 32 
= 4. −
4  3 5 
32  1 1 
= −
4  3 5 
 2  16
= 8   = = 1.0667
15  15

33. In a pre-stressed concrete beam section shown in the figure, the net loss is 10% and the final pre-
stressing force applied at X is 750 kN. The initial fiber stresses (in N/mm2) at the top and bottom
of the beam were:
b

b = 250 mm
d1 d1 = 200 mm
d 2 = 100 mm

X
d1
d2

(A) 4.166 and 20.833 (B) –4.166 and –20.833


(C) 4.166 and –20.833 (D) –4.166 and 20.833
Answer: (D)
Exp: Loss = 10%
Find force = 750 kN
750
Initial force = = 833.33 kN
0.9
P m
Top & Bottom stress = ±
A z
833.33 833.33 × 103 × 100 × 6
= × 103 ±
250 × 400 250 × 4002
= 8.33 ± 12.5
Top = -4.166 (T)
Bottom = 20.833 (C)

34. A 588 cm3 volume of moist sand weighs 1010 gm. Its dry weight is 918 gm and specific gravity
of solids, G is 2.67. Assuming density of water as 1 gm/cm3, the void ratio is __________.
Answer: 0.71

 India’s No.1 institute for GATE Training  1 Lakh+ Students trained till date  65+ Centers across India
16
CE-GATE-2015 PAPER| www.gateforum.com

Exp: e=? Gγ ω
γd =
γ moist sand =
1010
= 1.717 g cc 1+ e
588 2.67 × 1
1.561 =
γd =
918
= 1.561 g cc 1+ e
588 e = 0.71

35. A pipe of 0.7 m diameter has a length of 6 km and connects two reservoirs A and B. The water
level in reservoir A is at an elevation 30 m above the water level in reservoir B. Halfway along
the pipe line, there is a branch through which water can be supplied to a third reservoir C. The
friction factor of the pipe is 0.024. The quantity of water discharged into reservoir C is 0.15 m3/s.
Considering the acceleration due to gravity as 9.81 m/s2 and neglecting minor losses, the
discharge (in m3/s) into the reservoir B is __________.
Answer: 0.5716
Exp:
30m
Q
Q − 0.15

0.15
flQ 2
hf =
12.1d 5
h f1 + h f 2 = 30m → in parallel

0.024 × 3000 × Q 2 0.024 × 3000 ( Q − 0.15 )


2

+ = 30
12.1 × 0.7 5 12.1 × 0.75
Q = 0.7216 m3 s
Q B = Q − 0.15 = 0.5716 m3 s

36. A 4 m thick layer of normally consolidated clay has an average void ratio of 1.30. Its
compression index is 0.6 and coefficient of consolidation is 1 m2/yr. If the increase in vertical
pressure due to foundation load on the clay layer is equal to the existing effective overburden
pressure, the change in the thickness of the clay layer is ______ mm.
Answer: 314
CC  σ + ∆σ 
Exp: ∆H = H 0 log10  0 
1 + e0  σ0 
0.6  σ + σ0 
= × 4 log10  0 
1 + 1.3  σ0 
= 0.314 m
∆ H = 314mm

 India’s No.1 institute for GATE Training  1 Lakh+ Students trained till date  65+ Centers across India
17
CE-GATE-2015 PAPER| www.gateforum.com

37. According to the concept of Limit State Design as per IS456: 2000, the probability of failure of a
structure is __________.
Answer: 0.097

38. Two pegs A and B were fixed on opposite banks of a 50 m wide river. The level was set up at A
and the staff readings on Pegs A and B were observed as 1.350 m and 1.550 m, respectively.
Thereafter the instrument was shifted and set up at B. The staff readings on Pegs B and A were
observed as 0.750 m and 0.550 m, respectively. If the R.L. of Peg A is 100.200 m, the R.L. (in m)
of Peg B is _____
Answer: 100
Exp: Reciprocal leveling

∆h =
( b1 − a1 ) + ( b 2 − a 2 ) = (1.55 − 1.35) + (10.75 − 0.55)
2 2
∆h = 0.20
RL of B = RL of A − 0.20 = 100m
From the reading we can see A is at higher level than B.

39. Stress path equation for tri-axial test upon application of deviatoric stress is, q = 10 3 + 0.5p. .
The respective values of cohesion, c (in kPa) and angle of internal friction, φ are:
(A) 20 and 20° (B) 20 and 30°
(C) 30 and 30° (D) 30 and 20°
Answer: (B)
Exp: Stress path equations
q β
σ1 − σ3 σ + σ3
= Ccos φ + 1 sin φ
2 2
q = 10 3 + 0.5P
p
C cos φ = 10 3
Tanβ = sin φ = 0.5 φ = sin −1 cos 7 = 30o
C cos30 = 10 3 C = 20o

40. The velocity components of a two dimensional plane motion of a fluid are
y3 x3
u= + 2x − x 2 y and v = xy2 − 2y −
3 3
The correct statement is:
(A) Fluid is incompressible and flow is irrotational
(B) Fluid is incompressible and flow is rotational
(C) Fluid is compressible and flow is irrotational
(D) Fluid is compressible and flow is rotational
 India’s No.1 institute for GATE Training  1 Lakh+ Students trained till date  65+ Centers across India
18
CE-GATE-2015 PAPER| www.gateforum.com

Answer: (A)
∂u ∂v
Exp: For incompressible flow. + = 0 ....(i)
∂x ∂y
1  ∂v ∂u 
For irrotational flow.  − =0 ...(ii)
2  ∂x ∂y 
∂  y3 2  ∂  2 x3 
(1)  + 2x − x y  +  xy − 2y − =0
∂x  3  ∂y  3
2 − 2xy + 2xy − 2 = 0
∂  2 x 3  ∂  y3 2 
(2)  xy − 2y −  −  + 2x − x y 
∂x  3  ∂x  3 
y2 − x 2 − y2 + x 2 = 0

41. A triangular gate with a base width of 2 m and a height of 1.5 m lies in a vertical plane. The top
vertex of the gate is 1.5 m below the surface of a tank which contains oil of specific gravity 0.8.
Considering the density of water and acceleration due to gravity to be 1000 kg/m3 and 9.81 m/s2,
respectively, the hydrostatic force (in kN) exerted by the oil on the gate is __________.
Answer: 29.43
Exp:
1.5 m oil

1.5

1  2 
Force on gate = × 1.5 × 2 × Gγ ω 1.5 + × 1.5 
2  3 
= 0.8 × 9810 × 2.5 × 1.5
= 29.43 kN

42. The average surface area of a reservoir in the month of June is 20 km2. In the same month, the
average rate of inflow is 10 m3/s, outflow rate is 15 m3/s, monthly rainfall is 10 cm, monthly
seepage loss is 1.8 cm and the storage change is 16 million m3. The evaporation (in cm) in that
month is
(A) 46.8 (B) 136.0 (C) 13.6 (D) 23.4
Answer: (B)

 India’s No.1 institute for GATE Training  1 Lakh+ Students trained till date  65+ Centers across India
19
CE-GATE-2015 PAPER| www.gateforum.com

Exp:
10 m3 s 10cm
A = 20km2

Reservoir

15m3 s
Q E =? 1.8cm

∵ changein storage = inflow-outflow


∆S= Qi + Q R − Q0 − QS − Q E
(
⇒ 16 × 106 = (10 × 86400 × 30 ) + 0.1 × 20 × 106 )
(
− (15 × 86400 × 30 ) − 1.8 × 10−2 × 20 × 106 )
27320000
⇒ QE = = 1.366m  136.6cm
20 × 106

2 1 
43. The two Eigen values of the matrix   have a ratio of 3 : 1 for p = 2. What is another value
1 p 
of p for which the Eigen values have the same ratio of 3 : 1?
(A) –2 (B) 1 (C) 7/3 (D) 14/3
Answer: (D)
2 1 
Exp: Let A =  
1 p 
Given that two eigen values of A are in 3:1
Ratio for p = 2
⇒ Characteristic equation λ2-4λ+3 = 0 (by substituting p=2)
⇒ λ=1,3
2 1 
14
If we take p = then A =  14 
3 1 
 3
 14   28 
⇒ λ 2 −  2 +  λ +  − 1 = 0
 3   3 
20 25
⇒ λ2 − λ + =0
3 3
⇒ 3λ 2 − 20λ + 25 = 0
5
λ = 5,
3
 India’s No.1 institute for GATE Training  1 Lakh+ Students trained till date  65+ Centers across India
20
CE-GATE-2015 PAPER| www.gateforum.com

5
Eigen values 5, are in ratio 3:1
3
14
∴p =
3

44. Consider the following second order linear differential equation


d2 y
2
= −12x 2 + 24x − 20
dx
The boundary conditions are at x = 0, y = 5 and at x = 2, y = 21
The value of at x = 1 is __________.
Answer: -2
Exp: Given
d2 y
2
= −12x 2 + 24x − 20
dx
y ( 0 ) = 5 y ( 2 ) = 21
y (1) = ?

Auxillary equation m = 0
2

m = 0,0
y c = (c1 + c 2 x) e 0 x = c1 + c 2 x

yp =
1
D2
( −12x 2 + 24x − 20 )
x4 x3 x2
= −12 + 24. − 20.
12 6 2!
= − x + 4x − 10x
4 3 2

y = c1 + c 2 x + 10x 2 + 4x 3 − x 4
y ( 0) = 5 ⇒ c1 = 5
y ( 2 ) = 21 ⇒ 21 = 5 + 2c 2 + 40 + 32 − 16
21 = 2c2 + 61
c 2 = −20
y = 5 − 20x + 10x 2 + 4x 3 − x 4
y (1) = 5 − 20 + 10 + 4 − 1
= −2

45. For step-size ∆x = 0.4, the value of following int egral u sin g Simpson 's 1 3 rule is _________ .
0.8

∫ ( 0.2 + 25x − 200x + 675x 3 − 900x 4 + 400x 5 ) dx


2

 India’s No.1 institute for GATE Training  1 Lakh+ Students trained till date  65+ Centers across India
21
CE-GATE-2015 PAPER| www.gateforum.com

Answer: -3.8293
Exp: Given h = ∆x = 0.4
f ( x ) = 0.2 + 25x − 200x 2 + 675x 3
− 900x 4 + 400x 5
0.8 − 0
x 0 = 0 x n = 0.8 ⇒ n = =0
0.4
x 0 0.4 0.8
y = f ( x ) 0.2 24.456 −126.744
1
By Simpson’s Rule
3
0.8
0.4
∫ f ( x ) dx =
0
3 
( 0.2 − 126.744 ) + 4 ( 24.456 )  = −3.8293

46. A field channel has cultivable commanded area of 2000 hectares. The intensities of irrigation for
gram and wheat are 30% and 50% respectively. Gram has a kor period of 18 days, kor depth of
12 cm, while wheat has a kor period of 18 days and a kor depth of 15 cm. The discharge (in m3/s)
required in the field channel to supply water to the commanded area during the kor period is
_____.
Answer: 1.427
Exp: Rafi crops Gran and wheat
A1 200 × 0.3 B
Q1 = = = 0.463 m 3 / s ; Duty = 8.64
D1 8.64 × 18 ∆
0.12
A 2 2000 × 0.5
Q2 = = = 0.964 m 3 / s
D 2 8.64 × 18
0.15
∴ Q1 + Q 2 is required =0.964+0.463=1.427 m 3 / s

47. The relation between speed u (in km/h) and density k (number of vehicles/km) for a traffic stream
on a road is u = 70 – 0.7k. The capacity on this road is _________ vph (vehicles/hour).
Answer: 175
Exp: u = 70 − 7k
Capacity = u × k, q = uk
q = ( 70 − 0.7k ) k
dq
= 70 − 0.7 × 2k = 0 ⇒ k = 50 V / km.
dk
q = ( 70 − 0.7 × 50 ) × 50 = 175 V / hr.

 India’s No.1 institute for GATE Training  1 Lakh+ Students trained till date  65+ Centers across India
22
CE-GATE-2015 PAPER| www.gateforum.com

48. A water treatment plant of capacity, 1 m3/s has filter boxes of dimensions 6 m × 10 m. Loading
rate to the filters is 120 m3/day/m2. When two of the filters are out of service for back washing,
the loading rate (in m3/day/m2) is _______
Answer: 144
Exp: Total water filters = 24 × 3600 × 1 = 86400 m 3 / day.
86400
S.A = = 720 m2 .
120
Area of one filter = 6 × 10 = 60 m 2 .
720
Total no. o f filters = = 12 filters.
60
2 out of services, total filters =10.
S.A of filters = 60 × 10 = 600 m 2 .
86400
The loading rate = = 144 m3 / day/ m 2 .
600

49. A pile of diameter 0.4 m is fully embedded in a clay stratum having 5 layers, each 5 m thick as
shown in the figure below. Assume a constant unit weight of soil as 18 kN/m3 for all the layers.
Using λ-method (λ = 0.15 for 25 m embedment length) and neglecting the end bearing
component, the ultimate pile capacity (in kN) is ________.

G S
Pile Dia = 0.4m, L = 25 m

5m c = 40 kPa
5m c = 50 kPa
γ =18kN m 3
5m c = 60 kPa
for all layers
5m c = 70 kPa
5m c = 80 kPa

Answer: 1060.29
Exp: Ultimate Bearing capacity, Qu
= λ (σv.avg+2cu)As
= 0.15 [18×12.5+2Cu]×[π×0.4×25]
w=0
⇒ qu = 1060.29 kN

 India’s No.1 institute for GATE Training  1 Lakh+ Students trained till date  65+ Centers across India
23
CE-GATE-2015 PAPER| www.gateforum.com

50. In Marshall method of mix design, the coarse aggregate, fine aggregate, fines and bitumen having
respective values of specific gravity 2.60, 2.70, 2.65 and 1.01, are mixed in the relative
proportions (% by weight) of 55.0, 35.8, 3.7 and 5.5 respectively. The theoretical specific gravity
of the mix and the effective specific gravity of the aggregates in the mix respectively are
(A) 2.42 and 2.63 (B) 2.42 and 2.78
(C) 2.42 and 2.93 (D) 2.64 and 2.78
Answer: (A)
w1 + w 2 + w 3 + w 4
Exp: Gt =
w1 w 2
+ + ....
G1 G 2
100
= = 2.424
55 35.8 3.7 5.5
+ + +
2.0 2.7 2.65 1.01
Eff ‘G’ of aggregates G (fine+coarse)
(55 × 2.6) + (35.8 × 2.7)
G= = 2.64
55 + 35.8

51. In a system two connected rigid bars AC and BC are of identical length, L with pin supports at A
and B. The bars are interconnected at C by a frictionless hinge. The rotation of the hinge is
restrained by a rotational spring of stiffness, k. The system initially assumes a straight line
configuration, ACB. Assuming both the bars as weightless, the rotation at supports, A and B, due
to a transverse load, P applied at C is
PL PL P Pk
(A) (B) (C) (D)
4k 2k 4k 4L
Answer: (A)
Exp:
A L C L B
θ θ

External work done


1
= × ρ× L.θ ____ ( i )
2
Strain energy stored in spring

 India’s No.1 institute for GATE Training  1 Lakh+ Students trained till date  65+ Centers across India
24
CE-GATE-2015 PAPER| www.gateforum.com

1
= × k × ( 2θ ) × ( 2θ )
2
= 2k. θ2 ___ ( ii )
( i ) = ( ii )
1
⇒ PLθ = 2k.θ2
2
PL
⇒θ=
4k

52. A fixed end beam is subjected to a load, W at 1/3rd span from the left support as shown in the
figure. The collapse load of the beam is

X L3 Z
Y
2M P MP

L2 L2
(A) 16.5 MP/L (B) 15.5 MP/L (C) 15.0 MP/L (D) 16.0 MP/L
Answer: (C)
Exp: Plastic hinges formed =3

(1)

L/3 2L / 3
2M p θ α Mp
c

θ α

2M p 2M p
θ
α=
2
θ = 2α

θ=
L/3
L
∆= θ
3
α 2L
∆= .
3
θ = 2α.
 India’s No.1 institute for GATE Training  1 Lakh+ Students trained till date  65+ Centers across India
25
CE-GATE-2015 PAPER| www.gateforum.com

2M p θ + 2M p θ + 2M p α + M p α = W∆
θ L
2M p θ + 2M p θ + M p θ + M p = W× θ
2 3
WL 16.5
5.5M p = ⇒W= Mp
3 L
(2)
W

2M p θ θ Mp

θ
θ

Mp Mp
2M p θ + M p θ + M p ∆ + M p θ = W × ∆
L
5M p θ = W θ
3
Mp
15 =W
L
15M p
Lowest is collapse load
L

53. In a wastewater treatment plant, primary sedimentation tank (PST) designed at an overflow rate
of 32.5 m3/day/m2 is 32.5 m long, 80 m wide and liquid depth of 2.25 m. If the length of the weir
is 75 m, the weir loading rate (in m3/day/m) is ___________.
Answer: 112.67
Exp: Q = 32.5 m 3 / d / m 2
L = 32.5m
B = 8m
D = 2.25m
Q
V0 =
BL
Q = V0 BL
= 32.5 × 32.5 × 8
= 8450 m 3 / d
Weir length =75m.
8450
q= = 112.67 m3 / d / m
75

 India’s No.1 institute for GATE Training  1 Lakh+ Students trained till date  65+ Centers across India
26
CE-GATE-2015 PAPER| www.gateforum.com

54. A landfill is to be designed to serve a population of 200000 for a period of 25 years. The solid
waste (SW) generation is 2 kg/person/day. The density of the un-compacted SW is 100 kg/m3 and
a compaction ratio of 4 is suggested. The ratio of compacted fill (i.e. SW + cover) to compacted
SW is 1.5. The landfill volume (in million m3) required is _______.
Answer: 21.9
Exp: Total solid waste generated = 2kg×2×105
= 400000 kg/day
For 25 years = 400000×365×25
= 3.65×109 kg
volume after compaction
compaction ratio = 0.4 =
volume before compaction
3.65 × 109
V= = 3.65 × 107 m3
100
V ' = 0.4 × 3.65 × 107 = 1.46 × 107 m 3
sw + cover sw cover
= + = 1.5
sw sw sw
cover
⇒ = 0.5
sw
⇒ cover = 0.5 × 1.46 × 107 = 0.73 × 107 m 3
( )
Total volume = 1.46 + 0.73 × 107 = 21.9 × 106 m 3
= 21.9 million m3

55. The bearings of two inaccessible stations, S1 (Easting 500 m, Northing 500 m) and S2 (Easting
600 m, Northing 450 m) from a station S3 were observed as 225° and 153° 26′ respectively. The
independent Easting (in m) of station S3 is:
(A) 450.000 (B) 570.710
(C) 550.000 (D) 650.000
Answer: (C)
Exp:
S3 225 o

153 °28 '

l1 l2
45° 26 °34 '
S1

S2
( 500, 500 ) ( 450, 600 )

 India’s No.1 institute for GATE Training  1 Lakh+ Students trained till date  65+ Centers across India
27
CE-GATE-2015 PAPER| www.gateforum.com

Let s1s3 = l1, s2s3 = l2


Northing of S3 = 500 + l1 cos 45°
= 450 + l2 cos 26°34'
⇒ l1 cos 45° − l2 cos 26°34' = −50
Easting of S3
500 + l1 sin 45° = 600 − l2 sin 26°34'
l1 sin 45° + l2 sin 26°34' = 100
⇒ l1 = 70.71,l2 = 111.80
Easting of S3 = 500 + 70.71 × sin 45°
= 549.99 m  550 m

 India’s No.1 institute for GATE Training  1 Lakh+ Students trained till date  65+ Centers across India
28
CE-GATE-2016 PAPER-01| www.gateforum.com

General Aptitude
Q. No. 1 – 5 Carry One Mark Each

1. Out of the following four sentences, select the most suitable sentence with respect to
grammar and usage.
(A) I will not leave the place until the minister does not meet me.
(B) I will not leave the place until the minister doesn‟t meet me.
(C) I will not leave the place until the minister meet me.
(D) I will not leave the place until the minister meets me.
Key: (D)

2. A rewording of something written or spoken is a


________.
(A) paraphrase (B) paradox (C) paradigm (D) paraffin
Key: (A)

3. Archimedes s said, “Give me a lever long enough and a fulcrum on which to place it, and
I will move the world”
The sentence above is an example of a ___________statement.
(A) figurative (B) collateral (C) literal (D) figurine
Key: (A)

4. If „relftaga‟ means carefree, „otaga‟ means careful and „fertaga‟ means careless, which
of the following could mean „aftercare‟?
(A) zentaga (B) tagafer (C) tagazen (D) relffer
Key: (C)

5. A cube is built using 64 cubic blocks of side one unit. After it is built, one cubic block
is removed from every corner of the cube. The resulting surface area of the body (in
square units) after the removal is .
(A) 56 (B) 64 (C) 72 (D) 96
Key: (D)
Exp: Four blocks are needed for each direction(totally 3 directions) to build a bigger cube
containing 64 blocks. So area of one side of the bigger cube= 4  4  16units
There are 6 faces so total area= 6  16  96units
When cubes at the corners are removed they introduce new surfaces equal to exposes
surfaces so the area of the bigger cube does not change from 96

Q. No. 6 – 10 Carry Two Marks Each

6. A shaving set company sells 4 different types of razors, Elegance, Smooth, Soft and
Executive. Elegance sells at Rs. 48, Smooth at Rs. 63, Soft at Rs. 78 and Executive at Rs.

 India‟s No.1 institute for GATE Training  1 Lakh+ Students trained till date  65+ Centers across India
1
CE-GATE-2016 PAPER-01| www.gateforum.com

173 per piece. The table below shows the numbers of each razor sold in each quarter of a
year.
Quarter \ Product Elegance Smooth Soft Executive

Q1 27300 20009 17602 9999

Q2 25222 19392 18445 8942

Q3 28976 22429 19544 10234

Q4 21012 18229 16595 10109

Which product contributes the greatest fraction to the revenue of the company
in that year? (A) Elegance (B) Executive (C) Smooth
(D) Soft
Key: (B)

7. Indian currency notes show the denomination indicated in at least seventeen languages.
If this is not an indication of the nation‟s diversity, nothing else is.
Which of the following can be logically inferred from the above sentences?
(A) India is a country of exactly seventeen languages.
(B) Linguistic pluralism is the only indicator of a nation‟s diversity.
(C) Indian currency notes have sufficient space for all the
Indian languages. (D) Linguistic pluralism is strong evidence
of India‟s diversity.
Key: (D)

8. Consider the following statements relating to the level of poker play of four players P, Q,
R and S.
I. P always beats Q
II. R always beats S
III. S loses to P only sometimes
IV. R always loses to Q
Which of the following can be logically inferred from the
above statements?
(i) P is likely to beat all the three other players
(ii S is the absolute worst player in the set
(A) (i) only (B) (ii) only (C) (i) and (ii) (D) neither (i) nor (ii)
Key: (A)

9. If f(x7) = 2x7+3x-5, which of the following is a factor of f(x)?


(A) (x3+8) (B) (x-1) (C) (2x-5) (D) (x+1)
Key: (B)
Exp: from the option (b0 substitute x=1 in

 India‟s No.1 institute for GATE Training  1 Lakh+ Students trained till date  65+ Centers across India
2
CE-GATE-2016 PAPER-01| www.gateforum.com

2x 7  3x  5  0
2(1)7  3(1)  5  0; 5  5  0
So (x  1) is a factor of f (x)

10. In a process, the number of cycles to failure decreases exponentially with an increase
in load. At a load of 80 units, it takes 100 cycles for failure. When the load is halved, it
takes 10000 cycles for failure. The load for which the failure will happen in 5000
cycles is .
(A) 40.00 (B) 46.02 (C) 60.01 (D) 92.02
Key: (B)
Exp: From the data given we assume
exp onent
load 
log(cycles)
x
80   x  160
log(10000)
x
40   x  160
log(10000)
160
load   43.25
log 5000
Civil Engineering
Q. No. 1 – 25 Carry One Mark Each

1 Newton-Raphson method is to be used to find root of equation 3x-ex+sinx=𝑥 0. If the


initial trial value for the root is taken as 0.333, the next approximation for the root
would be .
(note: answer up to three decimal)

Key: (0.36)
1
Exp: Let f(x)  3x  e x  sin x and x 0  0.333 
3
 f (x)  3  e x  cos x
f (x 0 )  0.069 and f (x 0 )  2.55
f (x 0 )
 x1  x 0  (Using Newton  Rapshon method)
f (x 0 )
0.069
 0.333   0.360is the required next approximation
2.55

2P 2P 2P P P


2. The type of partial differential equation   3 2   0 is
x 2
y 2
xy x y
(A) elliptic (B) parabolic (C) hyperbolic (D) none of these
Key: (C)
Exp: Comparing the given equation with the general form of second order partial differential
equation, we have A=1, B=3, C=1  B2  4AC  5  0

 India‟s No.1 institute for GATE Training  1 Lakh+ Students trained till date  65+ Centers across India
3
CE-GATE-2016 PAPER-01| www.gateforum.com

 P.D.E is Hyperbola.

3. If the entries in each column of a square matrix M add up to 1, then an eigen value of M
is
(A) 4 (B) 3 (C) 2 (D) 1
Key: (D)
a b 
Exp: Consider the „2x2‟ square matrix M    Characteristic equation of M is
c d 
 2  (a  d)  (ad  bc)  0 …(1)
Put   1, weget
1  (a  d)  ad  bc  0
1  a  d  ad  (1  d)(1  a)  0
1  a  d  ad  1  a  d  ad  0
0 0, which is true
  1 Satisfies the equation (1) but   2,3,4 does not satisfy the equation (1). For all
possible values of a,d
Alternate Method: If sum of the elements in each row/column of a square matrix is equal
to „S‟ then „S‟ is an eigen value of that matrix.

4. Type II error in hypothesis testing is


(A) acceptance of the null hypothesis when it is false and
should be rejected
(B) rejection of the null hypothesis when it is true and should
be accepted
(C) rejection of the null hypothesis when it is false and should
be rejected
(D) acceptance of the null hypothesis when it is true and
should be accepted
Key: (A)
Exp: Type II Errors means acceptance of the null hypothesis when it is false and should be
rejected.
u 2u
5. The solution of the partial differential equation   2 is of the form
t x
    
(A) Ccos(kt) C1e
k/ x  k/ x
 C2 e
 
    
(B) Cekt C1e
k / x  k / x
 C2 e
 
(C) Cekt C1 cos
  
k /  x  C2 sin   
k /  x

(D) Csin(kt) C1 cos
  
k /  x  C2 sin   
k /  x

Key: (B)

 India‟s No.1 institute for GATE Training  1 Lakh+ Students trained till date  65+ Centers across India
4
CE-GATE-2016 PAPER-01| www.gateforum.com

u 2u
Exp: The P.D.E  2 …. (1) is called 1-D heat equations.
t x
Then the solution of (1) is
u(x,t)   Acospx  Bsin px  C.e p .t
2

k k
Put p2   k  p   .i
 
(1) becomes


u(x, t)  A cosh k /  .x  Bsinh k /  .x .C.e kt 
  k
x 
k
x   k
.x 
k
x 
 e 
e 
 e 
e 

 C.e .  A. 
kt
  B.  
  2   2 
    

  A  B   k /  .x  A  B 
 C.ekt e( k /  ).x .  e . 
  2   2 
 C.ekt c1e( k /  ).x  c2 e ( k /  ).x 
 

6. Consider the plane truss with load P as shown in the figure. Let the horizontal and
vertical reactions at the joint B be HB and VB, respectively and VC be the vertical
reaction at the joint C.

Which one of the following sets


gives the correct values of VB, HB and VC?
(A) VB = 0; HB = 0; VC = P
(B) VB = P/2; HB = 0; VC = P/2
(C) VB = P/2; HB = P (sin 60°); VC = P/2
(D) VB = P; HB = P (cos 60°); VC = 0
Key: (A)
Exp:  FH  0  HB  0
 M c  0  VB  2L  0  VB  0
 V  0  Vc  P

7. In shear design of an RC beam, other than the allowable shear strength of concrete
 c  , there is also an additional check suggested in IS 456-2000 with respect to the
maximum permissible shear stress c max   . The check for c max max is required to
take care of
(A) additional shear resistance from reinforcing steel
(B) additional shear stress that comes from accidental loading
(C) possibility of failure of concrete by diagonal tension
(D) possibility of crushing of concrete by diagonal compression

 India‟s No.1 institute for GATE Training  1 Lakh+ Students trained till date  65+ Centers across India
5
CE-GATE-2016 PAPER-01| www.gateforum.com

Key: (D)

8. The semi-compact section of a laterally unsupported steel beam has an elastic section
modulus, plastic section modulus and design bending compressive stress of 500 cm3,
650cm3 and 200MPa, respectively. The design flexural capacity (expressed in kNm) of
the section is .
Key: (100)
Exp: As per IS 800, the design bending strength of laterally unsupported beam as governed by
lateral torsional buckling is:
M d  b .Zp .f bd

Ze
b  for semi compact section,
Zp
Ze
So, M d  .Zp .f bd  Ze .f bd  500  103  200  106  100kN  m
Zp

9. Bull's trench kiln is used in the manufacturing of


(A) Lime (B) cement (C) bricks (D) none of these
Key: (C)

10. The compound which is largely responsible for initial setting and early strength gain of
Ordinary Portland Cement is
(A) C3A (B) C3S (C) C2S (D) C4AF
Key: (B)

11. In the consolidated undrained triaxial test on a saturated soil sample, the pore water
pressure is zero
(A) during shearing stage only
(B) at the end of consolidation stage only
(C) both at the end of consolidation and during shearing stages
(D) under none of the above conditions
Key: (B)

12. A fine grained soil is found to be plastic in the water content range of 26-48%. As per
Indian Standard Classification System, the soil is classified as
(A) CL (B) CH (C) CL-ML (D) CI
Key: (D)
Exp: Soil is plastic in range of 26% to 48%. So, plastic limit =26%, liquid limit = 48%
Since 35% <LL<50% So, CI

13. A vertical cut is to be made in a soil mass having cohesion c, angle of internal friction
ϕ, and unit weight γ. Considering Ka and Kp as the coefficients of active and

 India‟s No.1 institute for GATE Training  1 Lakh+ Students trained till date  65+ Centers across India
6
CE-GATE-2016 PAPER-01| www.gateforum.com

passive earth pressures, respectively, the maximum depth of unsupported excavation


is
4c 2c K p 4c K a 4c
(A) (B) (C) (D)
 Kp    Ka

Key: (D)
Exp: Pa  k A  z  2C k A
at z  z 0 , Pa  0
K A  z 0   2C K A  0
K A  z 0   2C K A
2C K A 2C
z0  
.K A  KA
2  2C 4C
Zcritic  2z 0  
 kA  KA

14. The direct runoff hydrograph in response to 5 cm rainfall excess in a catchment is


shown in the figure. The area of the catchment (expressed in hectares) is .

Key: (21.6)
Exp: Area under hydrograph = direct runoff volume
1 1
 1 6  60  60  5  A
2 100
1 6  60  60  100 2160000
A    216000m 2  21.6  104 m 2
2 5 10
A  21.6hectares

 India‟s No.1 institute for GATE Training  1 Lakh+ Students trained till date  65+ Centers across India
7
CE-GATE-2016 PAPER-01| www.gateforum.com

15. The type of flood routing (Group I) and the equation(s) used for the purpose (Group
II) are given below.
Group I Group II
P. Hydrologic flood routing 1. Continuity equation
Q. Hydraulic flood routing 2. Momentum equation
3. Energy equation
The correct match is
(A) P - 1; Q - 1, 2 & 3 (B) P - 1; Q - 1 & 2
(C) P - 1 & 2; Q - 1 (D) P - 1 & 2; Q - 1 & 2
Key: (B)

16. The pre-jump Froude Number for a particular flow in a horizontal rectangular channel
is 10. The ratio of sequent depths (i.e., post-jump depth to pre-jump depth) is ___

Key: (13.65)
Exp: Consider
Pre-jump depth  y1
Post-jump depth=y2
Fr(1)  10
We now that
Post jump depth y2 1 
  1  8F12  1
Pre jump depth y1 2  

1
 1  8  (10) 2  1
2 
1
  27.3  13.65
2

17. Pre-cursors to photochemical oxidants are


(A) NOX, VOCs and sunlight
(B) SO2, CO2 and sunlight
(C) H2S, CO and sunlight
(D) SO2, NH3 and sunlight
Key: (A)
18. Crown corrosion in a reinforced concrete sewer is caused by:
(A) H2S (B) CO2 (C) CH4 (D) NH3
Key: (A)

19. It was decided to construct a fabric filter, using bags of 0.45 m diameter and 7.5 m
long, for removing industrial stack gas containing particulates. The expected rate of
airflow into the filter is 10m3/s. If the filtering velocity is 2.0 m/min, the minimum
number of bags (rounded to nearest higher integer) required for continuous cleaning
operation is

 India‟s No.1 institute for GATE Training  1 Lakh+ Students trained till date  65+ Centers across India
8
CE-GATE-2016 PAPER-01| www.gateforum.com

(A) 27 (B) 29 (C) 31 (D) 32


Key: (B)
Exp: Given, D=0.45m, L=7.5m
At
No. of fabric filter bags, N 
Ab
10  60
Total area of filter (Af )   300m 2
2
Area of one bag (Ab)  dL   0.45  7.5  10.60m2
300
N  28.28  29
10.60

20. Match the items in Group – I with those in Group – II and choose the right combination.
Group - I Group - II

P. Activated sludge process 1. Nitrifiers and denitrifiers


Q. Rising of sludge 2. Autotrophic bacteria
R. Conventional nitrification 3. Heterotrophic bacteria
S. Biological nitrogen removal 4. Denitrifiers

(A) P-3, Q-4, R-2, S-1 (B) P-2, Q- R-4, S-1


(C) P-3, Q-2, R-4, S-1 (D) P-1, Q-4, R-2, S-3
Key: (A)
21. During a forensic investigation of pavement failure, an engineer reconstructed the
graphs P, Q, R and S, using partial and damaged old reports.

Theoretically plausible correct graphs according to the 'Marshall mixture design output'
are
(A) P, Q, R (B) P, Q, S (C) Q, R, S (D) R, S, P
Key: (B)

22. In a one-lane one-way homogeneous traffic stream, the observed average headway is
3.0s. The flow (expressed in vehicles/hr) in this traffic stream is .
Key: (1200)

 India‟s No.1 institute for GATE Training  1 Lakh+ Students trained till date  65+ Centers across India
9
CE-GATE-2016 PAPER-01| www.gateforum.com

3600
Exp: Maximum theoretical capacity, 
Ht
 3600
3
 1200 veh / hr.

23. The minimum number of satellites needed for a GPS to determine its position precisely is
(A) 2 (B) 3 (C) 4 (D) 24
Key: (C)

24. The system that uses the Sun as a source of electromagnetic energy and records the
naturally radiated and reflected energy from the object is called
(A) Geographical Information System
(B) Global Positioning System
(C) Passive Remote Sensing
(D) Active Remote Sensing
Key: (C)

25. The staff reading taken on a workshop floor using a level is 0.645 m. The inverted staff
reading taken to the bottom of a beam is 2.960 m. The reduced level of the floor is
40.500 m. The reduced level (expressed in m) of the bottom of the beam is
(A) 44.105 (B) 43.460 (C) 42.815 (D) 41.145
Key: (A)
Exp: RL of bottom of beam  40.5  2.96  0.645  44.105m

Q. No. 26 – 55 Carry Two Marks Each

26. Probability density function of a random variable X is given below

0.25 if 1  x  5
f (x)  
0 otherwise
P(X  4) is i
3 1 1 1
(A) (B) (C) (D)
4 2 4 8
Key: (A)

P  x  4    f (x)dx   (0)dx   (0.25) dx   (0)dx
4 1 4
Exp:
  1 4

1 1 3
 (x)14  (4  1) 
4 4 4

 India‟s No.1 institute for GATE Training  1 Lakh+ Students trained till date  65+ Centers across India
10
CE-GATE-2016 PAPER-01| www.gateforum.com

27. The value of

 1  sin x
 0 1 x 2
dx  
0 x
dx is

 3
(A) (B)  (C) (D) 1
2 2
Key: (B)
 1  
Exp: 0 1 x 2
dx   tan 1 x   tan 1   tan 1 0 
0 2
1  sin x   1
and L(sin x)   L   s 2 dx (Using “Division by x”)
s 1
2
 x  s 1


  tan 1 s   tan 1   tan 1 (s)  cot 1 (s)
s

 sin x
  e  sx . dx  cot 1 (s) (Using definition of Laplace transform)
0 x
Put s=0, we get

sin x 
0 x
dx  cot 1 (0) 
2
 1  sin x
 dx   dx  
0 1  x2 0 x

28. The area of the region bounded by the parabola y = x2 +1 and the straight line x+y = 3 is
59 9 10 7
(A) (B) (C) (D)
6 2 3 6
Key: (B)
Exp: At the point of intersection of the curves, y = x2+1 and x+y = 3 i.e., y=3-x, we have
x2 1  3  x  x2  x  2  0
 x  2,1 and 3  x  x 2  1

 Required area is  dydx


R


1
 3 x dy dx
x 2  y  x 2 1 

(3  x)  (x  1)dx
1
 2
2
1
 x3 x   9
   2x  
 3 2  2 2

 India‟s No.1 institute for GATE Training  1 Lakh+ Students trained till date  65+ Centers across India
11
CE-GATE-2016 PAPER-01| www.gateforum.com

29. The magnitudes of vectors P, Q and R are 100 kN, 250 kN and 150 kN, respectively as
shown in the figure.

The respective values of the magnitude (in kN) and the direction (with respect to the x-
axis) of the resultant vector are

(A) 290.9 and 96.0° (B) 368.1 and 94.7°


(C) 330.4 and 118.9° (D) 400.1 and 113.5°

Key: (C)
Exp: Resolving components w.r.t x-axis Q  250kN
 Fx  Pcos60  cos(60  45)  Rcos(90  45  60)
o

 Fx  100cos60o  250cos(95)  100cos(195)


 Fx  159.6kN P  100kN
 Fy  Psin 60  Qsin(60  45)  Rsin(90  45  60)
45o
 100sin 60  250sin(95)  100sin(195)
90o 60o
x
 Fy  289.3kN

F  Fx2  Fy2  (159.6) 2  (289.3) 2  330.4 kN R  150kN

Fy 289.3
tan       61.1o
Fx 159.6
. w.r.t x  axis  180  61.1  118.9 o

30. The respective expressions for complimentary function and particular integral part of
d4 y d2 y
the solution of the differential equation  3  108x 2 are
dx 4 dx 2
(A) c1  c2 x  c3 sin 3x  c4 cos 3x  and 3x 4  12x 2  c 

(B) c2 x  c3 sin 3x  c4 cos 3x  and 5x 4  12x 2  c 

(C) c1  c3 sin 3x  c4 cos 3x  and 3x 4  12x 2  c 

(D) c1  c2 x  c3 sin 3x  c4 cos 3x  and 5x 4  12x 2  c 

Key: (A)

 India‟s No.1 institute for GATE Training  1 Lakh+ Students trained till date  65+ Centers across India
12
CE-GATE-2016 PAPER-01| www.gateforum.com

d
Exp: D.E is (D 4  3D 2 ).y  108x 2 , D 
dx
A.E:- m 4  3m 2  0  m 2 (m 2  3)  0  m  0,0,  3i

 C.F   C1  C 2 x   C3 sin  
3x  C 4 cos  3x 
1
and P.I  (108x 2 )
D  3D 2
4

1
1 36  D2 
 (108x 2 )  1 
2
 (x )
 D2  D2  3 
3D 2 1 
 3 

36  D2  36  1 
 2 
1  ... (x 2 )  2  x 2  (2)  0 
D  3  D  3 
 2  x 4
2 x 2
    36x 2  dxdx  36     3x  12x
4 2

 3   (4)(3) 3 (2)(1) 

31. A 3 m long simply supported beam of uniform cross section is subjected to a uniformly
distributed load of w = 20 kN/m in the central 1 m as shown in the figure.

If the flexural rigidity (EI) of the beam is 30 x 106 N-m2, the maximum slope (expressed
in radians) of the deformed beam is
(A) 0.681 × 10-7 (B) 0.943 × 10-7 (C) 4.310 × 10-7 (D) 5.91× 10-7

Key: (*)
20kN / m
Exp:

P Q

1m 1m 1m

EI  30  106 N  m2
R p  R Q  10 kN

d2 y
M(x)  EI.  10x  0  x  1
dx 2
 10(1  y)  20y(0.5  y / 2)  0  y  0.5

 India‟s No.1 institute for GATE Training  1 Lakh+ Students trained till date  65+ Centers across India
13
CE-GATE-2016 PAPER-01| www.gateforum.com

0  x 1
dy 10x 2
 EI.   C1 0  y  0.5
dx 2
dy 10
 EI.  10y  y3  C1
dx 3
dy
at y  0.5; 0
dx
10
 0  10  0.5   (0.5)3  C1
3
 C1  4.583
dy dy

dx x 1 dx y 0

 5  C1  4.583
 C1  9.583
dy C 9.583
So,  1  3.19  107
dx max EI 30  10 6

32. Two beams PQ (fixed at P and with a roller support at Q, as shown in Figure I, which
allows vertical movement) and XZ (with a hinge at Y) are shown in the Figures I and II
respectively. The spans of PQ and XZ are L and 2L respectively. Both the beams are
under the action of uniformly distributed load (W) and have the same flexural stiffness,
EI (where, E and I respectively denote modulus of elasticity and moment of inertia
about axis of bending). Let the maximum deflection and maximum rotation be δmax1
and θmax1, respectively, in the case of beam PQ and the corresponding quantities
for the beam XZ be δmax2 and θmax2, respectively.

Which one of the following


relationships is true?
(A) δmax1 ≠ δmax2 and θmax1 ≠ θmax2 (B) δmax1 = δmax2 and θmax1 ≠ θmax2
(C) δmax1 ≠ δmax2 and θmax1 = θmax2 (D) δmax1 = δmax2 and θmax1 = θmax2

 India‟s No.1 institute for GATE Training  1 Lakh+ Students trained till date  65+ Centers across India
14
CE-GATE-2016 PAPER-01| www.gateforum.com

Key: (D)
Exp:
By principal of superposition,
g max1  g max 2 ; max 1  max 2

33. A plane truss with applied loads is shown


in the figure.

The members which do not carry any force are


(A) FT, TG, HU, MP, PL
(B) ET, GS, UR, VR, QL
(C) FT, GS, HU, MP, QL
(D) MP, PL, HU, FT, UR

Key: (A)
Exp: Conditions for zero force members are
(i) The member meets at a joint and they are non-collinear and no external force acts at
that joint. Both the members will be the zero force members.
(ii) When the members meet at joint and two are collinear and no external force acts at
the joint then third member will be zero force member.
According to the above statements
We can say that
FT, TG, HU, MP and PL members are zero force members.

34. A rigid member ACB is shown in the figure. The member is supported at A and B by
pinned and guided roller supports, respectively. A force P acts at C as shown. Let RAh
and RBh be the horizontal reactions at supports A and B, respectively, and RAv be the
vertical reaction at support A. Self- weight of the member may be ignored.

 India‟s No.1 institute for GATE Training  1 Lakh+ Students trained till date  65+ Centers across India
15
CE-GATE-2016 PAPER-01| www.gateforum.com

Which one of the following sets gives the correct magnitudes of RAv, RBh and RAh ?
1 2
(A) R Av  0; R Bh  P; and R Ah  P
3 3
2 1
(B) R Av  0; R Bh  P; and R Ah  P
3 3
3 1.5
(C) R Av  P; R Bh  P; and R Ah  P
8 8
1 2
(D) R Av  0; R Bh  P; and R Ah  P
3 3
Key: (D)
Exp: Taking moments about A = 0
 R Bh  8  P  1.5  0
1.5P
 R Bh 
8
 FH  0
1.5P
 R Ah  R Bh 
8
 F  0
 R Av  P

35. A reinforced concrete (RC) beam with width of 250 mm and effective depth of 400
mm is reinforced with Fe415 steel. As per the provisions of IS 456-2000, the minimum
and maximum amount of tensile reinforcement (expressed in mm2) for the section are,
respectively
(A) 250 and 3500
(B) 205 and 4000
(C) 270 and 2000
(D)300 and 2500
Key: (B)
Exp: Given:
Width of beam (b) = 250mm
Effective depth (d) =400 mm
As per IS-456:200
From clause 26.5.1.1 (a)
Minimum tension reinforcement
A s 0.85

bd fy

0.85bd 0.85  250  400


As    204.819  205mm 2
fy 4.15
From clause 26.5.1.2(b)

 India‟s No.1 institute for GATE Training  1 Lakh+ Students trained till date  65+ Centers across India
16
CE-GATE-2016 PAPER-01| www.gateforum.com

Maximum tension reinforcement  0.04bd  0.04  250  400  4000mm2

36. For M25 concrete with creep coefficient of 1.5, the long-term static modulus of
elasticity (expressed in MPa) as per the provisions of IS:456-2000 is .
Key: (10000)
Ec
Exp: Long term elasticity 
1 
E c  5000 f ck
 5000 25
 500  5
 25000
Creep coefficient     1.5
25000
long term elasticity   10,000
1  1.5

37. A propped cantilever of span L carries a vertical concentrated load at the mid-span. If
the plastic moment capacity of the section is MP, the magnitude of the collapse load is
8M p 6M p P
(A) (B)
L L
4M p 2M p
(C) (D)
L L l l
2 2

Key: (B)
Mp   Mp  0
L
Exp: M p   M p   M P   P    0
2
PL Mp  
3M p   Mp
2
6M p
P
L

38. Two plates are connected by fillet welds of size 10 mm and subjected to tension, as shown
in the figure. The thickness of each plate is 12 mm. The yield stress and the ultimate
tensile stress of steel are 250 MPa and 410 MPa, respectively. The welding is done in the
workshop (   mw  1.25  .

 India‟s No.1 institute for GATE Training  1 Lakh+ Students trained till date  65+ Centers across India
17
CE-GATE-2016 PAPER-01| www.gateforum.com

As per the Limit State Method of IS 800: 2007, the minimum length (rounded off to
the nearest higher multiple of 5 mm) of each weld to transmit a force P equal to 270
kN (factored) is
(A) 90 mm (B) 105 mm (C) 110 mm (D) 115 mm
Key: (B)
Exp: Maximum force carried by plates,
Ag f y 100  12  250
P   272.73kN
 m0 1.1

P
Load carried by each weld   136.36 kN
2
For minimum length of weld,

Strength of weld=Load carried by weld


fu
lw  136.36  103
3 m1
410
 l w  10  0.7    136.36  103
3  1.2
l w  102.9mm next multiple of 5 is
 105 mm

39. The Optimistic Time (O), most likely Time (M) and Pessimistic Time (P) (in days) of
the activities in the critical path are given below in the format O-M-P.

The expected completion time (in days) of the project is .

Key: (37.83)
8  4  10  14 6  8  4  11 5  7  4  10 7  4  12  18
Exp: te    
6 6 6 6
 10.333  8.1666  7.1666  12.166
 37.8328

40. The porosity (n) and the degree of saturation (S) of a soil sample are 0.7 and 40%,
respectively. In a 100m3 volume of the soil, the volume (expressed in m3) of air is
____ .
Key: (42)
Vv
Exp:   0.7 
V

 India‟s No.1 institute for GATE Training  1 Lakh+ Students trained till date  65+ Centers across India
18
CE-GATE-2016 PAPER-01| www.gateforum.com

Vw
S  40%  0.40 
Vv
Vv
V  Va  Vw  Vs  Vv  Vs V  ,
0.7
VV  0.7V

Vw
0.40 
Vv
Vw  0.4Vv
Vv  Va  0.4Vv
Vv  0.4Vv  Va
Va  0.6Vv
Va  0.6  0.7V  0.6  0.7  100
Va  42m 3

41. A homogeneous gravity retaining wall supporting a cohesionless backfill is shown in the
figure. The lateral active earth pressure at the bottom of the wall is 40 kPa.

The minimum weight of the wall (expressed in kN per m length) required to prevent it
from overturning about its toe (Point P) is
(A) 120 (B) 180 (C) 240 (D) 360
Key: (A)

Exp:

Pa

H
 2m
3
P
Ka  H  40  40
2m

 India‟s No.1 institute for GATE Training  1 Lakh+ Students trained till date  65+ Centers across India
19
CE-GATE-2016 PAPER-01| www.gateforum.com

40
 K a H  40  K a  
H
1
So, Pa  K a H 2  20H  120kN
2
Taking moment about P=0
 Pa  2  W  2
 Pa  W  120kN

42. An undisturbed soil sample was taken from the middle of a clay layer (i.e., 1.5 m
below GL), as shown in figure. The water table was at the top of clay layer. Laboratory
test results are as follows:

Natural water content of clay : 25%


Pre consolidation pressure of clay : 60 kPa
Compression index of clay : 0.50
Recompression index of clay : 0.05
Specific gravity of clay : 2.70
Bulk unit weight of sand : 17 kN/

A compacted fill of 2.5 m height with unit weight of 20kN/m3 is placed at the ground
level.

Assuming unit weight of water as10 kN/m3, the ultimate consolidation settlement
(expressed in mm) of the clay layer is ____ .
Key: (36.89)
Exp:
2.5m   20 kN / m3

1m
Sand   17 kN / m3

w  25%
0.5m
1m Clay 0  60 KPa
CC  0.50
CR  0.05
G S  2.7

 India‟s No.1 institute for GATE Training  1 Lakh+ Students trained till date  65+ Centers across India
20
CE-GATE-2016 PAPER-01| www.gateforum.com

For clay: w = 25% = 0.25


es  wG
wG 0.25  2.7
e   0.675
s 1
 sub   sat   w

Ge
  w  w
 1 e 
 G 1   2.7  1 
  w     10  10.15kN / m .
3

 1 e   1  0.675 

(Before Compaction)  17 1  0.5 10.15


 17  5.075

 22.075 kN / m 3  Pre consolidation pressure  60KPa 


Hence Over consolidation stage

 (After Compaction)  2.5  20  17  1  0.5  10.15


 50  17  5.075

 72.075kN / m3  60kPa
Hence Normal consolidation stage

CR H 0   C H     
Total settlement  log  c   c log  0 
1  e0  0  1  e0  c 
C R  0.05

e
CR   0.05
 
log  2 
 1 
 60 
e  0.05log  
 22.075 
e  0.0217
0.675  e0  0.0217

e0  0.653  For overconsolidation stage

 India‟s No.1 institute for GATE Training  1 Lakh+ Students trained till date  65+ Centers across India
21
CE-GATE-2016 PAPER-01| www.gateforum.com

0.05  1000  60  0.5  1000  72.075 


H  log   log  
1  0.653  22.075  1  0.675  60 
 60   72.075 
 30.25  log    298.5log  
 22.075   60 
 30.25  0.434  298.5  0.0796
 13.13  23.76
 36.89mm

43. A seepage flow condition is shown in the figure. The saturated unit weight of the soil
 sat  18 kN/m3. Using unit weight of water,  w γw = 9.81 kN/m3, the effective vertical
stress (expressed in kN/m2) on plane X-X is .

Key: (65.475)
Exp: Effective stress at x-x,   u
3
 5   sub   5 w
6
 5  18  9.81  2.5  9.81
 40.95  24.5  65.475kN / m 2

44. A drained triaxial compression test on a saturated clay yielded the effective
shear strength parameters as c' = 15 kPa and ϕ' = 22o. Consolidated Undrained triaxial
test on an identical sample of this clay at a cell pressure of 200 kPa developed a pore
water pressure of 150kPa at failure. The deviator stress (expressed in kPa) at failure is
_________ .
Key: (104.39)

Exp: Given effective shear strength parameters are effective are

 India‟s No.1 institute for GATE Training  1 Lakh+ Students trained till date  65+ Centers across India
22
CE-GATE-2016 PAPER-01| www.gateforum.com

C  15KPa, c  200KPa

  22o , u  150 KPa

'C  C  u  200  150  50KPa


We know that
1f'  'c  'd

3f'  c'  50KPa

1f'  3f' N   2C N 

    22 
N   tan 2  45    tan 2  45    tan 2 (45  11)  tan 2 (56)  2.198
 2  2 
1f'  50  2.198  2  15  2198
 109.9  30  (1.483)
 109.9  44.49  154.39

1f'  154.39
'c  'd  154.39
'd  154.39  'c
 154.39  50
Deviator stress('d )  104.39

45. A concrete gravity dam section is shown in the figure. Assuming unit weight of water
as 10kN/m3 and unit weight of concrete as 24kN/m3, the uplift force per unit length of
the dam (expressed in kN/m) at PQ is .

Key: (10500)

 India‟s No.1 institute for GATE Training  1 Lakh+ Students trained till date  65+ Centers across India
23
CE-GATE-2016 PAPER-01| www.gateforum.com

 w  H1
Exp:
 10  65  650kN / m2
1
 w H 2   w  H 2  H1 
3
1
 50   10  60  250kN / m 2
3
1 1
P    650  250   10   (250  50)  40
2 2
 4500  6000 10 m 40 m
 10500kN / m  w H2  10  5  50kN / m2

46. Seepage is occurring through a porous media shown in the figure. The hydraulic
conductivity values (k1, k2, k3) are in m/day.

The seepage discharge (m3/day per m) through the porous media at section PQ is
7 1 9 3
(A) (B) (C) (D)
12 2 16 4

Key: (B)
Exp: Flow is normal to bedding flame
 zi 20  30  10
K avg    2 m / day
z i 20 30 10
  
ki 3 3 1
Head difference 15  10 5 1
i   
Length 60 60 12
1
Seepage discharge q  K avg  i  A  2   3  1  0.5m3 / day / m
12

 India‟s No.1 institute for GATE Training  1 Lakh+ Students trained till date  65+ Centers across India
24
CE-GATE-2016 PAPER-01| www.gateforum.com

47. A 4 m wide rectangular channel, having bed slope of 0.001 carries a discharge of
16 m3/s. Considering Manning's roughness coefficient = 0.012 and g = 10 m/s2, the
category of the channel slope is
(A) Horizontal (B) mild (C) critical (D) steep
Key: (B)
Exp: Given discharge (Q) = 16 m3 / sec.
Bed slope (S) = 0.001
Manning‟s roughness coefficients (n) =0.012
g= 10m/s2
Width (B) =4m
Channel is wide rectangular

y B  y

Area (A) = B. y
Perimeter (P) =B
A By
Hydraulic Radius (R)   y
P B
1
Q  .A.R 2/3 .S1/ 2
n
1
16  (4  y).y 2/3 .s1/ 2
For meanings equation 0.02
16  0.012
 y5/3
4  0.001

y  2.95m
1/3
 q2 
yc   
 g  Q 16
q  4
1/3 B 4
 42 
 
 10 
yc  1.169m

y  yc  Channel is mild slope.

48. A sector gate is provided on a spillway as shown in the figure. Assuming g = 10 m/s2,
the resultant force per meter length (expressed in kN/m) on the gate will be _______ .

 India‟s No.1 institute for GATE Training  1 Lakh+ Students trained till date  65+ Centers across India
25
CE-GATE-2016 PAPER-01| www.gateforum.com

Key: (127)

Exp: A

5m

2.5 m

30o
B
30o

2.5 m

1 1000  10  (5) 2
FH   KN  125kN
2 1000
5
FH acts at a dis tan ce  1.67 m from the base.
3
Fv  Weight of water enclosed or supported (actual or imaginary) by the curved surface
 g  Vaccum of portion ABC
1 60 1 5 5 3
 1000  10    25     2   
2 180 2 2 2 
 25 
 1000  10      1.25  5 3 
6 
 1000  10  2.27  1N
 22.7kN

FR  FH2  FV2  (125)2  (22.7)2  127kN

 India‟s No.1 institute for GATE Training  1 Lakh+ Students trained till date  65+ Centers across India
26
CE-GATE-2016 PAPER-01| www.gateforum.com

49. A hydraulically efficient trapezoidal channel section has a uniform flow depth of 2m. The
bed width (expressed in m) of the channel is __ .

Key: (2.3)

Exp: 30o y  2m

For Hydraulically efficient channel,


2 2 4
B .y  2   2.31m
3 3 3

50. Effluent from an industry 'A' has a pH of 4.2. The effluent from another industry 'B'
has double the hydroxyl (OH-) ion concentration than the effluent from industry 'A'. pH
of effluent from the industry 'B' will be

Key: (4.5)

Exp: A
P H  4.2, POH  9.8,
 OH    109.8 mol / L

B
OH    2  109.8 mol / L
 P OH  9.8  log10 2  9.5
 P H  4.5

51. An electrostatic precipitator (ESP) with 5600 m2 of collector plate area is 96 percent
efficient in treating 185 m3/s of flue gas from a 200 MW thermal power plant. It was
found that in order to achieve 97 percent efficiency, the collector plate area should be
6100 m2. In order to increase the efficiency to 99 percent, the ESP collector plate area
(expressed in m2) would be ______
Key: (8011.8)

Q
Exp: A ln 1  0 
we
A1 A2
So, 
ln 1  1  ln 1  2 

 India‟s No.1 institute for GATE Training  1 Lakh+ Students trained till date  65+ Centers across India
27
CE-GATE-2016 PAPER-01| www.gateforum.com

5600 A
 
ln(1  9.6) ln(1  0.99)
 A  8011.8m2

52. The 2-day and 4-day BOD values of a sewage sample are 100 mg/L and 155 mg/L,
respectively. The value of BOD rate constant (expressed in per day) is

Key: (0.3)
Exp: BOD2  L0  1  e k2 

 100  L0  1  e2k  … (i)


Also, 155  L0  1  e4k  … (ii)

(i) / (ii)
100 1  e 2k
 
155 1  e 4k
 1  e 4k  1.55  1.55  e 2k
 e 4k  1.55  e 2k  0.55  0
Let e 2k  x
 x 2  1.55x  0.55  0
 x  0.55
e 2k  0.55  k  0.3day 1

53. A two lane, one-way road with radius of 50 m is predominantly carrying lorries with
wheelbase of 5 m. The speed of lorries is restricted to be between 60 kmph and 80
kmph. The mechanical widening and psychological widening required at 60 kmph are
designated as w me,80 and w ps,80, respectively. The mechanical widening and
psychological widening required at 80 kmph are designated as
and w me,80 and w ps,80 , respectively. The correct values of w m e ,, 6w 0 , p ws , 6 , 0 w m

respectively are
(A) 0.89 m, 0.50 m, 1.19 m, and 0.50 m (B) 0.50 m, 0.89 m, 0.50 m, and 1.19 m
(C) 0.50 m, 1.19 m, 0.50 m, and 0.89 m (D) 1.19 m, 0.50 m, 0.89 m, and 0.50 m

Key: (B)
n.l2 2  (5) 2
Exp: Wme, for 60kmph    0.5m
2R 2  50
V 60
Wps ,for 60kmph    0.89m
9.5 R 9.5  50
2  (5) 2
Wme ,for 80kmph   0.50m
2  50
80
Wps,80   1.19m
9.5  80

 India‟s No.1 institute for GATE Training  1 Lakh+ Students trained till date  65+ Centers across India
28
CE-GATE-2016 PAPER-01| www.gateforum.com

54. While traveling along and against the traffic stream, a moving observer measured the
relative flows as 50 vehicles/hr and 200 vehicles/hr, respectively. The average speeds
of the moving observer while traveling along and against the stream are 20 km/hr and
30 km/hr, respectively. The density of the traffic stream (expressed in vehicles/km) is
_______
Key: (3)

55. The vertical angles subtended by the top of a tower T at two instrument stations set up at
P and Q, are shown in the figure. The two stations are in line with the tower and spaced at
a distance of 60m. Readings taken from these two stations on a leveling staff placed at the
benchmark (BM = 450.000 m) are also shown in the figure. The reduced level of the top
of the tower T (expressed in m) is _____ .

Key: (476.9)

T
Exp: T
16.5o
2.555
R 10.5o S
555 T

10.5o

60 m x

TT
TST; tan16.5o  … (i)
x
TT TT  2
TRT '', tan(10.5o )   … (ii)
x  60 x  60
From (i) and (ii)
x  tan16.5o  (x  10)  tan(10.5o )  2
 x 0.296  (x  10)  0.185  2
 x  82.25m
So, TT  82.25  0.296  24.35m
So, RL of top of tower  450  2.555  24.35  476.905m

 India‟s No.1 institute for GATE Training  1 Lakh+ Students trained till date  65+ Centers across India
29
CE-GATE-2016 PAPER-02| www.gateforum.com

General Aptitude
Q. No. 1 – 5 Carry One Mark Each

1. If I were you, I ________ that laptop. It’s much too expensive.


(A) won’t buy (B) shan’t buy (C) wouldn’t buy (D) would buy
Key: (C)

2. He turned a deaf ear to my request.


What does the underlined phrasal verb mean?
(A) ignored (B) appreciated (C) twisted (D) returned
Key: (A)
3. Choose the most appropriate set of words from the options given below to complete the
following
sentence . _________ , ___________ is a will, _______ is a way.
(A) Wear, there, their (B) Were, their, there
(C) Where, there, there (D) Where, their, their
Key: (C)
4. (x % of y) + (y % of x) is equivalent to .
(A) 2 % of xy (B) 2 % of (xy/100) (C) xy % of 100 (D) 100 % of xy
Key: (A)
x xy
x% of y  y
100 100
y xy
Exp: y% of x  y
100 100
2
(x% of y)  (y% of x)  xy  2% of xy
100
5. The sum of the digits of a two digit number is 12. If the new number formed by reversing the
digits is greater than the original number by 54, find the original number.
(A) 39 (B) 57 (C) 66 (D) 93
Key: (A)
Exp: Let the original number be xy
y---unit digit of xy
x+y=12 -------(1)
10y+x=10x+y+54
9x-9y=-54-------(2)
Solving (1) & (2) we get, x=3 and y=9
So the number is 39.
Q. No. 6 – 10 Carry Two Marks Each

6. Two finance companies, P and Q, declared fixed annual rates of interest on the amounts
invested with them. The rates of interest offered by these companies may differ from year to
year. Year-wise annual rates of interest offered by these companies are shown by the line
graph provided below

 India’s No.1 institute for GATE Training  1 Lakh+ Students trained till date  65+ Centers across India
1
CE-GATE-2016 PAPER-02| www.gateforum.com

If the amounts invested in the companies, P and Q, in 2006 are in the ratio 8:9, then the
amounts received after one year as interests from companies P and Q would be in the ratio:
(A) 2:3 (B) 3:4 (C) 6:7 (D) 4:3

Key: (D)
Exp: let the deposited money in the company P is 8x
And the deposited money in the company Q is 9x

Interest after one year from the company P= 8x 


6 

 100 

Interest after one year from the company Q= 9x 


4 

 100 
8x  6
4
Ratio of Interest= 100 
9x  4 3
100
7. Today, we consider Ashoka as a great ruler because of the copious evidence he left behind in
the form of stone carved edicts. Historians tend to correlate greatness of a king at his time
with the availability of evidence today.
Which of the following can be logically inferred from the above sentences?
(A) Emperors who do not leave significant sculpted evidence are completely forgotten.
(B) Ashoka produced stone carved edicts to ensure that later historians will respect him.
(C) Statues of kings are a reminder of their greatness.
(D) A king’s greatness, as we know him today, is interpreted by historians
Key: (D)

8. Fact 1: Humans are mammals.


Fact 2: Some humans are engineers.
Fact 3: Engineers build houses.
If the above statements are facts, which of the following can be logically inferred?
I. All mammals build houses.
II. Engineers are mammals.
III. Some humans are not engineers.
(A) II only. (B) III only.
(C) I, II and III. (D) I only.

 India’s No.1 institute for GATE Training  1 Lakh+ Students trained till date  65+ Centers across India
2
CE-GATE-2016 PAPER-02| www.gateforum.com

Key: (B)
9. A square pyramid has a base perimeter x, and the slant height is half of the perimeter. What
is the lateral surface area of the pyramid?
(A) x2 (B) 0.75x2 (C) 0.50x2 (D) 0.25 x2
Key: (D)
Exp: Lateral surface area of the square pyramid
A  a a 2  4h 2 4a  perimeter
h  height
  slanting height
 a2 
2
a
2     h 2  h 2   2  
2  2
 a2 
A  a a 2  4   2    a2
 2

10. Ananth takes 6 hours and Bharath takes 4 hours to read a book. Both started reading copies
of the book at the same time. After how many hours is the number of pages to be read by
Ananth, twice that to be read by Bharath? Assume Ananth and Bharath read all the pages
with constant pace.
(A) 1 (B) 2 (C) 3 (D) 4
Key: (C)
Exp: Ananth covers 1/6 of the book in 1 hour.
Bharath covers 1/4 of the book in 1 hour
1
 x
6  2
1
 
4
x 4 1
  
6 2 2
6
 x   3hours
2

 India’s No.1 institute for GATE Training  1 Lakh+ Students trained till date  65+ Centers across India
3
CE-GATE-2016 PAPER-02| www.gateforum.com

Civil Engineering
Q. No. 1 – 25 Carry One Mark Each

1. The spot speeds (expressed in km/hr) observed at a road section are 66, 62, 45, 79, 32, 51,
56, 60, 53, and 49. The median speed (expressed in km/hr) is .
(Note: answer with one decimal accuracy)
Key: (54.5)
Exp. Median speed is the speed at the middle value in series of spot speeds that are arranged in
ascending order. 50% of speed values will be greater than the median 50% will be less than
the median.
Ascending order of spot speed studies are
32,39,45,51,53,56,60,62,66,79
53  56
Median speed   54.5 km / hr
2
2. The optimum value of the function f(x) = x2 – 4x +2 is
(A) 2 (maximum) (B) 2 (minimum) (C) −2 (maximum) (D) −2 (minimum)
Key: (D)
Exp: f (x)  0  2x  4  0
 x  2(statinary point)
f (x)  2  0  f (x) is minimum at x=2
And the minimum value is f(2)
i.e., (2)2  4(2)  2  2
 The optimum value of f(x) is -2 (minimum)

3. The Fourier series of the function,


f (x)     x  0
   x, 0  x  
in the interval  ,  is

 2  cos x cos3x   sin x sin 2x sin 3x 


f (x)    2   ....      ...
4  1 32
  1 2 3 
The convergence of the above Fourier series at x= 0 gives

1 2 
(1)n 1 2
(A) n
n 1
2

6
(B) 
n 1 n2

12

2 
(1)n 1 2
 2n  1 
1
(C)  (2n  1)
n 1
2

8
(D)
n 1 4
Key: (C)
Exp: The function is f(x) =0,   x  0
   x,0  x  
And Fourier series is

 India’s No.1 institute for GATE Training  1 Lakh+ Students trained till date  65+ Centers across India
4
CE-GATE-2016 PAPER-02| www.gateforum.com

 2  cos x cos3x cos5x   sin x sin 2x sin 3x 


f (x)    2    ...      ... …. (1)
4  1 32
52
  1 2 3 
1
At x=0, (a point of discontinuity), the fourier series converges to f (0 )  f (0 )  , where
2
f (0 )  lim(0)  0 and f (0 )  lim(  x)  
x 0 x 0

Hence, (1) becomes


  21 1 
    ...
2 4  12 32 
1 1 1 2
  2  2  ....
1 3 5 8
4. X and Y are two random independent events. It is known that P(X) = 0.40 and
P  X  YC   0.7. Which one of the following is the value of P(X ∪ Y)?

(A) 0.7 (B) 0.5 (C) 0.4 (D) 0.3

Key: (A)
Exp: P  X  Yc   0.7  P(x)  P(yc )  P(x).P(yc )  0.7

(Since, x,y are independent events)


 P(x)  1  P(y)  P(x) 1  P(y)  0.7
 P(y)  P(x  y)  0.3    (1)
P  x  y   P(x)  P(y)  P  x  y   0.4  0.3  0.7 3
Second Method:
We know that  P(x  y ' )  P(x)  P  x  y  
'
 
 0.7  0.4  1  P  x  y 
 P(x  y)  0.7

xy
5. What is the value of lim ?
x 0 x  y 2
2
y0

(A) 1 (B) −1 (C) 0 (D) Limit does not exist

Key: (D)
xy  0 
Exp: (i) lim  lim  2 2 
 0 (i .e., put x=0 and then y=0)
x  x y
2 2 y  0
0 y 
xy  0 
(ii) lim  lim  2   0 (i.e., put y=0 and then x=0)
x 0 x  y x 0 x  0
 
2 2
y 0

xy x(m.x)
(iii) lim  lim 2 (i.e., put y = mx)
x 0 x  y
2 2 x 0 x  m 2 x 2
y0

 India’s No.1 institute for GATE Training  1 Lakh+ Students trained till date  65+ Centers across India
5
CE-GATE-2016 PAPER-02| www.gateforum.com

 m  m
 lim    , which depends on ‘m’.
x  1  m
  1 m
2 2

Hence, the limit does not exists.

6. The kinematic indeterminacy of the plane truss shown in the figure is

(A) 11 (B) 8 (C) 3 (D) 0


Key: (A)
Exp: Number of joints (J) = 7
For rigid joint plan truss kinematic
Indeterminacy = 2J – R
 2  7  (2  1)  14  3  11

no. of reactors  (1)


no. of reactors  (2)

7. As per IS 456-2000 for the design of reinforced concrete beam, the maximum allowable
shear stress  c max  depends on the
(A) grade of concrete and grade of steel
(B) grade of concrete only
(C) grade of steel only
(D) grade of concrete and percentage of reinforcement
Key: (B)
Exp: By IS 456:2000
cmax  0.62 fck
a max depends on grade of concrete only.
8. An assembly made of a rigid arm A-B-C hinged at end A and supported by an elastic rope C-
D at end C is shown in the figure. The members may be assumed to be weightless and the
lengths of the respective members are as shown in the figure.

 India’s No.1 institute for GATE Training  1 Lakh+ Students trained till date  65+ Centers across India
6
CE-GATE-2016 PAPER-02| www.gateforum.com

Under the action of a concentrated load P at C as shown, the magnitude of tension developed
in the rope is
3P P 3P
(A) (B) (C) (D) 2P
2 2 8
Key: (B)

9. As per Indian standards for bricks, minimum acceptable compressive strength of any class
of burnt clay bricks in dry state is
(A) 10.0MPa (B) 7.5MPa (C) 5.0MPa (D) 3.5MPa

Key: (D)

10. A construction project consists of twelve activities. The estimated duration (in days) required
to complete each of the activities along with the corresponding network diagram is shown
below.
Activity Duration (days) Activity Duration (days)
A Inauguration 1 G Flooring 25
B Foundation work 7 H Electrification 7
C Structural construction-1 30 I Plumbing 7
D Structural construction-2 30 J Wood work 7
E Brick masonry work 25 K Coloring 3
F Plastering 7 L Handing over function 1

Total floats (in days) for the activities 5-7 and 11-12 for the project are, respectively,
(A) 25 and 1 (B) 1 and 1 (C) 0 and 0 (D) 81 and 0
 India’s No.1 institute for GATE Training  1 Lakh+ Students trained till date  65+ Centers across India
7
CE-GATE-2016 PAPER-02| www.gateforum.com

Key: (C)

Exp:

t e  38 t e  63 t  70
E  25 F7 e
4 6 8
c  30 t L  38 t L  63 J7
t L  70
te  0 te  1 t e  77
A 1 B  7 te  8 t e  80 t e  81
K 3 L 1
1 2 3 10 11 12
tL  0 tL  1 tL  8 t L  77 t L  80 t L  81
D  30 t e  38 t e  63 t e  70
I7
5 7 9
H  7t  70
t L  38 G  25t L  63 L

(5)  (7)  Total float  t e  t L  0


(11)  (12)  Total float  t e  t L  0

11. A strip footing is resting on the surface of a purely clayey soil deposit. If the width of the
footing is doubled, the ultimate bearing capacity of the soil
(A) becomes double (B) becomes half (C) becomes four-times (D) remains the same
Key: (D)

12. The relationship between the specific gravity of sand (G) and the hydraulic gradient (i) to
initiate quick condition in the sand layer having porosity of 30% is
(A) G = 0.7i + 1 (B) G = 1.43i − 1
(C) G = 1.43i + 1 (D) G = 0.7i − 1
Key: (C)
Exp: For quick sand conditions
G 1
i  G  i(1  e)  1
1 e
Given porosity   30%  0.3
n 0.3 0.3
e    0.43
1  n 1  0.3 0.7
G  i(1  0.43)  1
 i(1.43)  1
 1.43i  1

13. The results of a consolidation test on an undisturbed soil, sampled at a depth of 10 m below
the ground level are as follows:
Saturated unit weight : 16kN/m3
Pre-consolidation pressure : 90kPa
The water table was encountered at the ground level. Assuming the unit weight of water as
10kN/m3, the over-consolidation ratio of the soil is
(A) 0.67 (B) 1.50 (C) 1.77 (D) 2.00

 India’s No.1 institute for GATE Training  1 Lakh+ Students trained till date  65+ Centers across India
8
CE-GATE-2016 PAPER-02| www.gateforum.com

Key: (B)

Exp:

10 m  sat  16kN / m3
c  90kPa

c
Over consolidation ratio 

  10  (16  10)
 10  6
 60kN / m 3
90 9
O.C.R    1.5
60 6

14. Profile of a weir on permeable foundation is shown in figure I and an elementary profile of
'upstream pile only case' according to Khosla's theory is shown in figure II. The uplift
pressure heads at key points Q, R and S are 3.14 m, 2.75 m and 0 m, respectively (refer
figure II).

What is the uplift pressure head at point P downstream of the weir (junction of floor and pile
as shown in the figure I)?
(A) 2.75 m (B) 1.25 m (C) 0.8 m (D) Data not
sufficient
Key: (B)
2.75
R   100  68.75%
Exp: 4
p  100  R  31.25%
 India’s No.1 institute for GATE Training  1 Lakh+ Students trained till date  65+ Centers across India
9
CE-GATE-2016 PAPER-02| www.gateforum.com

Pr essure head at point


Now, p   100
Total head
h
 31.25   100  h  1.25m
4

15. Water table of an aquifer drops by 100 cm over an area of 1000 km2. The porosity and
specific retention of the aquifer material are 25% and 5%, respectively. The amount of water
(expressed in km3) drained out from the area is ______ .
Key: (0.2)
h  100cm, A  1000km 2
Exp:
n  0.25, r  0.05
 Porosity()  Sp. yield(y)  Sp Re tention (r)
 y  0.25  0.05  0.20
Amount of water drained out  y  A  h  0.2  1000 100 105  0.2km 2 .

16. Group I contains the types of fluids while Group II contains the shear stress - rate of shear
relationship of different types of fluids, as shown in the figure.
Group I Group II
P. Newtonian fluid 1. Curve 1
Q. Pseudo plastic fluid 2. Curve 2
R. Plastic fluid 3. Curve 3
S. Dilatant fluid 4. Curve 4
5. Curve 5

The correct match between Group I and Group II is


(A) P-2, Q-4, R-1, S-5 (B) P-2, Q-5, R-4, S-1
(C) P-2, Q-4, R-5, S-3 (D) P-2, Q-1, R-3, S-4
Key: (C)

17. The atmospheric layer closest to the earth surface is


(A) the mesosphere (B) the stratosphere
(C) the thermosphere (D) the troposphere
Key: (D)

18. A water supply board is responsible for treating 1500 m3/day of water. A settling column
analysis indicates that an overflow rate of 20 m/day will produce satisfactory removal for a
depth of 3.1 m. It is decided to have two circular settling tanks in parallel. The required
diameter (expressed in m) of the settling tanks is .
Key: (6.9)
Exp: Total area of settling tank required,

 India’s No.1 institute for GATE Training  1 Lakh+ Students trained till date  65+ Centers across India
10
CE-GATE-2016 PAPER-02| www.gateforum.com

Q 1500
A   75m 2
V 20
Since no. of tanks = 2
75
So, area of each tank   37.5m 2
2
d 2
 37.5  d  6.91m
4

19. The hardness of a ground water sample was found to be 420 mg/L as CaCO3. A softener
containing ion exchange resins was installed to reduce the total hardness to 75 mg/L as
CaCO3 before supplying to 4 households. Each household gets treated water at a rate of 540
L/day. If the efficiency of the softener is 100%, the bypass flow rate (expressed in L/day) is
.
Key: (385.7)
Exp: Since each household gets water = 540 L/day
So, total treated water = 540×4=2160 L/day
Let bypass flow rate is QL/day
So,
Q  42.0  (2160  Q)  0
75 
2160
 2160  75  Q  420
 Q  385.71 L / day

20. The sound pressure (expressed in µPa) of the faintest sound that a normal healthy individual
can hear is
(A) 0.2 (B) 2 (C) 20 (D) 55
Key: (C)
Exp: Faintest sound that a normal healthy individual can hear 20pa

21. In the context of the IRC 58-2011 guidelines for rigid pavement design, consider the
following pair of statements.
I: Radius of relative stiffness is directly related to modulus of elasticity of concrete and
inversely related to Poisson's ratio
II: Radius of relative stiffness is directly related to thickness of slab and modulus of
subgrade reaction.
Which one of the following combinations is correct?
(A) I: True; II: True (B) I: False; II: False
(C) I: True; II: False (D) I: False; II: True
Key: (C)

22. If the total number of commercial vehicles per day ranges from 3000 to 6000, the minimum
percentage of commercial traffic to be surveyed for axle load is
(A) 15 (B) 20 (C) 25 (D) 30
 India’s No.1 institute for GATE Training  1 Lakh+ Students trained till date  65+ Centers across India
11
CE-GATE-2016 PAPER-02| www.gateforum.com

Key: (A)
Exp: If Veh/day ranges from 3000 to 6000, min. 15% of traffic to be surveyed

23. Optimal flight planning for a photogrammetric survey should be carried out considering
(A) only side-lap
(B) only end-lap
(C) either side-lap or end-lap
(D) both side-lap as well as end-lap
Key: (D)
Exp: For optimal flight planning for a photogrammetric survey both side lap and end lap should be
considered.

24. The reduced bearing of a 10 m long line is N30°E. The departure of the line is
(A) 10.00 m (B) 8.66 m (C) 7.52 m (D) 5.00 m
N

Key: (D)
Exp:
l  10m
30O

w E

S
1
Departure = l. sin 30 = 10 × sin30  10   5m
2
25. A circular curve of radius R connects two straights with a deflection angle of 60°. The
tangent length is
(A) 0.577 R (B) 1.155 R (C) 1.732 R (D) 3.464 R

Key: (A)
R
Exp:

  60 
Tangent length  R.tan    R.tan    R tan 30o  0.557R
2  2 

 India’s No.1 institute for GATE Training  1 Lakh+ Students trained till date  65+ Centers across India
12
CE-GATE-2016 PAPER-02| www.gateforum.com

Q. No. 26 – 55 carry Two Marks Each

26. Consider the following linear system.


x  2y  3z  a
2x  3y  3z  b
5x  9y  6z  c
This system is consistent if a,b and c satisfy the equation
(A) 7a  b  c  0 (B) 3a  b  c  0
(C) 3a  b  c  0 (D) 7a  b  c  0

Key: (B)

27. If f(x) and g(x) are two probability density functions,


x
 a  1:  a  x  0

 x
f (x)    1: 0  x  a
 a
0 : otherwise


 x
 a : a  x  0

x
g(x)   : 0  x  a
a
0 : otherwise


Which one of the following statements is true?
(A) Mean of 𝑓(x) and 𝑔(x) are same; Variance of 𝑓 (𝑥) and 𝑔(𝑥) are same
(B) Mean of 𝑓(x) and 𝑔(x) are same; Variance of 𝑓 (𝑥) and 𝑔(𝑥) are different
(C) Mean of 𝑓(x) and 𝑔(x) are different; Variance of 𝑓 (𝑥) and 𝑔(𝑥) are same
(D) Mean of 𝑓(x) and 𝑔(x) are different; Variance of 𝑓 (𝑥) and 𝑔(𝑥) are different

Key: (B)
x   x 
Mean of f(x) is E(x)   x.  1dx   x.
0 a
Exp:  1dx
a
a  0
 a 
0 a
 x3 x 2   x 3 x 2 
      0
 3a 2 a  3a 2 0

 
Variance of f(x) is E x 2  E(x) where
2

 India’s No.1 institute for GATE Training  1 Lakh+ Students trained till date  65+ Centers across India
13
CE-GATE-2016 PAPER-02| www.gateforum.com

x   x 
E(x 2 )   x 2 .   1 dx   x 2 . 
0 a
 1 dx
a
a  0
 a 
0 a
 x 4 x3   x 4 x 3  a 3
      
 4a 3  a  4a 3 0 6
a3
 Variance is
6
 x  x
Next, mean of g(x) is E(x)   x.
0
dx   x. dx  0
a

a
 a  0
a
Variance of g(x) is E(x 2 )  E(x) , where
2

 x  2 x a3
E(x 2 )   x 2 .
0
0  a  2
a
dx  x . dx 
a
 a 
a
 Variance is
2
 Mean of f(x) and g(x) are same but variance of f(x) and g(x) are different.

28. The angle of intersection of the curves 𝑥2 = 4𝑦 and 𝑦2 = 4𝑥 at point (0, 0) is


(A) 0o (B) 30o (C) 45o (D) 90o
Key: (D)
Exp: Given curves x 2  4y … (1) and y 2  4x … (2)
Diff (1), (2) w.r.to ‘x’, we get
dy  dy 
2x  4    0  m1and (say)
dx  dx (0,0)
dy  dy 
2y  4       m 2 (say)
dx  dx 
1
Let m 2  , wherer m  0
m
m1  m 2 m m  1 0  1
tan    1  
1  m1m 2 m  m1 00

  90o ,
2

29. The area between the parabola 𝑥2 = 8𝑦 and the straight line y = 8 is ______ .
Key: (85.33)
x2
Exp: Parabola is x 2  8y  y  and straight line is y=8
8
At the point of intersection, we have
x2 x2
 8  x  8,8 and y  8  y 
8 8

 India’s No.1 institute for GATE Training  1 Lakh+ Students trained till date  65+ Centers across India
14
CE-GATE-2016 PAPER-02| www.gateforum.com

8  x2 
 Required area is x 8  8
8  dx

8 x2   x2 
 2   8  dx  8  is even function 
0
 8   8 
8
 x3  256
 2 8x     85.33 Sq.units
 24  0 3

The quadratic approximation of f (x)  x  3x  5 at the point 𝑥= 0 is


3 2
30.
(A) 3𝑥2 − 6𝑥 − 5 (B) −3𝑥2 − 5
(C) −3𝑥2 + 6𝑥− 5 (D) 3𝑥2 − 5
Key: (B)
Exp: The quadratic approximation of f(x) at the point x=0 is
x x
f (x)  f (0)  f (0)  f (0)
1! 2!
2
x
 (5)  x.{0}  {6}  3x 2  5
2

31. An elastic isotropic body is in a hydrostatic state of stress as shown in the figure. For no
change in the volume to occur, what should be its Poisson's ratio? y
(A) 0.00 (B) 0.25
(C) 0.50 (D) 1.00
Key: (C)
Exp: For no change in the volume
Volumetric strain ( v )  0 x

 x   y  z 
  (1  2)  0
 3 
z
1  2  0
1  2
1
Poissions ratio    0.5
2
32. For the stress state (in MPa) shown in the figure, the major principal stress is 10 MPa.

The shear stress τ is


(A) 10.0 MPa (B) 5.0 MPa (C) 2.5 MPa (D) 0.0 MPa

 India’s No.1 institute for GATE Training  1 Lakh+ Students trained till date  65+ Centers across India
15
CE-GATE-2016 PAPER-02| www.gateforum.com

Key: (B)
Exp: 5
 x  5MPa
 y  5MPa
xy  
We know that 5 5

 x  y  x  y 
2

 max/ min        xy
2

 2   2 

55 55
2

 max      
2
5
 2   2 
10  5  2  10  5  
  5MPa

33. The portal frame shown in the figure is subjected to a uniformly distributed vertical load w
(per unit length).
The bending moment in the beam at the joint ‘Q’ is

The bending moment in the beam at the joint ‘Q’ is


wL2
(A) zero (B) (hogging)
24
wL2 wL2
(C) (hogging) (D) (sagging)
12 8
Key: (A)
Exp: Since there is no external horizontal load.
So, Hp=0
 M  0
34. Consider the structural system shown in the figure under the action of weight W. All the
joints are hinged. The properties of the members in terms of length (L), area (A) and the
modulus of elasticity (E) are also given in the figure. Let L, A and E be 1 m, 0.05 m2 and 30
× 106 N/m2, respectively, and W be 100 kN.

 India’s No.1 institute for GATE Training  1 Lakh+ Students trained till date  65+ Centers across India
16
CE-GATE-2016 PAPER-02| www.gateforum.com

Which one of the following sets gives the correct values of the force, stress and change in
length of the horizontal member QR?
(A) Compressive force = 25 kN; Stress = 250 kN/m2; Shortening = 0.0118 m
(B) Compressive force = 14.14 kN; Stress = 141.4 kN/m2; Extension = 0.0118 m
(C) Compressive force = 100 kN; Stress = 1000 kN/m2; Shortening = 0.0417 m
(D) Compressive force = 100 kN; Stress = 1000 kN/m2; Extension = 0.0417 m
Key: (C)
Exp:
P
w
Q F1 F2 R T2

45o
Q R

S 50T2

W
100kN
FSQ  FSR FQP
w
 2FSQ cos 45o  w  FSQ 
2
w FQR
Similarly FPQ  FPR 
2
Now, Consider joint Q FQS

 India’s No.1 institute for GATE Training  1 Lakh+ Students trained till date  65+ Centers across India
17
CE-GATE-2016 PAPER-02| www.gateforum.com

 Fx  0
 FQP  cos 45o  FQS cos 45o  FQR  0
 FQR  w  100kN(Compressive)
FQR  L 100  2L
 QR    0.471 (Shortening)
2A E 4  0.05  0.3  106

35. A haunched (varying depth) reinforced concrete beam is simply supported at both ends, as
shown in the figure. The beam is subjected to a uniformly distributed factored load of
intensity 10 kN/m. The design shear force (expressed in kN) at the section X-X of the beam
is .

5m
Key: (65)
Exp:
M 400mm
Vx  .tan 
v  d
bd
600m
M
 Vd  v .bd  Vx  x tan 
dx
Vx  100  10  5  50kN;dx  500mm
55
M x  100  5  10   375kN  m
2
375
Vd  50   tan  100kN 100kN
0.5
600  400 200
tan   
10  1000 10,000
375 200
Vd  50    50  15  65kN
0.5 10,000

36. A 450 mm long plain concrete prism is subjected to the concentrated vertical loads as shown
in the figure. Cross section of the prism is given as 150 mm × 150 mm. Considering linear
stress distribution across the cross-section, the modulus of rupture (expressed in MPa) is
_____ .

 India’s No.1 institute for GATE Training  1 Lakh+ Students trained till date  65+ Centers across India
18
CE-GATE-2016 PAPER-02| www.gateforum.com

Key: (3) 11.25kN 11.25kN


150
Exp:

15m 150
15m 15m

11.25kN 11.25kN

Md

Md  11.25  0.15  1.6875kN  m


bd 2 0.15  (0.15) 2
Section Modulus, Z    0.000563m 3
6 6
M 1.6875
Modulus of rupture, f    103 MPa  3MPa
z 0.000563

37. Two bolted plates under tension with alternative arrangement of bolt holes are shown in
figures 1 and 2. The hole diameter, pitch, and gauge length are d, p and g, respectively.

Which one of the following conditions must be ensured to have higher net tensile capacity of
configuration shown in Figure 2 than that shown in Figure 1?
(A) p  2gd
2
(B) p 2  4gd

(C) p  4gd p  4gd


2
(D)

Key: (C)
Exp: p 2  4gd
This question can be solved by trick, Option (B) and (D) are not dimensionally correct.

38. A fixed-end beam is subjected to a concentrated load (P) as shown in the figure. The beam
has two different segments having different plastic moment capacities  Mp ,2Mp  as shown.

 India’s No.1 institute for GATE Training  1 Lakh+ Students trained till date  65+ Centers across India
19
CE-GATE-2016 PAPER-02| www.gateforum.com

The minimum value of load (P) at which the beam would collapse (ultimate load) is
(A) 7.5M p / L (B) 5.0 M p / L (C) 4.5 M p / L (D) 2.5M p / L
Key: (A)
P
Exp: 2L / 3

Mp
2M p

L L

Mechanism -I
P  2L 
2M p   M p   M p   M p   P  0
 3 
Mp 2PL
5M p   0
3
 
Mp 2PL
 
 5M p
2M p 3
Mp Mp 15M p
P  7.5Mp / L
2L
Mechanism -II

2l 4l
P  
2L 4L / 3 3 3
3   2
2M p Mp
 



2M p 2M
p

 India’s No.1 institute for GATE Training  1 Lakh+ Students trained till date  65+ Centers across India
20
CE-GATE-2016 PAPER-02| www.gateforum.com

 2L 
2M p   2M p   2M p   M p   P    0
 3 
2PL
4M p   3M p   0
3
   2PL
4M p   3M p    0
2 3
11 2PL
Mp  
2 3
33
P M p  8.25M p
4

So the minimum value of load =7.5 Mp/L

39. The activity-on-arrow network of activities for a construction project is shown in the figure.
The durations (expressed in days) of the activities are mentioned below the arrows.

The critical duration for this construction project is


(A) 13 days (B) 14 days (C) 15 days (D) 16 days

Key: (C)
Exp: te  5 t e  10
T w
Q
30 70
te  2 3 3
te  0
5 t e  13
P te  6 v x t e  15
R 50
10 20 40 80 90
4
2 te  6 2
S
60
3 te  5

40. The seepage occurring through an earthen dam is represented by a flow net comprising of 10
equi potential drops and 20 flow channels. The coefficient of permeability of the soil is 3
mm/min and the head loss is 5 m. The rate of seepage (expressed in cm3/s per m length of
the dam) through the earthen dam is ___.

Key: (500)
 India’s No.1 institute for GATE Training  1 Lakh+ Students trained till date  65+ Centers across India
21
CE-GATE-2016 PAPER-02| www.gateforum.com

Exp:
Given No. of flow channels (Nf) = 20
No. of equipotential drops (Nd)=10
Head loss (h) = 5m
3  10 3 m
Coefficient of permeable = 3mm/min   0.5  10 4 m / sec
60sec
Nf 20  m3 
Seepage q  kh  0.5  104  5   
Nd 10  sec 
 5  104 m3 / sec  5  104  106 cm3 / sec
q  500 cm3 / sec
41. The soil profile at a site consists of a 5 m thick sand layer underlain by a c-φ soil as shown in
figure. The water table is found 1 m below the ground level. The entire soil mass is retained
by a concrete retaining wall and is in the active state. The back of the wall is smooth and
vertical. The total active earth pressure (expressed in kN/m2) at point A as per Rankine's
theory is .

Key: (69.65)
Exp:

1m
 bulk  16.5kN / m3

Sand
4m  sat  19kN / m3 ,  w  9.81kN / w 3

3m
C   Soil
 sat  18.5kN / m3
 w  9.81kN / m3
C  25kN / m 2   24
In C   soil
1  sin  1  sin 24
kA    0.422
1  sin  1  sin24

 India’s No.1 institute for GATE Training  1 Lakh+ Students trained till date  65+ Centers across India
22
CE-GATE-2016 PAPER-02| www.gateforum.com

Active earth pressure in C   soil at A is

a  k A .A  2c k A
a  0.422 1  16.5  4  (19  9.81)  3  (18.5  9.81  7  9.8  2  25  0.422

  0.422  79.33  68.67  50  0.65


 33.48  68.67  32.5  69.65 kN / m2

42. OMC-SP and MDD-SP denote the optimum moisture content and maximum dry density
obtained from standard Proctor compaction test, respectively. OMC-MP and MDD-MP
denote the optimum moisture content and maximum dry density obtained from the modified
Proctor compaction test, respectively. Which one of the following is correct?
(A) OMC-SP < OMC-MP and MDD-SP < MDD-MP
(B) OMC-SP > OMC-MP and MDD-SP < MDD-MP
(C) OMC-SP < OMC-MP and MDD-SP > MDD-MP
(D) OMC-SP > OMC-MP and MDD-SP > MDD-MP

Key: (B)
MDD  MP
Exp:

MDD  SP

Dry
Density

OMC  MP OMC  SP
Water Content
So, OMC-SP>OMC-MP; MDD-SP<MDD-MP

43. Water flows from P to Q through two soil samples, Soil 1 and Soil 2, having cross sectional
area of 80cm2 as shown in the figure. Over a period of 15 minutes, 200 ml of water was
observed to pass through any cross section. The flow conditions can be assumed to be steady
state. If the coefficient of permeability of Soil 1 is 0.02 mm/s, the coefficient of
permeability of Soil 2 (expressed in mm/s) would be ____

Key: (0.045)
Exp: As per Darcy’s law,

 India’s No.1 institute for GATE Training  1 Lakh+ Students trained till date  65+ Centers across India
23
CE-GATE-2016 PAPER-02| www.gateforum.com

Q  K avg  i  A
 zi 150  150
K avg  
zi 150 150
 
k i 0.02 k
Q.L Q
K avg  
Aht Ait
150  150 200  103 1 1
   
150 150
 15  60 80  10 1
2

0.02 k
300 5
 
 1  180
150  50  
 k
 k  0.045 mm / sec

44. A 4 m wide strip footing is founded at a depth of 1.5 m below the ground surface in a c-φ
soil as shown in the figure. The water table is at a depth of 5.5 m below ground surface. The
soil properties are: c' = 35 kN/m2, φ' = 28.63°, γsat = 19 kN/m3, γbulk = 17 kN/m3 and γw =
9.81 kN/m3. The values of bearing capacity factors for different φ' are given below.

φ' Nc Nq Nγ
15° 12.9 4.4 2.5
20° 17.7 7.4 5.0
25° 25.1 12.7 9.7
30° 37.2 22.5 19.7

Using Terzaghi's bearing capacity equation and a factor of safety Fs = 2.5, the net safe
bearing capacity (expressed in kN/m2) for local shear failure of the soil is .

Key: (298.48)

Exp:

 India’s No.1 institute for GATE Training  1 Lakh+ Students trained till date  65+ Centers across India
24
CE-GATE-2016 PAPER-02| www.gateforum.com

1.5m

5.5m
C  35,   28.63
4m
 bulk   t  17
 sat  19
 w  9.81

As per Teraghis for local shear failure


2 2
Cm  C   35
3 3
2 
m  tan 1  tan  
3 
q nu  Cm N c'  q.  N q'  1  B..N 'r
1
2
 
q nu   c  N c'  q.  N q'  1  B..N 'r
2 1
3  2
2  2 
m  tan 1  tan    tan 1  tan 28.63   tan 1 (0.3639)
3  3 
m  19.998  20 o

From Table
for m  20  Nc  17.7, N q  7.4, N r  5

q nu  CN 'c    t Df   N 'q  1  (B t )N r


2 1
3 2
2 1
  35  17.7  17  1.5  (7.4  1)   4  17  5
3 2
 413  163.2  170  746.2
q nu 746.2
Net Safe bearing capacity    298.48kN / m 2
F.O.S 2.5

45. A square plate is suspended vertically from one of its edges using a hinge support as shown
in figure. A water jet of 20 mm diameter having a velocity of 10 m/s strikes the plate at its
mid-point, at an angle of 30° with the vertical. Consider g as 9.81 m/s2 and neglect the self-
weight of the plate. The force F (expressed in N) required to keep the plate in its vertical
position is

Key: (7.85)
Exp: For exerted by jet in X-direction

 India’s No.1 institute for GATE Training  1 Lakh+ Students trained till date  65+ Centers across India
25
CE-GATE-2016 PAPER-02| www.gateforum.com

Fx  a  V  v   sin 
2


 103   (0.02) 2  (10) 2  sin 30o
4
30o
 15.71N
Taking moment about hinge,
200 mm
Fx  0.1  F  0.2
Fx 15.71
F   7.85N
2 2
F

46. The ordinates of a one-hour unit hydrograph at sixty minute interval are 0, 3, 12, 8, 6, 3 and
0 m3/s. A two-hour storm of 4 cm excess rainfall occurred in the basin from 10 AM.
Considering constant base flow of 20m3/s, the flow of the river (expressed in m3/s) at 1 PM
is
Key: (60)
Exp:
Time Ordinate of 1 hr Lag Ordinate of 2h Ordinate of 2h
UH DRH UH
10:00 0 0 0
11:00 3 0 3 1.5
12:00 12 3 15 7.5
01:00 8 12 20 10
02:00 6 8 14 7
03:00 3 6 9 4.5
03:00 3 6 9 4.5
04:00 0 3 3 1.5
0 0 0
Flow of river = rainfall excess × ordinate of 2-h UH + Base flow
=4×10+20=60 m3/s

47. A 3 m wide rectangular channel carries a flow of 6m3/s. The depth of flow at a section P is
0.5 m. A flat-topped hump is to be placed at the downstream of the section P. Assume
negligible energy loss between section P and hump, and consider 𝑔𝑔 as 9.81 m/s2 . The
maximum height of the hump (expressed in m) which will not change the depth of flow at
section P is
Key: (0.205)
Exp: The maximum height of hump z is given by
E = Emin + z max

 India’s No.1 institute for GATE Training  1 Lakh+ Students trained till date  65+ Centers across India
26
CE-GATE-2016 PAPER-02| www.gateforum.com

q2 3
 y 2
 y c  z max
2gy 2
Q 6
q    2m 2 / s, y  0.5m
B 3
1/3 1/3
 q2   22 
yc       0.74m
 g   9.81 
(2) 2 3
So, 0.5    0.74  z max
2  9.81  (0.5) 2
2
 Zmax  0.205m

48. A penstock of 1 m diameter and 5 km length is used to supply water from a reservoir to an
impulse turbine. A nozzle of 15 cm diameter is fixed at the end of the penstock. The
elevation difference between the turbine and water level in the reservoir is 500 m. consider
the head loss due to friction as 5% of the velocity head available at the jet. Assume unit
weight of water = 10 kN/m3 and acceleration due to gravity (g) = 10 m/s2. If the overall
efficiency is 80%, power generated (expressed in kW and rounded to nearest integer) is
Key: (6570)
Exp: Energy equation,
P V2
H   hL
A 2g
V2 V2
 500   0.05 
2g 2g
2  10  500
V  97.59m / s
1.05
1
Water power  mv12
2
1 
  103   (0.15) 2  (97.59)
2 4
 8212.5kw
Power generated  0  w.p
 0.8  8212.5
 6570kw

49. A tracer takes 100 days to travel from Well-1 to Well-2 which are 100 m apart. The
elevation of water surface in Well-2 is 3m below that in Well-1. Assuming porosity equal to
15%, the coefficient of permeability (expressed in m/day) is
(A) 0.30 (B) 0.45 (C) 1.00 (D) 5.00
Key: (D)
100
Exp: Seepage velocity   1m / day
100
Discharge Velocity = n × seepage velocity = 0.15 ×1=0.15 m/day

 India’s No.1 institute for GATE Training  1 Lakh+ Students trained till date  65+ Centers across India
27
CE-GATE-2016 PAPER-02| www.gateforum.com

h 3
i 
L 100
3
V  k.i  0.15  k   k  5m / day
100

50. A sample of water has been analyzed for common ions and results are presented in the form
of a bar diagram as shown.

The non-carbonate hardness (expressed in mg/L as CaCO3) of the sample is


(A) 40 (B) 165 (C) 195 (D) 205
Key: (A)
Exp: Total hardness = Mg/L of Ca2+ and mg2+
= 4.1 ×50=205mg/L as Caco3
Alkalinity = 3.3×50=165 mg/L as CaCo3
NCH = TH-Alkalinity = 205-165= 40 mg/L

51. A noise meter located at a distance of 30 m from a point source recorded 74 dB. The reading
at a distance of 60 m from the point source would be ______

Key: (67.9)
 60 
Exp: L60  L30  20log10  
 30 
 74  20log10 2
 67.9dB

52. For a wastewater sample, the three-day biochemical oxygen demand at incubation
temperature of 20°C (BOD3day, 20°C) is estimated as 200 mg/L. Taking the value of the first
order BOD reaction rate constant as 0.22 day-1, the five-day BOD (expressed in mg/L) of the
wastewater at incubation temperature of 20°C (BOD5day, 20°c) would be
Key: (276.158)
Exp: Given

 India’s No.1 institute for GATE Training  1 Lakh+ Students trained till date  65+ Centers across India
28
CE-GATE-2016 PAPER-02| www.gateforum.com

(BOD)3  200mg / L
k D  0.22 / day
(BOD)5  ?
 BOD 3  L0 1  e k D t

200  L0 1  e 0.223

200 200
L0  0.66
  413.95
1 e 0.483
(BOD)5  L0 1  e  k D t   413.95 1  e 0.225   276.158 mg / L

53. The critical flow ratios for a three-phase signal are found to be 0.30, 0.25, and 0.25. The total
time lost in the cycle is 10 s. Pedestrian crossings at this junction are not significant. The
respective Green times (expressed in seconds and rounded off to the nearest integer) for the
three phases are
(A) 34, 28, and 28 (B) 40, 25, and 25 (C) 40, 30, and 30 (D) 50, 25, and 25
Key: (A)
Exp: Given
y1  0.30, y2  0.25, y3  0.25
Total cycle time (L) = 10
By Webster method
1.5L  5
Cycle time (C0 ) 
1 y
1.5L  5
(C0 ) 
1   y1  y 2  y3 
1.5  10  5
C0 
1  (0.3  0.25  0.25)
15  5 20
   100s
1  .08 0.2
(C  L)(y1 ) (100  10)  0.30
G1    33.75sec  34sec
y 0.8
(C  L)(y2 ) (100  10)  0.25
G2    28.125sec  28sec
y 0.8
(C  L)y3 (100  10)  0.25
G3    28.125sec  28sec
y 0.8

54. A motorist travelling at 100 km/h on a highway needs to take the next exit, which has a
speed limit of 50 km/h. The section of the roadway before the ramp entry has a downgrade
of 3% and coefficient of friction ( f ) is 0.35. In order to enter the ramp at the maximum
allowable speed limit, the braking distance (expressed in m) from the exit ramp is
________
Key: (92.14)

 India’s No.1 institute for GATE Training  1 Lakh+ Students trained till date  65+ Centers across India
29
CE-GATE-2016 PAPER-02| www.gateforum.com

55. A tall tower was photographed from an elevation of 700 m above the datum. The radial
distances of the top and bottom of the tower from the principal points are 112.50 mm and
82.40 mm, respectively. If the bottom of the tower is at an elevation 250 m above the datum,
then the height (expressed in m) of the tower is _____.
Key: (120.4)
r.h 2
d
Exp: Relief displacement is given by, H  h avg
d  112.5  82.40  30.1mm
h  112.5
 30.1 
700  250
 h  120.4m

 India’s No.1 institute for GATE Training  1 Lakh+ Students trained till date  65+ Centers across India
30
|CE| GATE-2017-PAPER-01 www.gateforum.com

CIVIL ENGINEERING
Q. No. 1 to 25 Carry One Mark Each

1. The ordinates of a 2-hour unit hydrograph for a catchment are given as


Time ( h ) 0 1 2 3 4
d
Ordinate ( m s ) 3
0 5 12 25 41
The ordinate (in m3/s) of a 4-hour unit hydrograph for this catchment at the time of 3 h would
be______
Key: (15)
Exp:
Time 0 1 2 3 4
(1) Ordinate lagged 0 5 12 25 41
(2) Lagged ordinate of 2 hr UH lag by 2hr
0 0 0 5 12

 1+ 2 
Ordinate 4hr UH   0 2.5 6 15 26.5
 2 
Ordinate of 3 hour UH at 3-hr is =15m3/s

2. A uniformly distributed line load of 500 kN/m is acting on the ground surface. Based on
Boussinesq’s theory, the ratio of vertical stress at a depth 2 m to that at 4 m, right below the
line of loading, is
(A) 0.25 (B) 0.5 (C) 2.0 (D) 4.0
Key: (C)
Exp: Due to UDL
2
 
 
2q  1 
Vertical stress =
πz   x  2 
1 +   
 z 
2q
at x = 0; veritcal stress ( σz ) =
πz
σ z1 z L 4
= = =2
σ z2 z1 2

3. According to IS 456-2000, which one of the following statements about the depth of neutral
axis χu,bal for a balanced reinforced concrete section is correct?

(A) χu,bal depends on the grade of concrete only.

(B) χu,bal depends on the grade of steel only.

(C) χu,bal depends on both the grade of concrete and grade of steel.

(D) χu,bal does not depend on the grade of concrete and grade of steel.
 ICP–Intensive Classroom Program  eGATE-Live Internet Based Classes DLP  TarGATE-All India Test Series
 Leaders in GATE Preparations  65+ Centers across India
© All rights reserved by Gateforum Educational Services Pvt. Ltd. No part of this booklet may be reproduced or utilized in any form without the written permission.
|CE| GATE-2017-PAPER-01 www.gateforum.com

Key: (B)
Exp: By limit state method
x u ,lim 0.0035
=
d 0.87f y
0.0055 +
ES
For balanced section
x u ,bal = x u ,lim
x u ,bal 0.0035
=
d 0.87f y
0.0055 +
ES
∴ x u,bal depends upon grade of steel only.

4. Group I lists the type of gain or loss of strength in soils. Group II lists the property or process
responsible for the loss or gain of strength in soils.
Group I Group II
P. Regain of strength with time 1. Boiling
Q. Loss of strength due to cyclic loading 2. Liquefaction
R. Loss of strength due to upward seepage 3. Thixotropy
S. Loss of strength due to remolding 4. Sensitivity
The correct match between Group I and Group II is
(A) P-4, Q-1, R-2, S-3 (B) P-3, Q-1, R-2, S-4
(C) P-3, Q-2, R-1, S-4 (D) P-4, Q-2, R-1, S-3
Key: (C)

5. A runway is being constructed in a new airport as per the International Civil Aviation
Organization (ICAO) recommendations. The elevation and the airport reference temperature
of this airport are 535 m above the mean sea level and 22.65°C, respectively. Consider the
effective gradient of runway as 1%. The length of runway required for a design-aircraft under
the standard conditions is 2000 m. Within the framework of applying sequential corrections as
per the ICAO recommendations, the length of runway corrected for the temperature is
(A) 2223 m (B) 2250 m (C) 2500 m (D) 2750 m
Key: (C)
Exp: Correction for elevation : 7 % increase per 300 m height
7 535
So, correction = × × 2000 = 249.7m
100 300
∴ Corrected length = 2000 + 249.7 = 2249.7 m
Correction for temperature :
std. Atm temp = 15 − 0.0065 × 535 = 11.52 °C
∆T = 22.65 − 11.52 = 11.13°C
2249.7
Correction = × 11.13 = 250.32m
100

 ICP–Intensive Classroom Program  eGATE-Live Internet Based Classes DLP  TarGATE-All India Test Series
 Leaders in GATE Preparations  65+ Centers across India
© All rights reserved by Gateforum Educational Services Pvt. Ltd. No part of this booklet may be reproduced or utilized in any form without the written permission.
|CE| GATE
GATE-2017-PAPER-01 www.gateforum.com

∴ Corrected length = 2249.74 + 250.32


= 2500.02 m
2500.02 − 2000
check : Total correction = × 100
2000
= 25% < 35%

6. A soil sample is subjected to a hydrostatic pressure, σ. The Mohr circle for any point in the
soil sample would be
(A) a circle of radius σ and center at the origin
(B) a circle of radius σ and center at a distance σ from the origin
(C) a point at a distance σ from the origin
(D) a circle of diameter σ and center at the origin
Key: (C)
Exp: σx = σy = σz = σ, τxy = 0
For Mohr’s Circle
2
 σ − σy 
2
σ−σ
Radius (R) =  x  + τ xy = 
2
 +0 =0
2

 2   2 
σx + σ y σ + σ
centre = = =σ
2 2

7. hinged parabolic arch of span L subjected to a uniformly distributed


The figure shows a two-hinged
load of intensity q per unit length.

The maximum bending moment in the arch is equal to


qL2 qL2 qL2
(A) (B) (C) zero (D)
8 12 10
Key: (C)
Exp: Bending moment at any point for two
two-hinged
hinged parabolic arch with uniformly distributed load is
zero.

8. For a steady incompressible laminar flow between two infinite parallel stationary plates, the
shear stress variation is
(A) linear with zero value at the plates
(B) linear with zero value at the center
(C) quadratic with zero value at the Plates
(D) quadratic with zero value at the centre

 ICP–Intensive Classroom Program  eGATE-Live Internet Based Classes DLP  TarGATE-All


All India Test Series
 Leaders in GATE Preparations  65+ Centers across India
© All rights reserved by Gateforum Educational Services Pvt. Ltd. No part of this booklet may be reproduced or utilized in any form without the written permission.
|CE| GATE-2017-PAPER-01 www.gateforum.com

Key: (B)
Exp: We know that velocity variation,
1  −∂p 
v=  ( ty − y )
2
 τ=0
2µ  ∂x 
∂u −1  ∂p 
τ=µ =   ( t − 2y )
∂y 2  ∂x 
y = 0 ⇒ τ = τmax
τmax
t
y= ⇒τ=0
2

9. An elastic bar of length L, uniform cross sectional area A, coefficient of thermal expansion
α , and Young’s modulus E is fixed at the two ends. The temperature of the bar is increased
by T, resulting in an axial stress σ. Keeping all other parameters unchanged, if the length of
the bar is doubled, the axial stress would be
(A) σ (B) 2 σ (C) 0.5σ (D) 0.25α σ
Key: (A)
Exp:

σ = E ( α∆T )
σ varies with ∆T and does not depends upon the length of bar.

10. The number of parameters in the univariate exponential and Gaussian distributions,
respectively, are
(A) 2 and 2 (B) 1 and 2 (C) 2 and 1 (D) 1 and 1
Key: (B)
Exp: Probability density functions of univariable exponential distributes
f ( x ) = λe− dx x ≥ 0
=0 others
where λ is parameter
2
1  x −µ 
1 −  
For Gaussian distribution f ( x ) = e 2 2 

σ 2π
where µ and σ are parameters

11. The wastewater form a city, containing a high concentration of biodegradable organics, is being
steadily discharged into a flowing river at a location S. If the rate of aeration of the river water is
lower than the rate of degradation of the organics, then the dissolved oxygen of the river water
(A) is lowest at the locations S.
(B) is lowest at a point upstream of the location S.
(C) remains constant all along the length of the river.
(D) is lowest at a point downstream of the location S.
Key: (D)

 ICP–Intensive Classroom Program  eGATE-Live Internet Based Classes DLP  TarGATE-All India Test Series
 Leaders in GATE Preparations  65+ Centers across India
© All rights reserved by Gateforum Educational Services Pvt. Ltd. No part of this booklet may be reproduced or utilized in any form without the written permission.
|CE| GATE
GATE-2017-PAPER-01 www.gateforum.com

The reaction rate involving reactants A and B is given by − k [ A ] [ B] . Which one of the
α β
12.
following statements is valid for the reaction to be first –order reaction?
(A) α = 0 and β = 0 (B) α = 1and β = 0 (C) α = 1and β = 1 (D) α = 1and β = 2
Key: (B)
Exp: In chemical kinetics, the order
rder of reaction with respect to given substance is defined as the
index or exponent to which its concentration term in the rate equation is raised.
r = k.[ A ] [ B]
α β

Order of reaction = α + β
For first order reaction, α + β = 1

13. A strip footing is resting on the ground surface of a pure clay bed having an undrained
cohesion Cu. The ultimate bearing capacity of the footing is equal to
(A) 2 πC u (B) πCu (C) ( π + 1) Cu (D) ( π + 2 ) Cu
Key: (D)
Exp: Ultimate bearing capacity = c.N C
For pure clay N C = 5.14 = ( π + 2 )
UBC = ( π + 2 ) C

14. A simply supported beam is subjected to a uniformly distributed load. Which one of the
following statements is true?
(A) Maximum or minimum shear force occurs where the curvature is zero.
(B) Maximum or minimum bending moment occurs where the shear force is zero.
(C) Maximum or minimum bending moment occurs where the curvature is zero.
(D) Maximum bending moment and maximum shear force occur at the same section.
Key: (B)
w
Exp:
A B
L
wL wL
↑ ↑
2 2

SFD wL
+
2
− wL
2

wL2
BM max =
BMD BM max 8

15. A triangular pipe network is shown in the figure.


The head loss in each pipe is given by hf = rQ ,
1.8

with the variables expressed in a consistent set of


units. The value of r for the pipe AB is 1 and for
the pipe BC is 2. If the discharge supplied at the
point A (i.e., 100) is equally divid
divided between the
pipes AB and AC, the value of r (up to two
decimal places) for the pipe AC should be_______
 ICP–Intensive Classroom Program  eGATE-Live Internet Based Classes DLP  TarGATE-All
All India Test Series
 Leaders in GATE Preparations  65+ Centers across India
© All rights reserved by Gateforum Educational Services Pvt. Ltd. No part of this booklet may be reproduced or utilized in any form without the written permission.
|CE| GATE-2017-PAPER-01 www.gateforum.com

Key: (0.62)
Exp: If the discharge supplied at point A is equally divided so Q AB = Q AC = 50 m 3 s
B
θ = 20
r=2
60 20
r =1

θ = 30
A 50 C
100

Σr Q n = 0
⇒ 1 × ( 50 ) − 2 × ( 20 ) − r × ( 50 )
1.8 1.8 1.8
=0
⇒ r × ( 50 ) = 703.84
1.8

⇒ r = 0.62

 tan x 
16. lim  2  is equal to ________
x →0 x − x
 
Key: (-1)
Tanx Tanx
Exp: lt = lt
x →0 x 2 − x x → 0 x ( x − 1)
Tanx 1
= lt . lt = 1 × −1 = −1
x →0 x x →0 ( x − 1)

17. A super-elevation e is provided on a circular horizontal curve such that a vehicle can be
stopped on the curve without sliding. Assuming a design speed v and maximum coefficient of
side friction fmax, which one of the following criteria should be satisfied?
(A) e ≤ f max (B) e > f max

1 − ( f max )
2

(C) no limit on e can be set (D) e =


f max
Key: (A)
Exp: θ = tan θ = e
For no sliding
w
v2 µ + tan θ
≤ e
gR 1 + µ tan θ
v2 θ
≤e+f
gR
For stopped vehicle v=0
f ≥ −e

 ICP–Intensive Classroom Program  eGATE-Live Internet Based Classes DLP  TarGATE-All India Test Series
 Leaders in GATE Preparations  65+ Centers across India
© All rights reserved by Gateforum Educational Services Pvt. Ltd. No part of this booklet may be reproduced or utilized in any form without the written permission.
|CE| GATE-2017-PAPER-01 www.gateforum.com

18. Which one of the following is NOT present in the acid rain?
(A) HNO3 (B) H2SO4 (C) H2CO3 (D) CH3COOH
Key: (D)

19. The accuracy of an Electronic Distance Measuring Instrument (EDMI) is specified as


± (a mm + b ppm). Which one of the following statements is correct?
(A) Both a and b remain constant, irrespective of the distance being measured.
(B) a remains constant and b varies in proportion to the distance being measured.
(C) a varies in proportion to the distance being measured and b remains constant.
(D) Both a and b vary in proportion to the distance being measured.
Key: (B)
Exp: Accuracy of EDMI is generally stated in terms of constant instrument error and measuring
error proportional to distance being measured.
± ( a mm + b ppm )
The first part in this expression indicates a constant instrument error that is independent of
length of line measured. Second component is distance related error.

20. Consider the following partial differential equation:


∂ 2φ ∂ 2φ ∂ 2φ
3 + B + 3 + 4φ = 0
∂x 2 ∂x∂y ∂y 2
For this equation to be classified as parabolic, the value of B2 must be_______
Key: (36)
∂ 2φ ∂ 2φ ∂ 2φ
Exp: Given 3 +B + 3 2 + 4φ = 0 is parabolic.
∂x 2
∂x∂y ∂y
By comparing with general form
∂ 2φ ∂2φ ∂ 2φ  ∂u ∂u 
A 2 +B + C 2 + F  φ, x, y, ,  = 0
∂x ∂x∂y ∂y  ∂x ∂y 
A = 3 ;B = B; C = 3
Condition for parabolic is
B2 − 4AC = 0
B2 − 4 ( 3 × 3) = 0 ⇒ B2 = 36
∴ B2 = 36

21. The matrix P is the inverse of a matrix Q. If I denotes the identity matrix, which one of the
following options is correct?
(A) PQ = I but QP ≠ I (B) QP = I but PQ ≠ I
(C) PQ = I and QP = I (D) PQ − QP = I
Key: (C)
Exp: Given P is inverse of Q
⇒ PQ = QP = I

 ICP–Intensive Classroom Program  eGATE-Live Internet Based Classes DLP  TarGATE-All India Test Series
 Leaders in GATE Preparations  65+ Centers across India
© All rights reserved by Gateforum Educational Services Pvt. Ltd. No part of this booklet may be reproduced or utilized in any form without the written permission.
|CE| GATE
GATE-2017-PAPER-01 www.gateforum.com

22. Vehicles arriving at an intersection from one of the approach road follow the Poisson
distribution. The mean rate of arrival is 900 vehicles per hour. If a gap is defined as the time
difference between two successive vehicle arrivals (with vehicles assumed to be points), the
probability (up to four decimal places) that the gap is greater than 8 seconds is______
Key: (0.1354)
Exp: By Poission’s distribution
p ( h ≥ 8 ) = e −8 λ
900
λ= = 0.256
36500
−8× 1
p ( h ≥ 8) = e 4
= 0.1354

Let x be a continuous variable defined over the interval ( −∞, ∞ ) ,and f ( x ) = e-x-e .
-x
23.

The integral g(x) = ∫ f ( x ) dx is equal to


−x −x
e −e
(A) e (B) e
x
−e
(C) e (D) e − x
Key: (B)
f ( x ) = e− x −e x ∈ ( −∞, ∞ ) is a continuous variable
−x
Exp:
g ( x ) = ∫ f ( x ) dx = ∫ e − x − e dx = ∫ e − x .e − e dx
−x −x

put e− x = t
−e− x dx = dt
 e− t  − t
∴ g ( x ) = ∫ e− t ( −dt ) = −  =e =e
− e− x

 −1 

24. The number of spectral bands in the Enhanced Thematic Mapper sensor on the remote sensing
satellite Landsat-7 is
(A) 64 (B) 10 (C) 8 (D) 15
Key: (C)

25. A 3 m thick clay layer is subjected to an initial uniform pore pressure of 145 kPa as shown in
the figure.

For the given ground conditions, the time (in days, rounded to the nearest integer) required for
90% consolidation would be ________

 ICP–Intensive Classroom Program  eGATE-Live Internet Based Classes DLP  TarGATE-All


All India Test Series
 Leaders in GATE Preparations  65+ Centers across India
© All rights reserved by Gateforum Educational Services Pvt. Ltd. No part of this booklet may be reproduced or utilized in any form without the written permission.
|CE| GATE
GATE-2017-PAPER-01 www.gateforum.com

Key: (1770.833)
Exp: It is single drainage
C V .t TV .H 2 0.85 × 3000 2 0.856 × 9 × 106
TV = ⇒ t = = = min
H2 CV 3 mm 2 / min 3
= 2.55 × 10 6 min = 1770.833days

Q. No. 26 to 55 Carry Two Marks Each

26. A planar truss tower structure is shown in the


th figure.

Consider the following statements about the external and internal determinacies of the truss.
(P) Externally Determinate
(Q) External Static Indeterminacy = 1
(R) External Static Indeterminacy = 2
(S) Internally Determinate
(T) Internal Static Indeterminacy = 1
(U) Internal Static Indeterminacy = 2
Which one of the following options is correct?
(A) P-False; Q-True; R-False;
False; S-False;
S T-False; U-True
(B) P-False; Q-True; R-False;
False; S-False:
S T-True; U-False
(C) P-False; Q-False; R-True;
True; S-False;
S T-False; U-True
(D) P-True; Q-True; R-False;
False; S-True;
S T-False; U-True
Key: (A)
Exp: Dse = r − 3
r = number if sup port reactions = 4
Dse = 4 − 3 = 1
Dsi = number of double diagonals = 2

27. Consider the stepped bar made with a linear elastic material and subjected to an axial load of
1 kN, as shown in the figure.

 ICP–Intensive Classroom Program  eGATE-Live Internet Based Classes DLP  TarGATE-All


All India Test Series
 Leaders in GATE Preparations  65+ Centers across India
© All rights reserved by Gateforum Educational Services Pvt. Ltd. No part of this booklet may be reproduced or utilized in any form without the written permission.
|CE| GATE
GATE-2017-PAPER-01 www.gateforum.com

cross-sectional are of 100 mm2 and 60 mm2, Young’s modulus of


Segments 1 and 2 have cross
2 × 10 MPa and 3 × 10 MPa, and length of 400 mm and 900 mm, respectively. The strain
5 5

energy (in N-mm,


mm, up to one decimal place) in the bar due to the axial load is_____
Key: (35)
1 KN
Exp:
A1
L1 = 400 mm
E1
1 KN
A2
L 2 = 900 mm
E2 1 KN
1 KN
U2L
Strain energy = ∑
2AE
10002 × 400 10002 × 900 1 KN
= +
2 × 100 × 2 × 105 2 × 60 × 3 × 105
= 10 + 25
= 35Nmm

28. Consider the beam ABCD shown in the figure.

For a moving concentrated load of 50 kN on the beam, the magnitude of the maximum
bending moment (in kN-m)
m) obtained at the support C will be equal to______
Key: (200)
Exp: By muller Breslau principle
ILD for moment at C x
x−0= 4
x=4 4B
C D
Load is acting at point B A
4m 4m 10
B.M = 50 × 4 = 200kN − m

 ICP–Intensive Classroom Program  eGATE-Live Internet Based Classes DLP  TarGATE-All


All India Test Series
 Leaders in GATE Preparations  65+ Centers across India
© All rights reserved by Gateforum Educational Services Pvt. Ltd. No part of this booklet may be reproduced or utilized in any form without the written permission.
|CE| GATE
GATE-2017-PAPER-01 www.gateforum.com

29. A column is subjected to a load through a bracket as shown in the figure.

The resultant force (in kN, up to one decimal place) in the bolt 1 is_____
Key: (5.9 to 6.1)
Exp: P = 10KN,e = 15cm, r1 = r2 = r3 = r4 = 5 cm
P 10
Fd = = = 2.5KN
4 4
Force in bolt 1 due to moment

Pe r1 10 × 15 × 5
Fs = = = 7.5KN
∑ r 2 4 × ( 5)2

135°

( 2.5 ) + ( 7.5 ) + 2 × 2.5 × 7.5cos135°  6 KN


2 2
FR =

30. The activity details of a project are given below:

Activity Depends on Duration (in days)


P -- 6
Q P 15
R Q,T 12
S R 16
T P 10
U Q,T 14
V U 16

The estimated minimum time (in days) for the completion of the project will be________

 ICP–Intensive Classroom Program  eGATE-Live Internet Based Classes DLP  TarGATE-All


All India Test Series
 Leaders in GATE Preparations  65+ Centers across India
© All rights reserved by Gateforum Educational Services Pvt. Ltd. No part of this booklet may be reproduced or utilized in any form without the written permission.
|CE| GATE
GATE-2017-PAPER-01 www.gateforum.com

Key: (51)
Exp: Activity on arrow (AoA) diagram:
diagram

3 R = 12 5
T = 10 S = 16
P=6
1 2 4 7 TE = 51
TE = 6 Q = 15 U = 14
TE = 0
TE = 21 6 V = 16
TE = 15
Time along path 1- 2- 4- 6- 7
= 6+15+14+16 = 51 days

31. The value of M in the beam ABC shown in the figure is such that the joint B does not rotate.

The value of support reaction (in kN) at B should be equal to______


Key: (60)
4EIθB 2EIθA w 2
Exp: M BA = + +
4 6 12 30 kN m
30 × 16 40
⇒ M BA =0 + 0 + = 40
12 4m RB
3EIθB
M BC = + M F BC = 0
6
M = M BA + M BC
⇒ M = M BA = 40
W 4 ML2 R B × L3
⇒ + =
8EI 2EI 3EI
30 × 4 40 RB
⇒ + = ⇒ R B = 60kN
8 2×4 3

32. Two wastewater streams A and B, having an identical ultimate BOD are getting mixed to
form the stream C. The temperature of the stream A is 20°C and the temperature of the stream
C is 10°C. It is given that
• day BOD of the stream A measured at 20°C=50 mg/l
The 5-day
• BOD rate constant (base 10) at 20°C=0.115 per day
• Temperature coefficient = 1.135
day BOD (in mg/l, up to one decimal place) of the stream C, calculated at 10°C,
The 5 –day
is______
Key: (21.21)
Exp:

 ICP–Intensive Classroom Program  eGATE-Live Internet Based Classes DLP  TarGATE-All


All India Test Series
 Leaders in GATE Preparations  65+ Centers across India
© All rights reserved by Gateforum Educational Services Pvt. Ltd. No part of this booklet may be reproduced or utilized in any form without the written permission.
|CE| GATE-2017-PAPER-01 www.gateforum.com

stream A stream B

T = 20°
BOD5 = 50mg / 

T1 = 10°C
stream C
K D , 20°C = 0.115d − 1

BOD5 = BOD U × (1 − 10− K D .t )


50
⇒ ( BOD U )A = = 68.13 mg / 
1 − 10−0.115×5
10 − 20
K D ,10°C = K D , 20°C × (1.135 )
−10
= 0.115 × (1.135 ) = 0.0324
( BOD U )A = ( BOD U )B
So, ( BOD U )C = 68.13 mg / 
For C,
( BOD5 )10°C = BOD U (1 − 10− K D ,10× t
)
= 68.13 × (1 − 10−0.0324×5 )
= 21.21mg / 

33. A particle of mass 2 kg is travelling at a velocity of 1.5 m/s. A force f(t)=3t2 (in N) is applied
to it in the direction of motion for a duration of 2 seconds, where t denotes time in seconds.
The velocity (in m/s, up to one decimal place) of the particle immediately after the removal of
the force is________
Key: (5.5)
Exp: f ( t ) = 3t 2
m.Q = 3t 2
dv
m. = 3t 2
dt
m.dv = 3t 2 dt
V 2
m ∫ dv = ∫ 3t 2 dt
1.5 0

t3 2
2 × ( v − 1.5 ) = 3.
3 0
2
2 ( v − 1.5) = 3 × = 8
3
v − 1.5 = 4
v = 5.5m / s
 ICP–Intensive Classroom Program  eGATE-Live Internet Based Classes DLP  TarGATE-All India Test Series
 Leaders in GATE Preparations  65+ Centers across India
© All rights reserved by Gateforum Educational Services Pvt. Ltd. No part of this booklet may be reproduced or utilized in any form without the written permission.
|CE| GATE
GATE-2017-PAPER-01 www.gateforum.com

34. The queue length (in number of vehicles) versus


time (in seconds) plot for an approach to a
signalized intersection with the cycle length of 96
seconds is shown in the figure (not drawn to
scale).
At time t = 0, the light has just turned red. The
effective green time is 36 seconds, during which
vehicles discharge at the saturation flow rate, s (in
vph). Vehicles arrive at a uniform rate, v (in vph),
throughout the cycle. Which one of the following statements is TRUE?
(A) v = 600 vph, and for this cycle, the average stopped delay per vehicle = 30 seconds
(B) s = 1800 vph, and for this cycle, the average stopped delay per vehicle = 28.125 seconds
(C) v = 600 vph, and for this cycle, the average stopped delay per vehicle = 45 seconds
(D) s = 1200 vph, and for this cycle, the average stopped delay per vehicle = 28.125 seconds
Key: (B)

35. For the function f ( x ) = a +bx, 0 ≤ x ≤ 1, to be a valid probability density function, which one
of the following statements is correct?
(A) a = 1, b = 4 (B) a = 0.5, b = 1 (C) a = 0, b = 1 (D) a = 1, b = -1
Key: (B)
Exp: f ( x ) = a + bx 0 ≤ x ≤ 1 is a valid probability density function
1
i.e., ∫ f ( x )dx = 1
0
1
∫ ( a + bx ) dx = 1
0
1
 bx 2  b
 ax +  =1⇒ a + =1
 2 0 2
⇒ 2a + b = 2
a=0.5, b=1
=1 satisfies the above relation

36. The infinite sand slope shown in the figure is on the


verge of sliding failure. The ground water table
coincides with the ground surface. Unit weight of
water γw = 9.81kN m .
3

The value of the effective angle of internal friction (in


degrees, up to one decimal place) of the sand is
________
Key: (34.335)
γ sub .tan φ
Exp: F.O.S =
γ sat .tan β
( 21 − 9.8 ) tan φ 21 × tan 20
1= ⇒ tan φ =
21 × tan ( 20° ) ( 21 − 9.81)
tan = 0.683
φ = 34.335

 ICP–Intensive Classroom Program  eGATE-Live Internet Based Classes DLP  TarGATE-All


All India Test Series
 Leaders in GATE Preparations  65+ Centers across India
© All rights reserved by Gateforum Educational Services Pvt. Ltd. No part of this booklet may be reproduced or utilized in any form without the written permission.
|CE| GATE
GATE-2017-PAPER-01 www.gateforum.com

37. A sluice gate used to control the flow in a horizontal channel of unit width is shown in the
figure.

It is observed that the depth of flow is 1.0 m upstream of the gate, while the depth is 0.2 m
downstream of the gate. Assuming a smooth flow transition across the sluice gate, i.e.,
without any energy loss, and the acceleration due to gravity
g as 10 m/s2, the discharge (in
3
m /s, up to two decimal places) passing under the sluice gate is_______
Key: (0.82)
Exp: Given Energy loss is zero ⇒ E1 = E 2
V12 V2
y1 + = y2 + 2
2g 2g
Q = AV
Q
V= y1 = 1.0
A
y 2 = 0.2 m
Q12 Q22
y1 + = y 2 +
2g.A12 2gA 2 2
Q22 Q12 (1) ( 2)
y1 − y 2 = −
2gA 2 2 2g.A12
Q1 = Q2 = Q

Q2  1 1 
1 − 0.2 =  2 − 2
2g  A 2 A1 
Q2  1 1
0.8 =  2
− 2
2g  0.2 1 
Q = 0.82 m3 / s

38. Group I contains three broad classes of irrigation supply canal outlets. Group II presents
hydraulic performance attributes.

Group I Group II
P. Non-modular outlet 1. Outlet discharge depends on the water levels in
both the supply canal as well as the receiving
water course
Q. Semi-modular outlet 2. Outlet discharge is fixed and is independent of
the water levels in both the supply canal as
well as the receiving water course
R. Modular outlet 3. Outlet discharge depends only on the water
level in the supply canal

 ICP–Intensive Classroom Program  eGATE-Live Internet Based Classes DLP  TarGATE-All


All India Test Series
 Leaders in GATE Preparations  65+ Centers across India
© All rights reserved by Gateforum Educational Services Pvt. Ltd. No part of this booklet may be reproduced or utilized in any form without the written permission.
|CE| GATE-2017-PAPER-01 www.gateforum.com

The correct match of the items in Group I with the items in Group II is
(A) P-1; Q-2; R-3 (B) P-3; Q-1; R-2
(C) P-2; Q-3; R-1 (D) P-1; Q-3; R-2
Key: (D)

 5 −1
39. Consider the matrix   . Which one of the following statements is TRUE for the
4 1 
eigenvalues and eigenvectors of this matrix?
(A) Eigenvalue 3 has a multiplicity of 2, and only one independent eigenvector exists.
(B) Eigenvalue 3 has a multiplicity of 2, and two independent eigenvectors exist.
(C) Eigenvalue 3 has a multiplicity of 2, and no independent eigenvector exists.
(D) Eigenvalues are 3 and -3, and two independent eigenvectors exist.
Key: (A)
5 −1
Exp: Let A =  
4 1 
Characteristic equations is λ 2 − 6λ + 9 = 0 ⇒ λ = 3,3
Eigen value 3 has multiplicity 2.
Eigen vectors corresponding to λ = 3 is ( A − 3I ) X = 0
 5 − 3 −1  x   0 
   =  
 4 1 − 3  y   0 
 2 −1  x   0 
   =  
 4 −2  4   0 
 2 −1 x   0 
R 2 → R 2 − 2R1 ⇒    =  
 0 0  4   0 
e(A) = 1
Number of linearly independent eigen vectors corresponding to eigen value λ = 3 is
n-r=2-1=1 where n= no. of unknowns, r= rank of ( A − λI )
∴ One linearly independent eigen vector exists corresponding to λ = 3

40. The laboratory test on a soil sample yields the following results: natural moisture content =
18%, liquid limit = 60%, plastic limit = 25%, percentage of clay sized fraction = 25%.
The liquidity index and activity (as per the expression proposed by skempton ) of the soil,
respectively, are
(A) -0.2 and 1.4 (B) 0.2 and 1.4
(C) -1.2 and 0.714 (D) 1.2 and 0.714
Key: (A)
Exp: Liquidity Index ( IL ) = 1 − IC
wL − w
IC =
wL − wP
60 − 18 6
= =
60 − 25 5

 ICP–Intensive Classroom Program  eGATE-Live Internet Based Classes DLP  TarGATE-All India Test Series
 Leaders in GATE Preparations  65+ Centers across India
© All rights reserved by Gateforum Educational Services Pvt. Ltd. No part of this booklet may be reproduced or utilized in any form without the written permission.
|CE| GATE
GATE-2017-PAPER-01 www.gateforum.com

6 −1
IL = 1 − = = −0.2
5 5
IP w − w P 60 − 25 7
Activity = = L = = = 1.4
% clay % clay 25 5

dQ
41. The solution of the equation + Q = 1with Q = 0 at t = 0 is
dt
(A) Q ( t ) = e− t − 1 (B) Q ( t ) = 1 + e− t

(C) Q ( t ) = 1 − et (D) Q ( t ) = 1 − e− t
Key: (D)

Exp: + θ = 1 and θ = 0 at t = 0
dt
Comparing with first order linear differential equations
dQ
+ pQ = q where p = 1;q = 1
dt
I.F = ∫ pdt = e t
e

Q. ( IF ) = ∫ 1. ( IF ) dt + c
Q.e t = ∫ e t dt + c
Q.e t = e t + c
Q = 0 at t = 0 ⇒ 0.1 = 1 + c ⇒ c = −1
∴ Q.e t = e t − 1 ⇒ Q = 1 − e − t

42. Water flows through a 90° bend in a horizontal plane as depicted in the figure.

A pressure of 140 kPa is measured at section 1-1.


1 1. The inlet diameter marked at section 11-1 is
27 14
cm, while the nozzle diameter marked at section 2-2
2 is cm. Assume the following:
π π
(i) Acceleration due to gravity = 10 m/s2.
(ii) Weights of both the bent pipe segment as well as water are negligible.
(iii) Friction across the bend is negligible.
The magnitude of the force (in kN, up to two decimal places) that would be required to hold
the pipe section is______
Key: (2.50 to 3.75)

 ICP–Intensive Classroom Program  eGATE-Live Internet Based Classes DLP  TarGATE-All


All India Test Series
 Leaders in GATE Preparations  65+ Centers across India
© All rights reserved by Gateforum Educational Services Pvt. Ltd. No part of this booklet may be reproduced or utilized in any form without the written permission.
|CE| GATE-2017-PAPER-01 www.gateforum.com

43. A pre-tensioned rectangular concrete beam 150 mm wide and 300 mm depth is prestressed
with three straight tendons, each having a cross-sectional area of 50 mm2, to an initial stress of
1200 N/mm2. The tendons are located at 100 mm from the soffit of the beam. If the modular
ratio is 6, the loss of prestressing force (in kN, up to one decimal place) due to the elastic
deformation of concrete only is ______.
Key: (4.8)
Exp: Stress = 1200 N / mm 2
P
= 1200
A 150
P = 1200 × ( 3 × 50 )
= 18 × 104 N
P Pe
fc = + y
A I 300
18× 104 18 × 104 × 50 × 50
= + 50 mm
150 × 300 3003
150 ×
12 100 mm
= 4 + 1.33 = 5.33 N/ mm 2
Loss due to elastic deformation = m.f c = 6 × 5.33 = 31.98
Prestress force = 31.98 × 3 × 50 = 4797 N = 4.8 KN

44. The spherical grit particles, having a radius of 0.01mm and specific gravity of 3.0, need to be
separated in a settling chamber. It is given that
• g = 9.81 m/s2
• the density of the liquid in the settling chamber = 1000 kg/m3
• the kinematic viscosity of the liquid in the settling chamber = 10-6 m2/s
Assuming laminar conditions, the settling velocity (in mm/s, up to one decimal place) is_____
Key: (0.436)
Exp: Kinematic viscocity = 10 −6 m 2 / s
µ
= 10−6 m 2 / s
ρ
µ = 10−6 × 1000 = 10−3 N − S / m2
( γ s − γ w ) .d 2
Settling velocity ( Vs ) =
18µ

( 9810 × 3 − 9810 ) × ( 2 × 0.01 × 10−3 )


2

Vs = = 4.36 × 10−4 m / s = 0.436 mm / s


18 × 10−3

45. The equivalent sound power level (in dB) of the four sources with the noise levels of 60 dB,
69 dB, 70 dB and 79 dB is_______
Key: (79.9)
Exp: Equivalent sound power level

 ICP–Intensive Classroom Program  eGATE-Live Internet Based Classes DLP  TarGATE-All India Test Series
 Leaders in GATE Preparations  65+ Centers across India
© All rights reserved by Gateforum Educational Services Pvt. Ltd. No part of this booklet may be reproduced or utilized in any form without the written permission.
|CE| GATE-2017-PAPER-01 www.gateforum.com

Li
= 10.log1010
 60 69 70 79

= 10log 10 10 + 10 10 + 10 10 + 10 10 
 
L eq = 79.9dB

du
46. Consider the equation = 3t 2 + 1with u = 0 at t = 0. This is numerically solved by using the
dt
forward Euler method with a step size. ∆t = 2. The absolute error in the solution at the end of
the first time step is_________
Key: (8)
Exp: Approximation value by Euler's Method:
du
= 3t 2 + 1 ; u ( 0 ) = 0; h = ∆t = 2
dt
u ( 2 ) = u ( 0 ) + hf ( 0,0 ) , f ( u, t ) = 3t 2 + 1
= 0 + 2 ( 0 + 1) = 2
Exact value:
du = ( 3t 2 + 1) dt ( var iable separable )
⇒ u = t 3 + t + c is sloution
u (0) = 0 ⇒ 0 = c
u = t3 + t
u ( 2 ) = 8 + 2 = 10
∴ absolute error = 10 − 2 = 8

47. It is proposed to drive H-piles up to a depth of 7 m at a construction site. The average surface
area of the H-pile is 3 m2 per meter length. The soil at the site is homogeneous sand, having
an effective friction angle of 32°. The ground water table (GWT) is at a depth of 2 m below
the ground surface. The unit weights of the soil above and below the GWT are 16 kN/m3 and
19 kN/m3, respectively. Assume the earth pressure coefficient, K= 1.0, and the angle of wall
friction, δ = 23°. The total axial frictional resistance (in kN, up to one decimal place)
mobilized on the pile against the driving is________
Key: (390.7)
Exp:
A

2m
2m
B

5m 16 × 2 = 32 KN / m 2
C

 ICP–Intensive Classroom Program  eGATE-Live Internet Based Classes DLP  TarGATE-All India Test Series
 Leaders in GATE Preparations  65+ Centers across India
© All rights reserved by Gateforum Educational Services Pvt. Ltd. No part of this booklet may be reproduced or utilized in any form without the written permission.
|CE| GATE-2017-PAPER-01 www.gateforum.com

32
Average stress in AB = = 16KN / m 2
2
θsf = K × σavg × tan θ × AS1
= 1 × 16 × tan 23° × ( 3 × 2 ) = 40.75 KN

BC:

Effective stress variation


32

77.95
32 + 77.95
σavg = = 54.97 kN / m 2
2
A BC = 3 × 5 = 15 m 2
θsf = Kσavg × tan θ × A
= 1 × 54.97 × tan 23° × 15 = 350KN

Total axial frictional resistance = 350+40.75 = 390.75KN

48. The wastewater having an organic concentration of 54 mg/l is flowing at a steady rate of
0.8m3/day through a detention tank of dimensions 2m × 4m × 2m. If the contents of the tank
are well mixed and the decay constant is 0.1 per day, the outlet concentration (in mg/l, up to
one decimal place) is ______
Key: (17.9 to 18.1)

49. The radius of a horizontal circular curve on a highway is 120 m. The design speed is 60
km/hour, and the design coefficient of lateral friction between the tyre and the road surface is
0.15. The estimated value of superelevation required (if full lateral friction is assumed to
develop), and the value of coefficient of friction needed (if no superelevation is provided)
will, respectively, be
1 1
(A) and0.10 (B) and0.37
11.6 10.5
1 1
(C) and0.24 (D) and0.24
11.6 12.9
Key: (C)
5
Exp: v = 60km / hr = 60 × = 16.67m / s
18
v2
R = 120 m; f = 0.15; e + f =
gR

 ICP–Intensive Classroom Program  eGATE-Live Internet Based Classes DLP  TarGATE-All India Test Series
 Leaders in GATE Preparations  65+ Centers across India
© All rights reserved by Gateforum Educational Services Pvt. Ltd. No part of this booklet may be reproduced or utilized in any form without the written permission.
|CE| GATE-2017-PAPER-01 www.gateforum.com

16.67 2
e + 0.15 = = 0.236
9.81 × 120
1
e = 0.236 − 0.15 = 0.086 =
11.6
v2
at e = 0; e + f =
gR
v2
f= = 0.236 ≅ 0.24
gR

50. Consider two axially loaded columns, namely, 1 and 2, made of a linear elastic material with
Young’s modulus 2 × 10 5 MPa, square cross-section with side 10 mm, and length 1 m. For
Column 1, one end is fixed and the other end is free. For Column 2, one end is fixed and the
other end is pinned. Based on the Euler’s theory, the ratio (up to one decimal place) of the
buckling load of Column 2 to the buckling load of Column 1 is ________
Key: (8)
πEI
Exp: Pcr =
 2 eff
For column -1; One end is fixed and other is free
 eff = 2
πEI
Pcr (1) =
4 2
For column-2
One end is fixed and other is pinned.
 eff = 
2
πEI
Pcr (2) = 2
  
 
 2
2πEI
=
2
2πEI
Pcr (2) 2
⇒ =  =8
Pcr (1) πEI
4 2
51. The observed bearings of a traverse are given below:
Line Bearing Line Bearing
PQ 46o 15′ QP 226o 15′
QR 108o 15′ RQ 286o 15′
RS 201o 30′ SR 20o 30′
ST 321o 45′ TS 141o 45′
The stations(s) most likely to be affected by the local attraction is/are

 ICP–Intensive Classroom Program  eGATE-Live Internet Based Classes DLP  TarGATE-All India Test Series
 Leaders in GATE Preparations  65+ Centers across India
© All rights reserved by Gateforum Educational Services Pvt. Ltd. No part of this booklet may be reproduced or utilized in any form without the written permission.
|CE| GATE-2017-PAPER-01 www.gateforum.com

(A) Only R (B) Only S (C) R and S (D) P and Q


Key: (A)
Exp: Line Bearing Back bearing Diff
PQ 46°15' 226°15' 1800
QR 108°15' 286°15' 1780
RS 201°15' 20°15' 1810
ST 32°15' 141°45' 1800
So, local attraction at only R

52. A 1 m wide rectangular channel has a bed slope of 0.0016 and the Manning’s roughness
coefficient is 0.04. Uniform flow takes place in the channel at a flow depth of 0.5 m. At a
particular section, gradually varied flow (GVF) is observed and the flow depth is measured as
0.6 m. The GVF profile at that section is classified as
(A) S1 (B) S2 (C) M1 (D) M2
Key: (C)
Exp: y n = 0.5m
1
 q2  3
yc =  
 g 
∴ For wide rec tan gular Channel 
1 2 1
 
Q = .A.R 3 .S 2
n  R = A = B.y = y 
 P B 
1 2 1
= × (1 × 0.5 ) × ( 0.5 ) 3 .( 0.0016 ) 2 = 0.315m3 / s
0.04 M1
Q 0.315 yn
q= = = 0.315
B 1 0.6 m M2
0.5
1 yc
 0.315  3
yc =   = 0.318m 0.3 M3
 9.81 
y n > yc ⇒ mild slope

53. A consolidated undrained CU ( ) triaxial compression test is conducted on a normally


consolidated clay at a confining pressure of 100 kPa. The deviator stress at failure is 80 kPa,
and the pore-water pressure measured at failure is 50 kPa. The effective angle of internal
friction (in degrees, up to one decimal place) of the soil is_________
Key: (26.4)
Exp: σc = 100kPa; σ d = 80kPa
σ1 = σc + σd = 100 + 80 = 180kPa
σ3 = σc = 100kPa
u = 50kPa
σ1 = 180 − 50 = 130kPa
σ3 = 100 − 50 = 50kPa
 ICP–Intensive Classroom Program  eGATE-Live Internet Based Classes DLP  TarGATE-All India Test Series
 Leaders in GATE Preparations  65+ Centers across India
© All rights reserved by Gateforum Educational Services Pvt. Ltd. No part of this booklet may be reproduced or utilized in any form without the written permission.
|CE| GATE-2017-PAPER-01 www.gateforum.com

For normally consolidated clay c=0


σ1 = σ3 N φ + 2c N φ
 1 + sin φ 
σ1 = σ3  
 1 − sin φ 
  φ 
130 = 50  tan 2  45 +  
  2 

(
tan 45 + φ
2 )= 130
50
= 1.6124

φ
⇒ 45 + = 58.193
2
φ = 2 ( 58.193 − 45 ) = 26.387 = 26.4

54. An effective rainfall of 2-hour duration produced a flood hydrograph peak of 200 m3/s. The
flood hydrograph has a base flow of 20 m3/s. If the spatial average rainfall in the watershed
for the duration of storm is 2 cm and the average loss rate is 0.4 cm/hour, the peak of 2-hour
unit hydrograph (in m3/s-cm, up to one decimal place) is_________
Key: (150)
Exp: Food hydrograph peak = 200 m 3 s
Baseflow = 20 m3 s
Excess rainfall = 2cm, φ= 0.4cm h
Effective rainfall = 2 − 0.4 × 2 = 1.2cm
Peak of DRH = 200 − 20 = 180 m3 s
let peak of UH beQp
So Qp × 1.2 = 180
⇒ Q p = 150m3 s

55. The following observations are made while testing aggregate for its suitability in pavement
construction:
i. Mass of oven-dry aggregate in air = 1000 g
ii. Mass of saturated surface-dry aggregate in air = 1025 g
iii. Mass of saturated surface-dry aggregate under water = 625 g
Based on the above observations, the correct statement is
(A) bulk specific gravity of aggregate = 2.5 and water absorption = 2.5 %
(B) bulk specific gravity of aggregate = 2.5 and water absorption = 2.4 %
(C) apparent specific gravity of aggregate = 2.5 and water absorption = 2.5%
(D) apparent specific gravity of aggregate = 2.5 and water absorption = 2.4 %
Key: (A)
Exp: Mass of oven dry aggregate w a = 1000g
Mass of water in saturated surface dry aggregate = w w

 ICP–Intensive Classroom Program  eGATE-Live Internet Based Classes DLP  TarGATE-All India Test Series
 Leaders in GATE Preparations  65+ Centers across India
© All rights reserved by Gateforum Educational Services Pvt. Ltd. No part of this booklet may be reproduced or utilized in any form without the written permission.
|CE| GATE-2017-PAPER-01 www.gateforum.com

So, w a + w w = 1025
⇒ w w = 25g
Mass of saturated surface dry aggregate under water = 625 g
⇒ w a − v a ρ w = 625g ( v a vol of aggregate )
1000 − 625
⇒ Va = = 375CC
1
ww
volume of void ( v v ) = volof water = v w = = 25CC
pw
wa
∴ Bulk density of aggregate ρba =
va + v v
1000
= g cc
375 + 25
∴ Bulk specific gravity of aggregate
ρba 2.5
= = = 2.5
ρw 1
ww 25
water absorption = × 100 = × 100 = 2.5%
wa 1000

General Aptitude
Q. No. 1 - 5 Carry One Mark Each

1. Consider the following sentences:


All benches are beds. No bed is a bulb. Some bulbs are lamps.
Which of the following can be inferred?
i. Some beds are lamps.
ii. Some lamps are beds.
(A) Only i (B) Only ii
(C) Both i and ii (D) Neither i nor ii

Key: (D)
Exp:
beds
beds bulbs bulbs lamps
benches

A B C
Since there is no direct relation given between lamps and beds. So, neither will be correct

2. The following sequence of numbers is arranged in increasing order: 1, x, x, x, y , y ,9,16,18.


Given that the mean and median are equal, and are also equal to twice the mode, the value of
y is
(A) 5 (B) 6 (C) 7 (D) 8
Key: (D)
Exp: Given, Mean = Median = 2Mode

 ICP–Intensive Classroom Program  eGATE-Live Internet Based Classes DLP  TarGATE-All India Test Series
 Leaders in GATE Preparations  65+ Centers across India
© All rights reserved by Gateforum Educational Services Pvt. Ltd. No part of this booklet may be reproduced or utilized in any form without the written permission.
|CE| GATE-2017-PAPER-01 www.gateforum.com

⇒ Mean = Median = 2x [∵ Mode = x ] → (1)


3x + 2y + 44
∴ Mean of the data =
9
3x + 2y + 44
⇒ 2x = ⇒ 15x − 2y = 44 → (2)
9
Median of the data = y → (3)
∴ y = 2x → (4) [∵ Median = 2Mode ]
From (2);11x = 44 ⇒ x = 4; ∴y = 8

3. The bacteria in milk are destroyed when it _________ heated to 80 degree Celsius.
(A) would be (B) will be (C) is (D) was
Key: (C)

4. If the radius of a right circular cone is increased by 50%, its volume increases by

(A) 75% (B) 100% (C) 125%` (D) 237.5%


Key: (C)
Exp: Given, radius of a right circular cone is increased by 50%.
Let, height of the circular cone=(h)
1
Initially, Volume of the cone(V) = πR 2 h ......... (1)
3
1
New volume of the cone(V') = πr 2 (1.5R ) h ......... ( 2 )
2

3
1 2 1
πR h = π ( 2.25 ) h = 2.25v
2
From (1) and (2);
3 3
 2.25v − v 
Hence increases by 125% as 
 v  × 100 = 125%
5. __________ with someone else’s email account is now very serious offence.

(A) Involving (B) Assisting (C) Tampering (D) Incubating

Key: (C)

Q. No. 6- 10 Carry Two Marks Each

6. Students applying for hostel rooms are allotted rooms in order of seniority. Students already
staying in a room will move if they get a room in their preferred list. Preferences of lower
ranked applicants are ignored during allocation.
Given the data below, which room will Ajit stay in?

Names Student Seniority Current room Room preference list


Amar 1 P R,S,Q

 ICP–Intensive Classroom Program  eGATE-Live Internet Based Classes DLP  TarGATE-All India Test Series
 Leaders in GATE Preparations  65+ Centers across India
© All rights reserved by Gateforum Educational Services Pvt. Ltd. No part of this booklet may be reproduced or utilized in any form without the written permission.
|CE| GATE-2017-PAPER-01 www.gateforum.com

Akbar 2 None
Anthony 3 Q P
Ajit 4 S Q,P,R

(A) P (B) Q (C) R (D) S

Key: (B)
Exp: Amar → R  As per their preferences given
Akbar → S 
Antony → P 

Ajit → Q 

7. The bar graph below shows the output of five carpenters over one month, each of whom made
different items of furniture: Chairs, tables, and beds.

20 Bed
18 Table
Number of furniture items

Chair
16
14
12
10
8
6
4 C1 C2 C3 C4 C5
2
0
Carpenter ( C )

Consider the following statements.


i. The number of beds made by carpenter C2 is exactly the same as the same as the number
of tables made by carpenter C3.
ii. The total number of chairs made by all carpenters is less than the total number of tables.
Which one of the following is true?
(A) Only i (B) Only ii
(C) Both i and ii (D) Neither i nor ii
Key: (C)
Exp: (i) The number of beds made by carpenter C2 is exactly the same as the number of tables
made by carpenter C3

i.e., beds made by carpenter C2 = 8 = tables made by carpenter C3 [∵ From the bar graph ]
So,(i) is correct.
(ii) Total Number of tables made by all carpenters=31.
Total Number of chairs made by all carpenters=23

 ICP–Intensive Classroom Program  eGATE-Live Internet Based Classes DLP  TarGATE-All India Test Series
 Leaders in GATE Preparations  65+ Centers across India
© All rights reserved by Gateforum Educational Services Pvt. Ltd. No part of this booklet may be reproduced or utilized in any form without the written permission.
|CE| GATE-2017-PAPER-01 www.gateforum.com

∵ 23 < 31
∴ (ii) is correct

The last digit of (2171) + (2172) + (2173) + (2174) is


7 9 11 13
8.
(A) 2 (B) 4 (C) 6 (D) 8
Key: (B)
Exp: The last digit of (2171)7 = 1
The last digit of (2172)9 = (2172)8 (2172)
= (2172) 4n (2172) [ Where, n = 2]
= 6 × 2 = 12 ∵ ( 2172 ) = Last digit = 6 
4n
 
Last digit of (2172) 4n =6
The last digit of (2173)11 = (2173)8 (2173)3
= (2173) 4n (2173)3 [ Where, n = 2]
Last digit of (2173) 4n = 1 = 1× 7 = 7
The last digit of (2174)13 = (2174)12 (2174)
= (2174)2n (2174) [∵ n = 6]
= 6 × 4 ∵ ( 2174 ) = last digit = 6 
2n
 
Last digit of (2174) 2n = 6 = 24
∴ The last digit of (2171)7 + (2172)9 + (2173)11 + (2174)13
= 1 + 2 + 7 + 4 = 14

9. Two machines M1 and M2 are able to execute any of four jobs P, Q, R and S. The machines
can perform one job on one object at a time. Jobs P, Q, R and S take 30 minutes, 20 minutes, 60
minutes and 15 minutes each respectively. There are 10 objects each requiring exactly 1 job.
Job P is to be performed on 2 objects. Job Q on 3 objects. Job R on 1 object and Job S on 4
objects. What is the minimum time needed to complete all the jobs?
(A) 2 hours (B) 2.5 hours (C) 3 hours (D) 3.5 hours

Key: (A)

Exp:

 ICP–Intensive Classroom Program  eGATE-Live Internet Based Classes DLP  TarGATE-All India Test Series
 Leaders in GATE Preparations  65+ Centers across India
© All rights reserved by Gateforum Educational Services Pvt. Ltd. No part of this booklet may be reproduced or utilized in any form without the written permission.
|CE| GATE-2017-PAPER-01 www.gateforum.com

M1 M2
60 ← R S ← 15min R takes 60 min s;
30 ← P S ← 15min Stakes15mins;
30 ← P S ← 15min Q takes 20 min;
S ← 15min P takes30 min;
Q ← 20 min
Q ← 20 min
Q ← 20 min
2hrs 2hrs

2hrs

(OR)

P = 30min × 2=60 min


Q = 20min × 3=60 min
R = 60min × 1=60 min
P = 15min × 4=60 min
M1 P Q = 2 hrs
M2 R S = 2 hrs

10. The old concert hall was demolished because of fears that the foundation would be affected by
the construction tried to mitigate the impact of pressurized air pockets created by the excavation
of large amounts of soil. But even with these safeguards, it was feared that the soil below the
concert hall would not be stable.
From this, one can infer that
(A) The foundations of old buildings create pressurized air pockets underground, which are
difficult to handle during metro construction.
(B) Metro construction has to be done carefully considering its impact on the foundations of
existing buildings.
(C) Old buildings in an area form an impossible hurdle to metro construction in that area.
(D) Pressurized air can be used to excavate large amounts of soil from underground areas.

Key: (B)

 ICP–Intensive Classroom Program  eGATE-Live Internet Based Classes DLP  TarGATE-All India Test Series
 Leaders in GATE Preparations  65+ Centers across India
© All rights reserved by Gateforum Educational Services Pvt. Ltd. No part of this booklet may be reproduced or utilized in any form without the written permission.
|CE| GATE-2017-PAPER-02 www.gateforum.com

Civil Engineering

Q. No. 1 – 25 Carry One Mark Each

1. Consider a rigid retaining wall with partially submerged cohesionless backfill with a
surcharge. Which one of the following diagrams closely represents the Rankine’s active earth
pressure distribution against this wall?
(A) (B)

(C) (D)

Key: (B)

2. The VPI (vertical point of intersection) is 100 m away (when measured along the horizontal)
from the VPC (vertical point of curvature). If the vertical curve is parabolic, the length of the
curve (in meters and measured along the horizontal) is _______
Key: (200)
Exp: VPI is horizontally midway between VPC and VPT
VPC = VPI – L/2
⇒ O =100 − L 2
⇒ L = 200m

3. For a construction project. The mean and standard deviation of the completion time are 200
days and 6.1 days, respectively. Assume normal distribution and use the value of standard
normal deviate Z = 1.64 for the 95% confidence level. The maximum time required (in days)
for the completion of the project would be __________
Key: (210)
TS − TE
Exp: z=
σ
For 95% confidence level z = 1.64
TE = 200 days
σ = 6.1days
TS = TE + zσ = 200 + 1.64 × 6.1 = 200 + 10.004 = 210.004 days

 ICP–Intensive Classroom Program  eGATE-Live Internet Based Classes DLP  TarGATE-All India Test Series
 Leaders in GATE Preparations  65+ Centers across India
© All rights reserved by Gateforum Educational Services Pvt. Ltd. No part of this booklet may be reproduced or utilized in any form without the written permission.
|CE| GATE-2017-PAPER-02 www.gateforum.com

4. Given that the scope of the construction work is well-defined with all its drawings,
specifications, quantities and estimates, which one of the following types of contract would be
most preferred?
(A) EPC contract (B) Percentage rate contract
(C) Item rate contract (D) Lump sum contract
Key: (D)

5. The divergence of the vector field V = x 2 i + 2y 3 j + z 4 k at x = 1, y = 2, z = 3 is_______


Key: (134)
Exp: Given
v = x 2i + 2y3 J + z 4 k
∂ ∂ ∂
div v = ( x 2 ) + ( 2y3 ) + ( z 4 )
∂x ∂y ∂z
= 2x + 6y 2 + 4 z3
div v (1,2,3) = 2 + 24 + 108 = 134

6. Consider the frame shown in the figure:

If the axial and shear deformations in different members of the frame are assumed to be
negligible, the reduction in the degree of kinematical indeterminacy would be equal to
(A) 5 (B) 6 (C) 7 (D) 8

Key: (B)
Exp:

Total degree of freedom = 3j-R


J= no. of joints = 6
R = no.of reactions = 2+2 = 4
DOF = 3 × 6 − 4 = 18 − 4 = 14
When axial deformation are neglected total axial deformations = 8
Reduction in D k = 14 − 8 = 6

 ICP–Intensive Classroom Program  eGATE-Live Internet Based Classes DLP  TarGATE-All India Test Series
 Leaders in GATE Preparations  65+ Centers across India
© All rights reserved by Gateforum Educational Services Pvt. Ltd. No part of this booklet may be reproduced or utilized in any form without the written permission.
|CE| GATE-2017-PAPER-02 www.gateforum.com

7. As per Noise Pollution (Regulation and Control) Rules 2000 of India, the day time noise limit
for a residential zone, expressed in dB (A) Leq, is
(A) 55 (B) 65 (C) 75 (D) 85
Key: (A)

8. Let G be the specific gravity of soil solids, w the water content in the soil sample, γ w the unit
weight of water, and γ d the dry unit weight of the soil. The equation for the zero air voids line
in a compaction test plot is
Gγ w Gγ w Gw Gw
(A) γ d = (B) γ d = (C) γ d = (D) γ d =
1 + Gw Gw 1+ γw 1− γw

Key: (A)
Exp: From zero air void line
G.γ w
γd =
1 + wG

9. The safety within a roundabout and the efficiency of a roundabout can be increased,
respectively, by
(A) increasing the entry radius and increasing the exit radius
(B) increasing the entry radius and decreasing the exit radius
(C) decreasing the entry radius and increasing the exit radius
(D) decreasing the entry radius and decreasing the exit radius
Key: (C)

10. In a material under a state of plane strain, a 10 × 10 mm square centered at a point gets
deformed as shown in the figure.

y
10 mm

10 mm

π
+ 0.0005 rad
2
0.004 mm

If the shear strain γ xy at this point is expressed as 0.001k (in rad), the value of k is
(A) 0.50 (B) 0.25 (C) -0.25 (D) -0.50
Key: (D)
Exp: Shear strain in an element is positive when the angle between two faces is reduced and
negative when angle is increased.

 ICP–Intensive Classroom Program  eGATE-Live Internet Based Classes DLP  TarGATE-All India Test Series
 Leaders in GATE Preparations  65+ Centers across India
© All rights reserved by Gateforum Educational Services Pvt. Ltd. No part of this booklet may be reproduced or utilized in any form without the written permission.
|CE| GATE-2017-PAPER-02 www.gateforum.com

Here since angle has increased, so shear strain should be negative.

Yxy = −0.0056 rad = 0.001k


So, −0.005 = 0.001 K
⇒ K = −0.50

11. The method of orientation used, when the plane table occupies a position not yet located on
the map, is called as
(A) traversing (B) radiation (C) levelling (D) resection
Key: (D)
Exp: Resection is the method of orientation used when the table occupies a position not yet located
in the map.

12. If a centrifugal pump has an impeller speed of N (in rpm), discharge Q (in m3/s) and the total
head H (in m), the expression for the specific speed Ns of the pump is given by
NQ0.5 NQ0.5 NQ0.5 NQ
(A) N s = (B) N s = (C) N s = (D) N s =
H 0.5 H H 0.75 H 0.75
Key: (C)
N Q N.Q 0.5
Exp: The specific speed of pump ( N s ) = = 0.75
H3 4 H

13. Following observations have been made for the elevation and temperature to ascertain the
stability of the atmosphere:

Elevation (in m) Temperature (in °C)


10 15.5
60 15.0
130 14.3
The atmosphere is classified as
(A) Stable (B) Unstable (C) Neutral (D) Inverse
Key: (C)

 ICP–Intensive Classroom Program  eGATE-Live Internet Based Classes DLP  TarGATE-All India Test Series
 Leaders in GATE Preparations  65+ Centers across India
© All rights reserved by Gateforum Educational Services Pvt. Ltd. No part of this booklet may be reproduced or utilized in any form without the written permission.
|CE| GATE-2017-PAPER-02 www.gateforum.com

∆T 15 − 15.5 0.5 1.00°C


Exp: = =− =−
∆Z 60 − 10 50 100 m
∆T 14.3 − 15.0 0.7 1.00°C
= =− =−
∆Z 130 − 60 70 100 m
So, Neutral

14. During a storm event in a certain period, the rainfall intensity is 3.5 cm/hour and the
Φ − index is 1.5 cm/hour. The intensity of effective rainfall (in cm/hour, up to one decimal
place) for this period is ______
Key: (2)
P−R
Exp: φ − index =
t
3.5 − R
1.5 = ⇒ R = 3.5 − 1.5 = 2cm hr
1

15. For a broad gauge railway track on a horizontal curve of radius R (in m), the equilibrium cant
e required for a train moving at a speed of V (in km per hour) is
V2 V2
(A) e = 1.676 (B) e = 1.315
R R
V2 V2
(C) e = 0.80 (D) e = 0.60
R R
Key: (B)
G.V 2
Exp: Equilibrium cant =
127 R
For Broad guage ( G ) = 1.676
1.676 V 2
e=
127 R
V2
= 0.01319 m
R
V2
e =1.319 cm
R

16. Consider the following simultaneous equations (with c1 and c2 being constants):
3x1 + 2x 2 = c1
4x 1 + x 2 = c 2
The characteristic equation for these simultaneous equations is
(A) λ 2 − 4λ − 5 = 0 (B) λ 2 − 4λ + 5 = 0
(C) λ 2 + 4λ − 5 = 0 (D) λ 2 + 4λ + 5 = 0
Key: (A)
Exp: given systems

 ICP–Intensive Classroom Program  eGATE-Live Internet Based Classes DLP  TarGATE-All India Test Series
 Leaders in GATE Preparations  65+ Centers across India
© All rights reserved by Gateforum Educational Services Pvt. Ltd. No part of this booklet may be reproduced or utilized in any form without the written permission.
|CE| GATE-2017-PAPER-02 www.gateforum.com

3x1 + 2x 2 = c1
4x1 + x 2 = c 2

 3 2   x1   c1 
Matrix Form is    =  
 4 1  x 2  c 2 
AX =B
Characteristic equations of above systems is
A − λI = 0
3−λ 2
=0
4 1− λ

By expanding λ 2 − 4λ − 5 = 0

17. A sheet pile has an embedment depth of 12 m in a homogeneous soil stratum. The coefficient
of permeability of soil is 10 −6 m s. Difference in the water levels between the two sides of the
sheet pile is 4 m. The flow net is constructed with five number of flow lines and eleven
number of equipotential lines. The quantity of seepage (in cm3/s per m, up to one decimal
place) under the sheet pile is _____

Key: (1.6)
Exp: Nf = No. of flow channels = No. of flow lines -1 = 5-1 = 4
Nd = No. of equipotential drops= no. of equipotenial lines-1 = 11-1=10
Nf
q = k.h
Nd
4
= 10−6 × 4 ×
10
= 1.6 × 10−6 m 3 s
q =1.6cm 3 sec.per.m.width

18. While aligning a hill road with a ruling gradient of 6 %, a horizontal curve of radius 50 m is
encountered. The grade compensation (in percentage, up to two decimal places) to be
provided for this case would be______

Key: (1.5)
30 + R 30 + 50
Exp: The grade compensation = = = 1.6%
R 50
75 75
Maximum grade compensation = = =1.5%
R 50
Grade compensation = 1.5

19. Consider the following statements related to the pore pressure parameters, A and B:
P. A always lies between 0 and 1.0

 ICP–Intensive Classroom Program  eGATE-Live Internet Based Classes DLP  TarGATE-All India Test Series
 Leaders in GATE Preparations  65+ Centers across India
© All rights reserved by Gateforum Educational Services Pvt. Ltd. No part of this booklet may be reproduced or utilized in any form without the written permission.
|CE| GATE-2017-PAPER-02 www.gateforum.com

Q. A can be less than 0 or greater than 1.0


R. B always lies between 0 and 1.0
S. B can be less than 0 or greater than 1.0
For these statements, which one of the following options is correct?
(A) P and R (B) P and S (C) Q and R (D) Q and S
Key: (C)

20. Let the characteristic strength be defined as that value, below which not more than 50% of the
results are expected to fall. Assuming a standard deviation of 4 MPa, the target mean strength
(in MPa) to be considered in the mix design of a M25 concrete would be
(A) 18.42 (B) 21.00 (C) 25.00 (D) 31.58
Key: (C)
Exp: For the given condition,
Target mean strength = characteristic strength = 25 MPa

21. The plate load test was conducted on a clayey strata by using a plate of 0.3m × 0.3 m
dimensions, and the ultimate load per unit area for the plate was found to be 180 kPa. The
ultimate bearing capacity (in kPa) of a 2 m wide square footing would be
(A) 27 (B) 180 (C) 1200 (D) 2000
Key: (B)
Exp: In plate load test,
The ultimate bearing capacity does not depends upon width of footing
q uf = q up = 180kPa

22. The infiltration capacity of a soil follows the Horton’s exponential model, f = c1 + c 2 e − kt .
During and experiment, the initial infiltration capacity was observed to be 200 mm/h. After a
long time, the infiltration capacity was reduced to 25 mm/h. If the infiltration capacity after 1
hour was 90 mm/h, the value of the decay rate constant, k (in h-1, up to two decimal places)
is_______
Key: (0.99)
Exp: Horton’s infiltration capacity f = f e + ( fo − f e ) e−∝t
fe = ultimate infiltration capacity = 25 mm/hr
fo = Initial infiltration capacity = 200 mm/hr
f = Infiltration capacity = 90 mm/hr
90 = 25 + ( 200 − 25) e−∝×1
175e −∝ = 65
e −∝ = 0.371
∝= 0.9915; ∝= 0.99 / hour

23. A two-faced fair coin has its faces designated as head (H) and tail (T). This coin is tossed
three times in succession to record the following outcomes: H, H, H. If the coin is tossed one
 ICP–Intensive Classroom Program  eGATE-Live Internet Based Classes DLP  TarGATE-All India Test Series
 Leaders in GATE Preparations  65+ Centers across India
© All rights reserved by Gateforum Educational Services Pvt. Ltd. No part of this booklet may be reproduced or utilized in any form without the written permission.
|CE| GATE-2017-PAPER-02 www.gateforum.com

more time, the probability (up to one decimal place) of obtaining H again, given the previous
realizations of H, H and H, would be_______
Key: (0.5)
Exp: Given first three are already heads. If the coin is tossed again, the outcome does not depends
on previous outcomes.
1
Probability getting head = = = 0.5
2
(or)
probability of first three are heads
1 1 1 1
= P (H × H × H) = × × =
2 2 2 8
Probability of fourth time head is
1 1 1 1 1
= P(H × H × H × H) = × × × =
2 2 2 2 16
Given condition is that ( H.H.H ) is already realized
1
The required probability = 16 = 0.5
1
8

24. The most important type of species involved in the degradation of organic matter in the case
of activated sludge process is
(A) Autotrophs (B) Heterotrophs (C) Prototrophs (D) Photo-autotrophs
Key: (B)

dw
25. Let w = f (x,y), where x and y are functions of t. Then, according to the chain rule, is
dt
equal to
dw dx dw dt ∂w ∂x ∂w ∂y
(A) + (B) +
dx dt dy dt ∂x ∂t ∂y ∂t
∂w dx ∂w dy dw ∂x dw ∂y
(C) + (D) +
∂x dt ∂y dt dx ∂t dy ∂t
Key: (C)
Exp: w = f ( x, y ) where x and y are functions of t.
By Chain Rule
dw ∂w dx ∂w dy
= . + .
dt ∂x dt ∂y dt

Q. No. 26 to 55 Carry Two Marks Each

26. Consider the portal frame shown in the figure and assume the modulus of elasticity
E = 2.5 × 104 MPa and the moment of inertial, I = 8 × 108 mm 4 for all the members of the
frame.

 ICP–Intensive Classroom Program  eGATE-Live Internet Based Classes DLP  TarGATE-All India Test Series
 Leaders in GATE Preparations  65+ Centers across India
© All rights reserved by Gateforum Educational Services Pvt. Ltd. No part of this booklet may be reproduced or utilized in any form without the written permission.
|CE| GATE-2017-PAPER-02 www.gateforum.com

2000 kN
P
1650 kN / m E, I 2m

Q E, I S

E, I 4m

4m

The rotation (in degrees, up to one decimal place) at the rigid joint Q would be_______
Key: (1.003)
2
( 2)
Exp: Moment at θ , M θ = 2000 × 2 − 1650 ×
2
= 700 KN − m anticlockwise

P
2000 KN

1650 KN / m 2m
S
θ

4m

uEI
KθR = = EI,
4
uEI
KθS = = EI
4
Kθ = KθR + Kθ2EI
∵ Mθ = Kθ.θθ
Mθ 700 × 106 × 1000
∴θθ = =
Kθ 2 × 2.5 × 104 × 108
= 0.0175 rad
= 1.003°  1°

 ICP–Intensive Classroom Program  eGATE-Live Internet Based Classes DLP  TarGATE-All India Test Series
 Leaders in GATE Preparations  65+ Centers across India
© All rights reserved by Gateforum Educational Services Pvt. Ltd. No part of this booklet may be reproduced or utilized in any form without the written permission.
|CE| GATE-2017-PAPER-02 www.gateforum.com

27. The radii of relative stiffness of the rigid pavements P and Q are denoted by  p and  Q ,
respectively. The geometric and material properties of the concrete slab and underlying soil
are given below:
Concrete Soil
Pavement Length of Breadth Thickness Modulus Poisson’s Sub grade
Slab of Slab of Slab of Ratio Reaction
Elasticity Modulus
P L B h E µ K
Q L B 0.5h E µ 2K
The ratio (up to one decimal place) of  p  Q is__________
Key: (2)
14
 Eh 3 
Exp: Radius of relative stiffness (  ) =  
12K (1 − µ ) 
2

14
 Eh 3 
 
12K (1 − µ ) 
14
P
2
 2  14
= 14
= 3
 = ( 24 ) =2
Q  E ( 0.5h )
3
 
 ( 0.5 ) 
 
12 × 2K × (1 − µ ) 
2

28. A 1 m wide rectangular channel carries a discharge of 2m 3 s. The specific energy-depth


diagram is prepared for the channel. It is observed in this diagram that corresponding to a
particular specific energy, the subcritical depth is twice the supercritical depth. The subcritical
depth (in meters, up to two decimal places) is equal to _______
Key: (1.069)
Exp: Given sub critical depth = 2 × super critical depth
y= super critical depth
Sub critical depth = 2 y
E1 = E 2
Q12 Q 22
y1 + 2
= y2 +
2g A1 2gA 22
Q2 Q2
2y + 2
= y+
2gB2 ( 2y ) 2g.B2 y 2

Q2  −1 1
y=  + 2
2gB2  ( 2y )
2
y 

 1  1
2
2
y= − +1 ⇒ y3 = 0.153 ⇒ y = 0.534
2 × 9.81 × 1  4  y 2
Super critical depth ( y ) = 0.5347
Sub critical depth = 2y = 2 × 0.5347 = 1.0694 m

 ICP–Intensive Classroom Program  eGATE-Live Internet Based Classes DLP  TarGATE-All India Test Series
 Leaders in GATE Preparations  65+ Centers across India
© All rights reserved by Gateforum Educational Services Pvt. Ltd. No part of this booklet may be reproduced or utilized in any form without the written permission.
|CE| GATE-2017-PAPER-02 www.gateforum.com

29. A hollow circular shaft has an outer diameter of 100 mm and inner diameter of 50 mm. If the
allowable shear stress is 125 MPa, the maximum torque (in kN-m) that the shaft can resist
is_______
Key: (23.009)
T1 τ
Exp: =
J r
τ
T= J
r
125  π
=
 100   32
( Do4 − Di4 )
 
 2 
125 × 2  π
=
100  32
(1004 − 504 )
= 23009711.82 N − m
= 23.009 kN − m

30. Consider the following statements:


P. Walls of one brick thick are measured in square meters.
Q. Walls of one brick thick are measured in cubic meters.
R. No deduction in the brickwork quantity is made for openings in walls up to 0.1 m2 area.
S. For the measurement of excavation from the borrow pit in a fairly uniform ground, dead
men are left at suitable intervals.
For the above statements, the correct option is
(A) P-False; Q-True; R- False: S-True
(B) P-False; Q-True: R-False; S-False
(C) P-True; Q-False; R-True; S- False
(D) P-True; Q-False; R-True; S-True
Key: (D)

31. Following are the statements related to the stress paths in a triaxial testing of soils:
P. Ifσ1 = σ3 , the stress point lies at the origin of the p-q plot
Q. Ifσ1 = σ3 , the stress point lies on the p-axis of the p-q plot.
R. Ifσ1 > σ3 , both the stress points p and q are positive.
For the above statements, the correct combination is
(A) P − False; Q − True; R − True
(B) P − False; Q − True; R − True
(C) P − False; Q − True; R − False
(D) P − True; Q − False; R − False
Key: (A)

 ICP–Intensive Classroom Program  eGATE-Live Internet Based Classes DLP  TarGATE-All India Test Series
 Leaders in GATE Preparations  65+ Centers across India
© All rights reserved by Gateforum Educational Services Pvt. Ltd. No part of this booklet may be reproduced or utilized in any form without the written permission.
|CE| GATE-2017-PAPER-02 www.gateforum.com

Exp: φ
σ1 − σ3
q=
2

σ1 + σ3
p=
2

If σ1 = σ3 ⇒ P = 1 = 2, q = 0
2
∴ lies on P.axis
σ1 + σ3 σ − σ3
If σ1 > σ3 ⇒ P = > 0, q = 1 > 0.
2 2
p & q are positive

32. A simply supported rectangular concrete beam of span 8m has to be prestressed with a force
of 1600kN. The tendon is of parabolic profile having zero eccentricity at the supports. The
beam has to carry an external uniformly distributed load of intensity 30 kN/m. Neglecting the
self-weight of the beam, the maximum dip (in meters, up to two decimal places) of the tendon
at the mid-span to balance the external load should be________
Key: (0.15)
Exp: 30 kN m

8m
e

Maximum moment = Pe
w 2
= Pe
8
w 2 30 × 82
e= = = 0.15 m
8P 8 × 1600

33. An observer standing on the deck of a ship just sees the top of a lighthouse. The top of the
lighthouse is 40 m above the sea level and the height of the observer’s eye is 5 m above the
sea level. The distance (in km, up to one decimal place) of the observer form the lighthouse is
_______
Key: (33)
Exp: 0.0673d12 = 5 ⇒ d1 = 8.62 Km
0.0673d 2 2 = 40 ⇒ d 2 = 24.38 Km

Therefore distance of observer from light house

 ICP–Intensive Classroom Program  eGATE-Live Internet Based Classes DLP  TarGATE-All India Test Series
 Leaders in GATE Preparations  65+ Centers across India
© All rights reserved by Gateforum Educational Services Pvt. Ltd. No part of this booklet may be reproduced or utilized in any form without the written permission.
|CE| GATE-2017-PAPER-02 www.gateforum.com

d = d1 + d 2
= 8.62 + 24.38
= 33 km

1 5  3 7 
34. If A =   and B=   , ABT is equal to
6 2  8 4
38 28  3 40 
(A)   (B)  
32 56   42 8 
 43 27  38 32 
(C)   (D)  
34 50   28 56 
Key: (A)
1 5 3 7
Exp: A=  B= 
 6 2 8 4
 1 5  3 8 
ABT =   
 6 2  7 4 
38 28
= 
32 56 

35. A 2 m long, axially loaded mild steel rod of 8 mm diameter exhibits the load-displacement
( p − δ) behavior as shown in the figure.
14000
12000
10000
Axial Load, P (kN)

8000
6000
4000
2000

0 1 2 3 4 5 6 7 8 9 10
Displacement δ ( mm )

Assume the yield stress of steel as 250 Mpa. The complementary energy (in N-mm) stored in
the bar up to its linear elastic behavior will be_______

Key: (15707. 963)


Exp: Strain up to linear elastic behaviour is
δ 2.5 1
∈= = =
 2000 800

 ICP–Intensive Classroom Program  eGATE-Live Internet Based Classes DLP  TarGATE-All India Test Series
 Leaders in GATE Preparations  65+ Centers across India
© All rights reserved by Gateforum Educational Services Pvt. Ltd. No part of this booklet may be reproduced or utilized in any form without the written permission.
|CE| GATE-2017-PAPER-02 www.gateforum.com

1
Strain energy =× f y ×∈×V
2
1 1  π 
= × 250 × ×  2000 × × 82 
2 800  4 
= 5000π
= 15707.963 Nmm

36. Two cars P and Q are moving in a racing track continuously for two hour. Assume that no
other vehicles are using the track during this time. The expressions relating the distance
travelled d (in km) and time t (in hour) for both the vehicles are given as
P:d = 60t
Q:d = 60t2
Within the first one hour, the maximum space headway would be
(A) 15 km at 30 minutes (B) 15 km at 15 minutes
(C) 30 km at 30 minutes (D) 30 km at 15 minutes
Key: (A)
Exp: P: d = 60t , Q : d = 60t2
Distance at any time t between P and Q is given by d(t) = 60t - 60t2
For space headway to be maximum
d2
dt 2
(d ( t )) = 0
⇒ 60t − 120t = 0
⇒ t =1 2 hour
2
1
so,d (1 2 ) = 60 × 1 2 − 60 ×   = 30 − 15 = 15km
2
So, space headway would be max at t = 30 minutes

37. Consider the following definite integral:


2

I=∫
1
( sin x )
−1

dx
0 1 − x2
The value of the integral is
π3 π3 π3 π3
(A) (B) (C) (D)
24 12 48 64
Key: (A)
2

Exp: given I = ∫
1
( sin x )
−1

dx
0 1 − x2

 ICP–Intensive Classroom Program  eGATE-Live Internet Based Classes DLP  TarGATE-All India Test Series
 Leaders in GATE Preparations  65+ Centers across India
© All rights reserved by Gateforum Educational Services Pvt. Ltd. No part of this booklet may be reproduced or utilized in any form without the written permission.
|CE| GATE-2017-PAPER-02 www.gateforum.com

∵ f n ( x ) f 1 ( x ) dx 
 ∫
3 1

=
(sin x ) −1

3  f n +1 
= 
0  n +1 
1  π   π3
3
1
= ( sin −1 ) − sin −1 0  =   − 0  =
3

3   3  2 
  24

38. A municipal corporation is required to treat 1000 m3/day of water. It is found that an overflow
rate of 20 m/day will produce a satisfactory removal of the discrete suspended particles at a
depth of 3 m. The diameter (in meters, rounded to the nearest integer) of a circular settling
tank designed for the removal of these particles would be______
Key: (8)
Exp: Detention time of a circular tank is given by
d 2 ( 0.011d + 0.785H )
td =
Q
d ( 0.011d + 0.785H )
2
3
So, =
Q 20
d 2 ( 0.011d + 0.785 × 3)3
⇒ =
1000 20
⇒ 0.011d 3 + 2.355d 2 − 150 = 0
⇒ d = 7.83m  8m ( rounded to nearest int eger )

39. For the construction of a highway. A cut is to be made as shown in the figure.

Potential
shear surface

2m

4m

point A

The soil exhibits c ' = 20 kPa, φ = 18°, and the undrained shear strength = 80 kPa. The unit
weight of water is 9.81 kN/m3. The unit weights of the soil above and below the ground water
table are 18 and 20 kN/m3, respectively. If the shear stress at Point A is 50 kPa, the factors of
safety against the shear failure at this point, considering the undrained and drained conditions,
respectively, would be
(A) 1.6 and 0.9 (B) 0.9 and 1.6
(C) 0.6 and 1.2 (D) 1.2 and 0.6

 ICP–Intensive Classroom Program  eGATE-Live Internet Based Classes DLP  TarGATE-All India Test Series
 Leaders in GATE Preparations  65+ Centers across India
© All rights reserved by Gateforum Educational Services Pvt. Ltd. No part of this booklet may be reproduced or utilized in any form without the written permission.
|CE| GATE-2017-PAPER-02 www.gateforum.com

Key: (A)
Exp: Case-I: Undrained condition
Resisting shear stress
F.O.S =
Actual shear stress
80
= = 1.6
50

Case-I: Drained condition

σ tan φ '+ c '


F.O.S =
50
 2 × 18 + 4 ( 20 − 9.81)  × tan18° + 20
=
50
= 0.9

40. A catchment is idealized as a 25 km × 25 km square. It has five rain gauges, one at each
corner and one at the center, as shown in the figure.
G1 G2

G5

G4 G3

During a month, the precipitation at these gauges is measured as G1 = 300 mm, G 2 = 285 mm,
G 3 = 272 mm, G 4 = 290 mm and G 5 = 288 mm. The average precipitation (in mm, up to one
decimal place) over the catchment during this month by using the Thiessen polygon method
is_______
Key: (287.375)
Exp: By thiesen polygon method
ΣPi A i
Pavg =
ΣA i
G1 G2
A1 A2

G5
A5

A4 A3

G4 G3

 ICP–Intensive Classroom Program  eGATE-Live Internet Based Classes DLP  TarGATE-All India Test Series
 Leaders in GATE Preparations  65+ Centers across India
© All rights reserved by Gateforum Educational Services Pvt. Ltd. No part of this booklet may be reproduced or utilized in any form without the written permission.
|CE| GATE-2017-PAPER-02 www.gateforum.com

2
 25  2
A5 =   = 312.5 km
 2
625 − 312.5
A1 = A 2 = A 3 = A 4 = = 78.125 km 2
4
G A + G 2 A 2 + G 3A3 + G 4 A 4 + G 5A5
Pavg = 1 1
A
300 × 78.125 + 285 × 78.125 + 272 × 78.125 + 290 × 78.125 + 288 × 312.5
=
625
= 287.375mm

41. The tangent to the curve represented by y = x ln x is required to have 45° inclination with the
x-axis. The coordinates of the tangent point would be
(A) (1,0) (B) (0,1) (C) (1,1) (D) ( 2, 2 )
Key: (A)
Exp: y = x ln x
dy 1
= x. + ln x
dx x
Given that θ = 450 ⇒ Tanθ = 1
dy
= Tanθ = 1
dx
1
ie ⇒ x. + ln x = 1 ⇒ 1 + ln x = 1
x
x = 1 , y = 0 satisfies
∴ A is correct

42. Water is pumped at a steady uniform flow rate of 0.01 m3/s through a horizontal smooth
circular pipe of 100 mm diameter. Given that the Reynolds number is 800 and g is 9.81 m s2 ,
the head loss (in meters, up to one decimal place) per km length due to friction would be____
Key: (66.67)
fLQ 2
Exp: head loss ( h f ) =
12d 5
64 64
f= = = 0.08
R e 800
2
0.08 × ( 0.01)
hf = 5
× 1000
12 × ( 0.1)
= 66.67 per km

43. Group I given a list of test methods and test apparatus for evaluating some of the properties of
ordinary Portland cement (OPC) and concrete. Group II gives the list of these properties.

 ICP–Intensive Classroom Program  eGATE-Live Internet Based Classes DLP  TarGATE-All India Test Series
 Leaders in GATE Preparations  65+ Centers across India
© All rights reserved by Gateforum Educational Services Pvt. Ltd. No part of this booklet may be reproduced or utilized in any form without the written permission.
|CE| GATE-2017-PAPER-02 www.gateforum.com

Group I Group II
P. Le chatelier test 1. Soundness of OPC
Q. Vee-Bee test 2. Consistency and setting time of OPC
R. Blaine air permeability test 3. Consistency or workability of concrete
S. The Vicat apparatus 4. Fineness of OPC
The correct match of the items in Group I with the items in Group II is
(A) P-1, Q-3, R-4, S-2 (B) P-2, Q-3, R-1, S-4
(C) P-4, Q-2, R-4, S-1 (D) P-1, Q-4, R-2, S-3
Key: (A)
Exp: P - Le chattier test - soundness of OPC
Q - Vee-Bee test – consistency or workability of concrete
R - Blaine air permeability test - Fineness of OPC
S - The vicat apparatus – consistency and setting time of OPC

44. The figure shows a U-tube having a 5 mm × 5 mm square cross-section filled with mercury
(specific gravity = 13.6) up to a height of 20 cm in each limb (open to the atmosphere).

50 cm

Mercury 20 cm

If 5cm 3 of water is added to the right limb, the new height (in cm, up to two decimal places)
of mercury in the LEFT limb will be______
Key: (20.735)
Exp: 1

50cm x
x
20cm
20cm

5cm 3 0.5 × 0.5 × 


 = 20cm
P1 + (1000 × 9.81 × 0.2 ) − (13000 × 9.81 × 2x ) = P2 P1 = P2
1000 × 9.81 × 0.2 = 13600 × 9.81 × 2x
100
x= = 7.35 × 10−3 m = 0.735cm
13600
New height = 20 + x = 20 + 0.735
= 20.735cm
 ICP–Intensive Classroom Program  eGATE-Live Internet Based Classes DLP  TarGATE-All India Test Series
 Leaders in GATE Preparations  65+ Centers across India
© All rights reserved by Gateforum Educational Services Pvt. Ltd. No part of this booklet may be reproduced or utilized in any form without the written permission.
|CE| GATE-2017-PAPER-02 www.gateforum.com

44. The figure shows a U-tube having a 5 mm × 5 mm square cross-section filled with mercury
(specific gravity = 13.6) up to a height of 20 cm in each limb (open to the atmosphere).

50 cm

Mercury 20 cm

If 5cm 3 of water is added to the right limb, the new height (in cm, up to two decimal places)
of mercury in the LEFT limb will be______

Key: (20.735)
Exp: 1
2

x
50cm
20cm x
20cm

5cm 3 0.5 × 0.5 × 


 = 20cm
P1 + (1000 × 9.81 × 0.2 ) − (13000 × 9.81 × 2x ) = P2
P1 = P2
1000 × 9.81 × 0.2 = 13600 × 9.81 × 2x
100
x= = 7.35 × 10−3 m = 0.735cm
13600
New height = 20 + x = 20 + 0.735 = 20.735cm

45. The analysis of a water sample produces the following results:


Ion Milligram per milli- Concentration (mg/L)
equivalent for the ion
Ca 2+ 20.0 60
2+
Mg 12.2 36.6
Na + 23.0 92
K+ 39.1 78.2
C1− 35.5 71
SO24 − 48.0 72
HCO 3− 61.0 122
The total hardness (in mg/L as CaCO3) of the water sample is_____

 ICP–Intensive Classroom Program  eGATE-Live Internet Based Classes DLP  TarGATE-All India Test Series
 Leaders in GATE Preparations  65+ Centers across India
© All rights reserved by Gateforum Educational Services Pvt. Ltd. No part of this booklet may be reproduced or utilized in any form without the written permission.
|CE| GATE-2017-PAPER-02 www.gateforum.com

Key: (300)
 50   50 
Exp: Total hardness as CaCo 3 =  Ca +2 ×  +  Mg 2+ ×
 20   12.2 
 50   50 
=  60 ×  + 36.6 ×
 20   12.2 
= 150 + 150 = 300mg / L as CaCo3 .

46. Consider the three prismatic beams with the clamped supports P, Q, and R as shown in the
figures.
80 N 20 N / m

P Q
E.I
E.I
8m
8m

640 N.m
R
E.I
8m
Given that the modulus of elasticity, E is 2.5 × 104 MPa; and the moment of inertia, I is
8 × 108 mm 4 , the correct comparison of the magnitudes of the shear forces S and the bending
moment M developed at the supports is
(A) Sp < SQ < SR ; M P = MQ = MR
(B) SP = SQ > SR ; M P = M Q > M R
(C) SP < SQ > SR ; M P = M Q = M R
(D) SP < SQ < SR ; M P < M Q < M R

Key: (C)
Exp: 80 kN 20 N m
P R 640 Nm
Q
8m, EI 8m, EI 8m, EI

SP = 80 N SQ = 20 × 8 = 160 N SR = 0
M P = 80 × 8 = 640 Nm 8 M R = 640 Nm
M Q = 20 × 8 × = 640 Nm
2
SP < SQ > SR MP = MQ = MR

47. Consider a square-shaped area ABCD on the ground with its centre at M as shown in the
figure. Four concentrated vertical load of P=5000 kN are applied on this area, at each corner.

 ICP–Intensive Classroom Program  eGATE-Live Internet Based Classes DLP  TarGATE-All India Test Series
 Leaders in GATE Preparations  65+ Centers across India
© All rights reserved by Gateforum Educational Services Pvt. Ltd. No part of this booklet may be reproduced or utilized in any form without the written permission.
|CE| GATE-2017-PAPER-02 www.gateforum.com
P

B
P
P
M
A C

4m P 4m

The vertical stress increment (in kPa, up to one decimal place) due to these loads according to
the Boussinesq’s equation, at a point 5 m right below M, is______
Key: (191.36)
Exp: P

P P

4m rp 4m

4
r= = 2 2 = 2.82
2
52
 
 
3Q  1 
σx = 4 ×
2πz 2   r  2 
1 +   
 z 
52
 
 
3 × 5000  1 
=4×
2π × 52   2.82 2 
1 +   
  5  
=191.36 kPa

48. For a given water sample, the ratio between BOD5-day, 20°C and the ultimate BOD is 0.68. The
value of the reaction rate constant k (on base e)( in day-1, up to two decimal places) is ______
Key: (0.23)
Exp: y = L o 1 − e − K D × t 

y
= 0.68
Lo
y
= 1 − e − K D ×t
Lo
0.68 = 1 − e − K D ×5
e − K D ×5 = 1 − 0.68 = 0.32
K D = 0.2276  0.23 / day
 ICP–Intensive Classroom Program  eGATE-Live Internet Based Classes DLP  TarGATE-All India Test Series
 Leaders in GATE Preparations  65+ Centers across India
© All rights reserved by Gateforum Educational Services Pvt. Ltd. No part of this booklet may be reproduced or utilized in any form without the written permission.
|CE| GATE-2017-PAPER-02 www.gateforum.com

49. Two identical concrete piles having the plan dimensions 50 cm × 50 cm are driven into a
homogeneous sandy layer as shown in the figures. Consider the bearing capacity factor Nq for
φ = 30°as24.

Dry sand
Saturated sand
20m γ = 18 kN m 3 20m
γ sat = 19 kN m 3
φ = 30°
Q P1 φ = 30°
Q P2

If QP1 and QP2 represent the ultimate point bearing resistance of the piles under dry and
submerged conditions, respectively, which one of the following statements is correct ?
(A) Q P1 > Q P 2 by about100% (B) Q P1 < Q P2 by about100%
(C) Q P1 > Q P2 by about 5% (D) Q P1 < Q P 2 about 5%
Key: (A)
50. The culturable command area of a canal is 10,000 ha. The area grows only two crops-rice in
the Kharif season and wheat in the Rabi season. The design discharge of the canal is based on
the rice requirements, which has an irrigated area of 2500 ha, base period of 150 days and
delta of 130 cm. The maximum permissible irrigated area (in ha) for wheat, with a base period
of 120 days and delta of 50 cm, is_______
Key: 5200
Exp: For rice,
8.64B 8.6 × 150
Duty = = = 996.923
∆ 1.30
A 2500
Q= = = 2.5077
D 996.923
For wheat
A 8.64 × 120
Q= ⇒ A = Q.D = 2.5077 × = 5200ha
D 0.5

51. The composition of a municipal solid waste sample is given below:


Component Percent by Mass Moisture Energy content
Content (kJ/kg, on as –
discarded basis)
Food Waste 20 70 2500
Paper 10 4 10000
Cardboard 10 4 8000
Plastics 10 1 14000
Garden Trimmings 40 60 3500
Wood 5 20 14000
Tin Cans 5 2 100

 ICP–Intensive Classroom Program  eGATE-Live Internet Based Classes DLP  TarGATE-All India Test Series
 Leaders in GATE Preparations  65+ Centers across India
© All rights reserved by Gateforum Educational Services Pvt. Ltd. No part of this booklet may be reproduced or utilized in any form without the written permission.
|CE| GATE-2017-PAPER-02 www.gateforum.com

The difference between the energy content of the waste sample calculated on dry basis and as-
discarded basis (in kJ/kg) would be______
Key: (3870)
Exp: Dry weight = Total weight –m.c

Component %mass M.C Dry wt (%) Energy Total


content Energy
Food waste 20 70 6 2500 50,000
Paper 10 4 9.6 10,000 1,00,000
Cardboard 10 4 9.6 8,000 80,000
Plastic 10 1 9.9 14,000 140000
Garden trim 40 60 16 3,500 140000
Wood 5 20 4 14,000 70000
Tin cans 5 2 4.9 100 500
60 580500 KJ

580500
Unit energy = = 5805kJ kg
100
Moisture content = 100 − 60 = 40%
100 × 5805
Energy on dry basis = = 9675kJ kg
100 − 40
So,different ofenergy = 9675 − 5805 = 3870 kJ kg

52. Consider the following second-order differential equation:


y"− 4y '+ 3y = 2t − 3t 2
The particular solution of the differential equations is
(A) −2 − 2t − t 2 (B) −2t − t 2 (C) 2t − 3t 2 (D) −2 − 2t − 3t 2

Key: (A)
Exp: given y11 − 4y1 + 3y = 2t − 3t 2
⇒ ( D2 − 4D + 3) y = ( 2t − 3t 2 )
By the definition of particular solution
1
yp =
D 2 − 4D + 3
( 2t − 3t 2 )
⇒ ( D2 − 4D + 3) y p = 2t − 3t 2
verifying options, option (a) satisfies,
(D 2
− 4D + 3)( −2 − 2t − t 2 )
= −2 + 8 + 8t − 6 − 6t − 3t 2 = 2t − 3t 2

 ICP–Intensive Classroom Program  eGATE-Live Internet Based Classes DLP  TarGATE-All India Test Series
 Leaders in GATE Preparations  65+ Centers across India
© All rights reserved by Gateforum Educational Services Pvt. Ltd. No part of this booklet may be reproduced or utilized in any form without the written permission.
|CE| GATE-2017-PAPER-02 www.gateforum.com

∴ (A) is correct
Alternate solution:
Given
y" − 4y' + 3y = 2t − 3t 2
(D 2
− 4D + 3) y = 2t − 3t 2

1
Particular solution = y p =
D 2 − 4D + 3
( 2t − 3t 2 )
 −1 1 
 2 
= + 2  ( 2t − 3t 2 )
 D −1 D − 3
 
1 1 1   3 2
=  − 2 t − t 
2 1 − D 3 − D   2 
 −1 1 D   3 
−1

= (1 − D ) − 1 −    t − t 2 
 3 3    2 

 3  −1  D D  3 
2
= (1 + D + D2 + D3 ...)  t − t 2  1 + + + ...   t − t 2 
 2  3 3 9  2 
 3  −1  3 1 − 3t 1 
=  t − t 2 + 1 − 3t − 3  t − t 2 + + ( −3) 
 2  3  2 3 9 
−3 t 1 1 t 1
= −2 − 2t − t 2 − + t 2 − + + = −2 − 2t − t 2
2 3 2 9 3 9

53. Two plates of 8 mm thickness each are connected by a fillet weld of 6 mm thickness as shown
in the figure.
6

50 mm
100 mm
P P

100 mm

The permissible stresses in the plate and the weld are 150 MPa and 110 MPa, respectively.
Assuming the length of the weld shown in the figure to be the effective length, the permissible
load P (in KN) is______
Key: (60)
Exp:

50 mm P
P

100 mm

 ICP–Intensive Classroom Program  eGATE-Live Internet Based Classes DLP  TarGATE-All India Test Series
 Leaders in GATE Preparations  65+ Centers across India
© All rights reserved by Gateforum Educational Services Pvt. Ltd. No part of this booklet may be reproduced or utilized in any form without the written permission.
|CE| GATE-2017-PAPER-02 www.gateforum.com

Maximum load taken by plate = 150 × 50 × 8 = 60KN

= k.s left.τ
Maximum load taken by weld = 0.7 × 6 × (100 + 100 + 50 ) × 110
= 115.5KN

So, permissible load = min. Of {60KN,115.5KN}


= 60KN
54. Two prismatic beams having the same flexural rigidity of 1000 kN-m2 are shown in the
figures. 6 kN / m

δ1
4m
120 kN

δ2
1m 1m

If the mid-span deflections of these beams are denoted by δ1 and δ2 (as indicated in the
figures). The correct option is
(A) δ1 = δ2 (B) δ1 < δ2 (C) δ1 > δ2 (D) δ1 >> δ 2
Key: (A)
Exp: 6 kN m
4m
δ1

130kN

1m 1m
δ2

5 wL4 5 6 × 44 1
δ1 = = × = = 0.02m
384 EI 384 1000 50
wL3 130 × 23 13
δ2 = = = = 0.02m
48EI 48 × 1000 600
δ2 = δ1

 ICP–Intensive Classroom Program  eGATE-Live Internet Based Classes DLP  TarGATE-All India Test Series
 Leaders in GATE Preparations  65+ Centers across India
© All rights reserved by Gateforum Educational Services Pvt. Ltd. No part of this booklet may be reproduced or utilized in any form without the written permission.
|CE| GATE-2017-PAPER-02 www.gateforum.com

55. Two towers A and B, standing vertically on a horizontal ground, appear in a vertical aerial
photograph as shown in the figure.

B
A
P

The length of the image of the tower A on the photograph is1.5 cm and of the tower B is 2.0
cm. The distance of the top the tower A (as shown by the arrowhead) is 4.0 cm and the
distance of the top of the tower B is 6.0 cm, as measured form the principal point p of the
photograph. If the height of the tower B is 80 m, the height (in meters) of the tower A is
______
Key: (90)
Exp:
For Tower B,

Radial distance of top of tower, r = 6 cm

Length of image, d=2 cm

Height of tower, h 2 =80 m

r.h 2
∵d =
H − h1
r.h 2 6 × 80
⇒ H − h1 = = = 240 m
d 2

For Tower A,

r.h 2
∵d =
H − h1
r = 4cm,d = 1.5 cm,H − h1 = 240 m
⇒ h 2 = 90 m

General Aptitude
Q. No. 1 - 5 Carry One Mark Each

1. There was no doubt that their work was thorough.


Which of the words below is closest in meaning to the underlined word above?
(A) Pretty (B) Complete (C) Sloppy (D) Haphazard
Key: (B)

 ICP–Intensive Classroom Program  eGATE-Live Internet Based Classes DLP  TarGATE-All India Test Series
 Leaders in GATE Preparations  65+ Centers across India
© All rights reserved by Gateforum Educational Services Pvt. Ltd. No part of this booklet may be reproduced or utilized in any form without the written permission.
|CE| GATE-2017-PAPER-02 www.gateforum.com

2. Four cards lie on a table. Each card has a number printed on one side and a colour on the
other. The faces visible on the cards are 2, 3, red, and blue.
Proposition: If a card has an even value on one side, then its opposite face is red.
The cards which MUST be turned over to verify the above proposition are
(A) 2, Red (B) 2, 3, red (C) 2, blue (D) 2, red, blue
Key: (C)
Exp: In order to verify this propositions we have to turn to card 2 and blue from given 4 cards as
proposition says it has even an one side opposite is red. Vice-verse might or might not be true
so, answer (C) as all other options are eliminated.

3. Two dice are thrown simultaneously. The probability that the product of the numbers
appearing on the top faces of the dice is a perfect square is
(A) 1/9 (B) 2/9 (C) 1/3 (D) 4/9
Key: (B)
8 2
Exp: Required probability= =
36 9
x +2 2x + 4
 16   3
4. What is the value of x when 81 ×   ÷  = 144?
 25  5
(A) 1 (B) -1 (C) -2 (D)Cannot be determined
Key: (B)
x +2
 16 
 
Exp:
 25  × 81 = 144
2x + 4
( )
3
5
2x + 4 2x
4 4 4
4
⇒  × 81 = 144 ⇒   . 4 × 81 = 144
3 3 3
2x 9
⇒ 4 ( 3) = ⇒ x = −1
16

5. The event would have been successful if you ______ able to come.
(A) are (B) had been (D) have been (D) would have been
Key: (B)

Q. No. 6- 10 Carry Two Marks Each

6. P,Q,R,S,T and U are seated around a circular table. R is seated two places to the right of Q.P
is seated three places to the left of R. S is seated opposite U. If P and U now switch seats.
Which of the following must necessarily be true?
(A) P is immediately to the right of R
(B) T is immediately to the left of P

 ICP–Intensive Classroom Program  eGATE-Live Internet Based Classes DLP  TarGATE-All India Test Series
 Leaders in GATE Preparations  65+ Centers across India
© All rights reserved by Gateforum Educational Services Pvt. Ltd. No part of this booklet may be reproduced or utilized in any form without the written permission.
|CE| GATE-2017-PAPER-02 www.gateforum.com

(C) T is immediately to the left of P or P is immediately to the right of Q


(D) U is immediately to the right of R or P is immediately to the left of T
Key: (C)
Exp: Q Q
S U

R P
R P ( OR )

U T S
T
P ↔ U Now, P & U switch seats; then there are 2 possibilities
Q
S Q
P

R U R U

T S
T P

7. Bhaichung was observing the pattern of people entering and leaving a car service centre.
There was a single window where customers were being served. He saw that people
inevitably came out of the centre in the order that they went in. However, the time they spent
inside seemed to vary a lot: some people came out in a matter of minutes while for others it
took much longer.
From this, what can one conclude?
(A) The centre operates on a first-come-first-served basis but with variable service times,
depending on specific customer needs.
(B) Customers were served in an arbitrary order since they took varying amounts of time for
service completion in the centre.
(C) Since some people came out within a few minutes of entering the centre. The system is
likely to operate on a last-come-first-served basis.
(D) Entering the centre early ensured that one would have shorter service times and most
people attempted to do this.
Key: (A)
Exp: People coming out in the same order in which they enter indicates that the centre operates on
a first come first serve basis.

8. A map shows the elevations of Darjeeling, Gangtok, Kalimpong, pelling, and Siliguri.
Kalimpong is at a lower elevation than Gangtok. Pelling is at a lower elevation than Gangtok.
Pelling is at a higher elevation that siliguri. Darjeeling is at a higher elevation than Gangtok.
Which of the following statements can be inferred from the paragraph above?
i. Pelling is at a higher elevation than Kalimpong
ii. Kalimpong is at a lower elevation than Darjeeling

 ICP–Intensive Classroom Program  eGATE-Live Internet Based Classes DLP  TarGATE-All India Test Series
 Leaders in GATE Preparations  65+ Centers across India
© All rights reserved by Gateforum Educational Services Pvt. Ltd. No part of this booklet may be reproduced or utilized in any form without the written permission.
|CE| GATE-2017-PAPER-02 www.gateforum.com

iii. Kalimpong is at a higher elevation than siliguri


iv. Siliguri is at a lower elevation than Gangtok
(A) Only ii (B) Only ii and iii (C) Only ii and iv (D) Only iii and iv
Key: (C)

9. Budhan covers a distance of 19 km in 2 hours by cycling one fourth of the time and walking
the rest. The next day he cycles (at the same speed as before) for half the time and walks the
rest (at the same speed as before) and covers 26 km in 2 hours. The speed in km/h at which
Budhan walks is
(A) 1 (B) 4 (C) 5 (D) 6
Key: (D)
Exp: Let cycling speed=C; and walking speed=W
1 3
C   + W   = 19 ....... (1)
 
2 2
C + W = 26 ....... ( 2 )
On solving (1) & (2), we get W=6 km/hr

10. The points in the graph below represent the halts of a lift for duration of 1 minute, over a
period of 1 hour.

Which of the following statements are correct?


i. The elevator never moves directly from any non-ground floor to another non-ground
floor over the one hour period
ii. The elevator stays on the fourth floor for the longest duration over the one hour period
(A) Only i (B) Only ii (C) Both i and ii (D) Neither i nor ii

Key: (D)
Exp: (i). is incorrect as its has more directly.
(ii). is incorrect as it stayed for maximum duration on ground floor.

 ICP–Intensive Classroom Program  eGATE-Live Internet Based Classes DLP  TarGATE-All India Test Series
 Leaders in GATE Preparations  65+ Centers across India
© All rights reserved by Gateforum Educational Services Pvt. Ltd. No part of this booklet may be reproduced or utilized in any form without the written permission.

You might also like